239
MANAGEMENT ADVISORY SERVICES STANDARD COSTS AND VARIANCE ANALYSIS NORMAL COSTING One-Way Variance Actual Overhead 25. Hayward applies overhead at $5 per machine hour. During March it worked 10,000 hours and overapplied overhead by $3,000. Actual overhead was (E) a. $53,000. c . $47,000. b. $50,000. d. none of the above. L & H 10e Applied Overhead 36. Gonzalez Company uses the equation $520,000 + $2 per direct labor hour to budget manufacturing overhead. Gonzalez has budgeted 150,000 direct labor hours for the year. Actual results were 150,000 direct labor hours and $817,500 total manufacturing overhead. The total overhead applied for the year is (E) a. $300,000. c. $817,500. b. $520,000. d . $820,000. L & H 10e 37. Gonzales Company uses the equation $540,000 + $2 per direct labor hour to budget manufacturing overhead. Gonzalez has budgeted 160,000 direct labor hours for the year. Actual results were 160,000 direct labor hours and $857,500 total manufacturing overhead. The total overhead variance for the year is (E) a . $2,500 favorable. c. $2,500 unfavorable. b. $12,500 favorable. d. Some other number. D, L & H 9e Over-Applied 24. Spooner applies overhead based on direct labor cost. It had budgeted manufacturing overhead of $50,000 and budgeted direct labor of $25,000. Actual overhead was $52,500, actual labor cost was $27,000. Overhead was (E) a . overapplied by $1,500. c. overapplied by $2,500. b. overapplied by $2,000. d. underapplied by $2,000. L & H 10e 37. Gonzalez Company uses the equation $520,000 + $2 per direct labor hour to budget manufacturing overhead. Gonzalez has budgeted 150,000 direct labor hours for the year. Actual results were 150,000 direct labor hours and $817,500 total manufacturing overhead. The total overhead variance for the year is (E) a . $2,500 favorable. c. $2,500 unfavorable. b. $12,500 favorable. d. some other number. L & H 10e 44. Antaya Company uses the equation $375,000 + $1.20 per direct labor hour to budget manufacturing overhead. Antaya has budgeted 75,000 direct labor hours for the year. Actual results were 81,000 direct labor hours, $388,000 fixed overhead, and $98,600 variable overhead. The total overhead variance for the year is (E) a. $14,400 c. $37,200 b . $15,600 d. $30,000. L & H 10e CMA EXAMINATION QUESTIONS Page 1 of 239

P01 - Standard Costs and Variance Analysis

Embed Size (px)

DESCRIPTION

Tan

Citation preview

MANAGEMENT ADVISORY SERVICES STANDARD COSTS AND VARIANCE ANALYSIS

NORMAL COSTINGOne-Way VarianceActual Overhead25. Hayward applies overhead at $5 per machine hour. During March it worked 10,000 hours and

overapplied overhead by $3,000. Actual overhead was (E)a. $53,000. c. $47,000.b. $50,000. d. none of the above. L & H 10e

Applied Overhead36. Gonzalez Company uses the equation $520,000 + $2 per direct labor hour to budget

manufacturing overhead. Gonzalez has budgeted 150,000 direct labor hours for the year. Actual results were 150,000 direct labor hours and $817,500 total manufacturing overhead. The total overhead applied for the year is (E)a. $300,000. c. $817,500.b. $520,000. d. $820,000. L & H 10e

37. Gonzales Company uses the equation $540,000 + $2 per direct labor hour to budget manufacturing overhead. Gonzalez has budgeted 160,000 direct labor hours for the year. Actual results were 160,000 direct labor hours and $857,500 total manufacturing overhead. The total overhead variance for the year is (E)a. $2,500 favorable. c. $2,500 unfavorable.b. $12,500 favorable. d. Some other number. D, L & H 9e

Over-Applied24. Spooner applies overhead based on direct labor cost. It had budgeted manufacturing

overhead of $50,000 and budgeted direct labor of $25,000. Actual overhead was $52,500, actual labor cost was $27,000. Overhead was (E)a. overapplied by $1,500. c. overapplied by $2,500.b. overapplied by $2,000. d. underapplied by $2,000. L & H 10e

37. Gonzalez Company uses the equation $520,000 + $2 per direct labor hour to budget manufacturing overhead. Gonzalez has budgeted 150,000 direct labor hours for the year. Actual results were 150,000 direct labor hours and $817,500 total manufacturing overhead. The total overhead variance for the year is (E)a. $2,500 favorable. c. $2,500 unfavorable.b. $12,500 favorable. d. some other number. L & H 10e

44. Antaya Company uses the equation $375,000 + $1.20 per direct labor hour to budget manufacturing overhead. Antaya has budgeted 75,000 direct labor hours for the year. Actual results were 81,000 direct labor hours, $388,000 fixed overhead, and $98,600 variable overhead. The total overhead variance for the year is (E)

a. $14,400 c. $37,200b. $15,600 d. $30,000. L & H 10e

1. Nil Co. uses a predetermined factory O/H application rate based on direct labor cost. For the year ended December 31, Nil’s budgeted factory O/H was $600,000, based on a budgeted volume of 50,000 direct labor hours, at a standard direct labor rate of $6 per hour. Actual factory O/H amounted to $620,000, with actual direct labor cost of $325,000. For the year, over-applied factory O/H was (M)a. $20,000 c. $30,000b. $25,000 d. $50,000 AICPA 1186 II-29

30. Nil Co. uses a predetermined overhead rate based on direct labor cost to apply manufacturing overhead to jobs. For the year ended December 31, Nil's estimated manufacturing overhead was $600,000, based on an estimated volume of 50,000 direct labor hours, at a direct labor rate of $6.00 per hour. Actual manufacturing overhead amounted to $620,000, with actual direct labor cost of $325,000. For the year, manufacturing overhead was: (M)A. overapplied by $20,000. C. overapplied by $30,000.B. underapplied by $22,000. D. underapplied by $30,000. G & N 10e

2. Watson Company uses a predetermined factory overhead application rate based on direct labor cost. Watson's budgeted factory overhead was $756,000 based on a budgeted volume of 60,000 direct labor hours, at a standard direct labor rate of $7.20 per hour. Actual factory overhead amounted to $775,000 with actual direct labor cost of $450,000 for the year ended December 31. How much was Watson's overapplied factory overhead? (M)A. $12,500 C. $19,000B. $18,000 D. $37,000 Gleim

Under-Applied3. The Kelley Company uses a predetermined overhead rate of $9 per direct labor hour to apply

overhead. During the year, 30,000 direct labor hours were worked. Actual overhead costs for the year were $240,000. The overhead variance is (E)a. $27,000 overlapped c. $30,000 underappliedb. $26,670 underapplied d. $24,000 overapplied H & M

38. Bonds Company uses the equation $300,000 + $1.75 per direct labor hour to budget manufacturing overhead. Bonds has budgeted 125,000 direct labor hours for the year. Actual results were 110,000 direct labor hours, $297,000 fixed overhead, and $194,500 variable overhead. The total overhead variance for the year is (E)a. $1,000. c. $35,000b. $48,000. d. $36,000. L & H 10e

CMA EXAMINATION QUESTIONS Page 1 of 142

MANAGEMENT ADVISORY SERVICES STANDARD COSTS AND VARIANCE ANALYSIS

29. Malcolm Company uses a predetermined overhead rate based on direct labor hours to apply manufacturing overhead to jobs.On September 1, the estimates for the month were:

Manufacturing overhead $17,000Direct labor hours 13,600

During September, the actual results were:Manufacturing overhead $18,500Direct labor hours 12,000

The cost records for September will show: G & N 10eA. Overapplied overhead of $1,500. C. Overapplied overhead of $3,500.B. Underapplied overhead of $1,500. D. Underapplied overhead of $3,500.

Actual Direct Labor Hours30. Hoyt Company applies overhead at $6 per direct labor hour. In March Hoyt incurred overhead

of $144,000. Under-applied overhead was $6,000. How many direct labor hours did TYV work? (E)a. 25,000 c. 23,000b. 24,000 d. 22,000 D, L & H 9e

. MNO Company applies overhead at P5 per direct labor hour. In March 2001, MNO incurred overhead of P120,000. Under-applied overhead was P5,000. How many direct labor hours did MNO work? (E)A. 25,000 C. 24,000B. 22,000 D. 23,000 RPCPA 1001

4. Margolos, Inc. ends the month with a volume variance of $6,360 unfavorable. If budgeted fixed factory O/H was $480,000, O/H was applied on the basis of 32,000 budgeted machine hours, and budgeted variable factory O/H was $170,000, what were the actual machine hours (AH) for the month? (M)a. 32,424 c. 31,687b. 32,000 d. 31,576 J.B. Romal

41. Pinnini Co. uses a predetermined overhead rate based on direct labor hours to apply manufacturing overhead to jobs. Last year, Pinnini Company incurred $225,000 in actual manufacturing overhead cost. The Manufacturing Overhead account showed that overhead was overapplied $14,500 for the year. If the predetermined overhead rate was $5.00 per direct labor hour, how many hours did the company work during the year? (M)A. 45,000 hours C. 42,100 hoursB. 47,900 hours D. 44,000 hours G & N 10e

42. Parsons Co. uses a predetermined overhead rate based on direct labor hours to apply manufacturing overhead to jobs. Last year Parsons incurred $250,000 in actual manufacturing overhead cost. The Manufacturing Overhead account showed that overhead was overapplied in the amount of $12,000 for the year. If the predetermined overhead rate was $8.00 per direct labor hour, how many hours were worked during the year? (M)A. 31,250 hours C. 32,750 hoursB. 30,250 hours D. 29,750 hours G & N 10e

Three-Way Variance Variable Overhead Spending Variance39. Bonds Company uses the equation $300,000 + $1.75 per direct labor hour to budget

manufacturing overhead. Bonds has budgeted 125,000 direct labor hours for the year. Actual results were 110,000 direct labor hours, $297,000 fixed overhead, and $194,500 variable overhead. The variable overhead spending variance for the year is (E)a. $2,000. c. $47,000.b. $3,000. d. $48,000. L & H 10e

29. Baxter Corporation's master budget calls for the production of 5,000 units of its product monthly. The master budget includes indirect labor of $144,000 annually; Baxter considers indirect labor to be a variable cost. During the month of April, 4,500 units of product were produced, and indirect labor costs of $10,100 were incurred. A performance report utilizing flexible budgeting would report a spending variance for indirect labor of: (E)A. $1,900 unfavorable. C. $1,900 favorable.B. $700 favorable. D. $700 unfavorable. G & N 10e

Fixed Overhead Budget Variance40. Bonds Company uses the equation $300,000 + $1.75 per direct labor hour to budget

manufacturing overhead. Bonds has budgeted 125,000 direct labor hours for the year. Actual results were 110,000 direct labor hours, $297,000 fixed overhead, and $194,500 variable overhead. The fixed overhead budget variance for the year is (E)a. $2,000. c. $47,000.b. $3,000. d. $48,000. L & H 10e

46. Antaya Company uses the equation $375,000 + $1.20 per direct labor hour to budget manufacturing overhead. Antaya has budgeted 75,000 direct labor hours for the year. Actual results were 81,000 direct labor hours, $388,000 fixed overhead, and $98,600 variable overhead. The fixed overhead budget variance for the year is (E)a. $13,000. c. $17,000. b. $47,000 d. $30,000. L & H 10e

CMA EXAMINATION QUESTIONS Page 2 of 142

MANAGEMENT ADVISORY SERVICES STANDARD COSTS AND VARIANCE ANALYSIS

Volume Variance. ABC Company uses the equation P300,000 + P1.75 per direct labor hour to budget

manufacturing overhead. ABC has budgeted 125,000 direct labor hours for the year. Actual results were 110,000 direct labor hours, P297,000 fixed overhead, and P194,500 variable overhead. What is the fixed overhead volume variance for the year? (M)A. P35,000 unfavorable. C. P2,000 favorable.B. P36,000 unfavorable. D. P3,000 favorable. RPCPA 1001

41. Bonds Company uses the equation $300,000 + $1.75 per direct labor hour to budget manufacturing overhead. Bonds has budgeted 125,000 direct labor hours for the year. Actual results were 110,000 direct labor hours, $297,000 fixed overhead, and $194,500 variable overhead. The fixed overhead volume variance for the year is (E)a. $39,000. c. $33,000.b. $3,000. d. $36,000. L & H 10e

47. Antaya Company uses the equation $375,000 + $1.20 per direct labor hour to budget manufacturing overhead. Antaya has budgeted 75,000 direct labor hours for the year. Actual results were 81,000 direct labor hours, $388,000 fixed overhead, and $98,600 variable overhead. The fixed overhead volume variance for the year is (E)a. $1,400. c. $15,600.b. $13,000. d. $30,000. L & H 10e

Fixed OH Budget Variance, Volume Variance & Variable OH Spending VarianceOver-(Under) Applied Overhead34. Waldorf had a $10,000 unfavorable fixed overhead budget variance, a $6,000 unfavorable

variable overhead spending variance, and a $2,000 favorable volume variance. The total overhead was (E)a. $14,000 overapplied. c. $18,000 overapplied.b. $14,000 underapplied. d. $18,000 underapplied. L & H 10e

Variable Overhead Spending Variance35. Bacon had a $18,000 unfavorable volume variance, a $5,000 unfavorable fixed overhead

budget variance, and $12,000 total under-applied overhead. The variable overhead spending variance was (E)a. $11,000 favorable. c. $11,000 unfavorable.b. $1,000 favorable. d. $23,000 unfavorable. D, L & H 9e

Fixed Overhead Budget Variance32. Daly had a $9,000 favorable volume variance, a $7,500 unfavorable variable overhead

spending variance, and $6,000 total overapplied overhead. The fixed overhead budget variance was (E)

a. $4,500 favorable. c. $4,500 unfavorable.b. $8,000 favorable. d. $8,000 unfavorable. L & H 10e

141.ABC had a P28,000 favorable volume variance, a P25,000 unfavorable variable overhead spending variance, and P12,000 total overapplied overhead. The fixed overhead budget variance wasa. P9,000 favorable. c. P9,000 unfavorable.b. P26,000 favorable. d. P26,000 unfavorable. Pol Bobadilla

49. Rhoda had a $2,000 favorable volume variance, a $7,000 unfavorable variable overhead spending variance, and $3,000 total underapplied overhead. The fixed overhead budget variance was (E)a. $2,000 favorable. c. $2,000 unfavorable.b. $8,000 favorable. d. $8,000 unfavorable. L & H 10e

16. XYZ had an $8,000 unfavorable volume variance, a $11,500 unfavorable variable overhead spending variance, and $1,500 total underapplied overhead. The fixed overhead budget variance was (E)a. $18,000 favorable. c. $17,500 unfavorable.b. $21,000 favorable. d. $21,000 unfavorable. L & H 10e

Volume Variance50. Katrina Inc. had a $30,000 favorable fixed overhead budget variance, a $44,000 unfavorable

variable overhead spending variance, and $44,000 total underapplied overhead. The volume variance was (E)a. $30,000 overapplied. c. $58,000 overapplied.b. $30,000 underapplied. d. $58,000 underapplied. L & H 10e

33. Acme had a $6,000 favorable fixed overhead budget variance, a $2,500 unfavorable variable overhead spending variance, and $1,000 total overapplied overhead. The volume variance was (E)a. $4,500 overapplied. c. $2,500 overapplied.b. $4,500 underapplied. d. $2,500 underapplied. L & H 10e

142.Acme had a P22,000 favorable fixed overhead budget variance, a P15,000 unfavorable variable overhead spending variance, and P2,000 total overapplied overhead. The volume variance wasa. P13,000 overapplied c. P5,000 overappliedb. P13,000 underapplied d. P5,000 underapplied Pol Bobadilla

CMA EXAMINATION QUESTIONS Page 3 of 142

MANAGEMENT ADVISORY SERVICES STANDARD COSTS AND VARIANCE ANALYSIS

ComprehensiveQuestions 38 through 41 are based on the following information. D, L & H 9eAlcatraz Company uses the equation $400,000 + $1.75 per direct labor hour to budget manufacturing overhead. Alcatraz has budgeted 125,000 direct labor hours for the year. Actual results were 110,000 direct labor hours, $397,000 fixed overhead, and $194,500 variable overhead.

38. The total overhead variance for the year is (M)a. $2,000 c. $47,000b. $3,000 d. $48,000

39. The variable overhead spending variance for the year is (M)a. $2,000 c. $47,000b. $3,000 d. $48,000

40. The fixed overhead budget variance for the year is (E)a. $2,000 c. $47,000b. $3,000 d. $48,000

41. The fixed overhead volume variance for the year is (M)a. $2,000 c. $47,000b. $3,000 d. $48,000

Questions 44 through 47 are based on the following information. D, L & H 9eHughes Company uses the equation $375,000 + $1.20 per direct labor hour to budget manufacturing overhead. Hughes had budgeted 75,000 direct labor hours for the year. Actual results were 81,000 direct labor hours, $397,000 fixed overhead, and $94,500 variable overhead. 44. The total overhead variance for the year is (M)

a. $2,700 c. $22,000b. $10,700 d. $30,000

45. The variable overhead spending variance for the year is (M)a. $2,700 c. $22,000b. $10,700 d. $30,000

46. The fixed overhead budget variance for the year is (E)a. $2,700 c. $22,000b. $10,700 d. $30,000

47. The fixed overhead volume variance for the year is (M)

a. $1,400 c. $15,600b. $13,000 d. $30,000

ACTIVITY-BASED COSTINGQuestions 116 thru 120 are based on the following information. HorngrenMunoz, Inc. produces a special line of plastic toy racing cars. Munoz, Inc. produces the cars in batches. To manufacture a batch of the cars, Munoz, Inc. must set up the machines and molds. Setup costs are batch-level costs because they are associated with batches rather than individual units of products. A separate Setup Department is responsible for setting up machines and molds for different styles of car.Setup overhead costs consist of some costs that are variable and some costs that are fixed with respect to the number of setup-hours. The following information pertains to June 2004.

Actual Amounts Static-budget AmountsUnits produced and sold 15,000 11,250Batch size (number of units per batch) 250 225Setup-hours per batch 5 5.25Variable overhead cost per setup-hour $40 $38Total fixed setup overhead costs $14,400 $14,000

5. Calculate the efficiency variance for variable setup overhead costs. (D)a. $1,500 unfavorable c. $975 unfavorableb. $525 favorable d. $1,500 favorable

6. Calculate the spending variance for variable setup overhead costs. (D)a. $1,500 unfavorable c. $975 unfavorableb. $525 favorable d. $1,500 favorable

7. Calculate the flexible-budget variance for variable setup overhead costs. (M)a. $1,500 unfavorable c. $975 unfavorableb. $525 favorable d. $1,500 favorable

8. Calculate the spending variance for fixed setup overhead costs. (E)a. $3,200 unfavorable c. $3,600 unfavorableb. $400 unfavorable d. $400 favorable

9. Calculate the production-volume variance for fixed setup overhead costs.(M)a. $3,200 unfavorable c. $3,600 unfavorableb. $400 unfavorable d. $400 favorable

CMA EXAMINATION QUESTIONS Page 4 of 142

MANAGEMENT ADVISORY SERVICES STANDARD COSTS AND VARIANCE ANALYSIS

STATIC BUDGET VARIANCEQuestions 48 thru 50 are based on the following information. HorngrenAbernathy Corporation used the following data to evaluate their current operating system. The company sells items for $10 each and used a budgeted selling price of $10 per unit.

Actual BudgetedUnits sold 92,000 units 90,000 unitsVariable costs $450,800 $432,000Fixed costs $ 95,000 $100,000

10. What is the static-budget variance of revenues?a. $20,000 favorable c. $2,000 favorableb. $20,000 unfavorable d. $2,000 unfavorable

11. What is the static-budget variance of variable costs? (E)a. $1,200 favorable c. $20,000 favorableb. $18,800 unfavorable d. $1,200 unfavorable

12. What is the static-budget variance of operating income? (E)a. $3,800 favorable c. $6,200 favorableb. $3,800 unfavorable d. $6,200 unfavorable

Questions 51 thru 53 are based on the following information. HorngrenBates Corporation used the following data to evaluate their current operating system. The company sells items for $10 each and used a budgeted selling price of $10 per unit.

Actual BudgetedUnits sold 495,000 units 500,000 unitsVariable costs $1,250,000 $1,500,000Fixed costs $ 925,000 $ 900,000

13. What is the static-budget variance of revenues?a. $50,000 favorable c. $5,000 favorableb. $50,000 unfavorable d. $5,000 unfavorable

14. What is the static-budget variance of variable costs?a. $200,000 favorable c. $250,000 favorableb. $50,000 unfavorable d. $250,000 unfavorable

15. What is the static-budget variance of operating income? (E)a. $175,000 favorable c. $225,000 favorableb. $195,000 unfavorable d. $325,000 unfavorable

FLEXIBLE BUDGET VARIANCETotal Manufacturing CostFlexible Budget16. Aebi Corporation currently produces cardboard boxes in an automated process. Expected

production per month is 20,000 units, direct-material costs are $0.60 per unit, and manufacturing overhead costs are $9,000 per month. Manufacturing overhead is allocated based on units of production. What is the flexible budget for 10,000 and 20,000 units, respectively? (E)a. $10,500; $16,500 c. $15,000; $21,000b. $10,500; $21,000 d. none of the above Horngren

17. Hemberger Corporation currently produces baseball caps in an automated process. Expected production per month is 20,000 units, direct material costs are $1.50 per unit, and manufacturing overhead costs are $23,000 per month. Manufacturing overhead is allocated based on units of production. What is the flexible budget for 10,000 and 20,000 units, respectively? (E)a. $26,500; $41,500 c. $38,000; $53,000b. $26,500; $53,000 d. none of the above Horngren

*. Based on normal capacity operations, Sta. Ana Company employs 25 workers in its Refining Department, working 8 hours a day, 20 days per month at a wage rate of P6 per hour. At normal capacity, production in the department is 5,000 units per month. Indirect materials average P0.25 per direct labor hour; indirect labor cost is 12½% of direct labor cost; and other overhead are P0.15 per direct labor hour.The flexible budget at the normal capacity activity level follows:

Direct materials P 4,000Direct labor 24,000Fixed factory overhead 1,200Indirect materials 1,000Indirect labor 3,000Other overhead 600Total P 33,800Cost per unit P 6.76

The total production cost for one month at 80% capacity is (M)a. P20,760 c. P27,280b. P21,500 d. P30,160 RPCPA 1082

CMA EXAMINATION QUESTIONS Page 5 of 142

MANAGEMENT ADVISORY SERVICES STANDARD COSTS AND VARIANCE ANALYSIS

Questions 36-38 are based on the following information: G & N 10eBarrick Company has established a flexible budget for manufacturing overhead based on direct labor-hours. Total budgeted costs at 200,000 direct labor-hours are as follows:

Variable costs (total):Packing supplies $120,000Indirect labor $180,000Fixed costs (total):Utilities $100,000Rent $ 40,000Insurance $ 20,000

36. The flexible budget for factory overhead would show that the variable factory overhead cost per direct labor-hour is: A. $1.80. C. $0.90.B. $1.50. D. $0.60.

37. At an activity level of 170,000 direct labor-hours, the flexible budget for factory overhead would show the budgeted amount for utilities as: A. $ 85,000. C. $160,000.B. $140,000. D. $100,000.

38. If Barrick Company plans to operate at 190,000 direct labor-hours during the next period, the flexible budget would show indirect labor costs of: A. $171,000. C. $114,000.B. $180,000. D. $270,000.

Questions 39-41 are based on the following information: G & N 10eWicks Company has established a flexible budget for manufacturing overhead based on direct labor-hours. Budgeted costs at 100,000 direct labor-hours are as follows:

Variable costs (total):Packing supplies $70,000Indirect labor $90,000Fixed costs (total):Utilities $50,000Rent $20,000Insurance $10,000

39. The flexible budget for factory overhead would show that the variable overhead per direct labor-hour is: A. $1.90. C. $0.90.B. $1.60. D. $0.70.

40. At an activity level of 70,000 direct labor-hours, the flexible budget would show the budgeted amount for utilities as: A. $35,000. C. $80,000. B. $70,000. D. $50,000.

41. If Wicks Company plans to operate at 90,000 direct labor-hours during the next period, the flexible budget would show indirect labor costs of: A. $144,000. C. $90,000. B. $63,000. D. $81,000.

Manufacturing Cost Variance18. A defense contractor for a government space project has incurred $2,500,000 in actual design

costs to date for a guidance system whose total budgeted design cost is $3,000,000. If the design phase of the project is 60% complete, what is the amount of the contractor's current overrun or savings on this design work? (M)A. $300,000 savings. C. $500,000 savings.B. $500,000 overrun. D. $700,000 overrun. CIA 0596 III-87

19. A manufacturing firm planned to manufacture and sell 100,000 units of product during the year at a variable cost per unit of $4.00 and a fixed cost per unit of $2.00. The firm fell short of its goal and only manufactured 80,000 units at a total incurred cost of $515,000. The firm’s manufacturing cost variance was (D)a. $85,000 favorable. c. $5,000 favorable. b. $35,000 unfavorable. d. $5,000 unfavorable. CMA 1293 3-25

Operating IncomeQuestions 116 and 117 are based on the following information. CMA 0695 3-26 & 27Clear Plus, Inc. manufactures and sells boxes of pocket protectors. The static master budget and the actual results for May 1995 appear below.

Actual Static BudgetUnit sales 12,000 10,000Sales $132,000 $100,000Variable costs of sales 70,800 60,000 Contribution margin 61,200 40,000Fixed costs 32,000 30,000 Operating income $ 29,200 $ 10,000

20. The operating income for Clear Plus, Inc. using a flexible budget for May 1995 isa. $12,000 c. $30,000b. $19,200 d. $18,000

CMA EXAMINATION QUESTIONS Page 6 of 142

MANAGEMENT ADVISORY SERVICES STANDARD COSTS AND VARIANCE ANALYSIS

21. Which one of the following statements concerning Clear Plus, Inc.’s actual results for May 1995 is correct?a. The flexible budget variance is $8,000 favorable.b. The sales price variance is $32,000 favorable.c. The sales volume variance is $8,000 favorable.d. The fixed cost flexible budget variance is $4,000 favorable.

Comprehensive

1 . REQUIRED: The overapplied factory O/H for the period.DISCUSSION: (C) Nil Co. applies factory O/H using a predetermined O/H rate, based on direct labor cost. O/H was budgeted for $600,000 based on a budgeted labor cost of $300,000 ($6 x 50,000 hrs.) Thus, $2 of O/H was applied for each $1 of labor. Given actual labor cost of $325,000, $650,000 (2 x $325,000) was applied during the period. Actual O/H was $620,000, so $30,000 ($650,000 – $620,000) was over-applied.Answer (A) is incorrect because $20,000 is the difference between budgeted and actual factory O/H. Answer (B) is incorrect because $25,000 is the difference between budgeted direct labor costs and actual direct labor costs. Answer (D) is incorrect because $50,000 is the difference between the applied O/H and the budgeted amount.

2 . Answer (A) is correct. Overhead was budgeted at $756,000 based on a budgeted labor cost of $432,000 ($7.20 x 60,000 hours). Thus, $1.75 of overhead was applied for each $1 of labor cost. Given actual labor costs of $450,000, $787,500 ($1.75 x $450,000) of overhead was applied during the period. Actual overhead was $775,000, so $12,500 ($787,500 - $775,000) was overapplied. Answer (B) is incorrect because $18,000 is the difference between budgeted direct labor cost at 60,000 direct labor hours and actual direct labor cost ($450,000 - $432,000). Answer (C) is incorrect because $19,000 is the difference between budgeted overhead ($756,000) and the actual overhead ($775,000). Answer (D) is incorrect because $37,000 is the sum of the difference between budgeted overhead ($756,000) and the actual overhead ($775,000) and the difference between the applied overhead ($787,500) and the budgeted overhead ($756,000).

3.Applied overhead ($9 x 30,000)$270,000Actual overhead 240,000Overapplied overhead$ 30,0004 . Answer (D) is correct. The volume variance (VV) arises from the difference between

budgeted fixed O/H and the fixed O/H applied at the standard rate based on the standard input allowed for actual output. The O/H rate is $15 per machine hour ($480,000 ÷ 32,000). VV = Budgeted Fixed O/H - Applied Fixed O/H$6,360 = $480,000 - ($15 x AH) $15 x AH = $480,000 - $6,360 AH = $473,640 ÷ $15 AH = 31,576.Answer (A) is incorrect because

THE FOLLOWING INFORMATION APPLIES TO QUESTIONS 74 THROUGH 77. HorngrenThe actual information pertains to the month of August. As part of the budgeting process Alloway’s Fencing Company developed the following static budget for August. Alloway is in the process of preparing the flexible budget and understanding the results.

Actual Results Flexible Budget Static BudgetSales volume (in units) # 20,000 # 25,000Sales revenues $1,000,000 $ $1,250,000Variable costs 512,000 $ 600,000Contribution margin 488,000 $ 650,000Fixed costs 458,000 $ 450,000Operating profit $ 30,000 $ $ 200,000

22. The flexible budget will report __________ for variable costs. (E)a. $512,000 c. $480,000b. $600,000 d. $640,000

23. The flexible budget will report __________ for the fixed costs. (E)a. $458,000 c. $360,000b. $450,000 d. $572,500

24. The flexible-budget variance for variable costs is (E)a. $32,000 unfavorable. c. $32,000 favorable.b. $120,000 unfavorable. d. $120,000 favorable.

32,425 assumes the volume variable was favorable. Answer (B) is incorrect because the actual machine hours can be found by using the following equation: Volume Variance = Budgeted Fixed O/H - Applied Fixed O/H. The applied fixed O/H is equal to the O/H rate multiplied by the actual hours. The O/H rate is found by dividing the budgeted O/H ($480,000) by the budgeted hours (32,000). Actual machine hours are 31,576. Answer (C) is incorrect because 32,000 equals the budgeted machine hours.

5 . [(11,250 / 225) x 5.25 x $40] – [(11,250 / 250) x 5 x $40] = $1,500 (U)

6 . (11,250 / 225) x 5.25 x ($38 - $40) = $525 (F)

7 . $1,500 (U) + $525 (F) = $975 (U)

8 . $14,000 - $14,400 = $400 (U)

CMA EXAMINATION QUESTIONS Page 7 of 142

MANAGEMENT ADVISORY SERVICES STANDARD COSTS AND VARIANCE ANALYSIS

77. The PRIMARY reason for low operating profits was (M)a. the variable-cost variance.b. increased fixed costs.c. a poor management accounting system.d. lower sales volume than planned.

STATIC BUDGET VS. FLEXIBLE BUDGETTHE FOLLOWING INFORMATION APPLIES TO QUESTIONS 78 THROUGH 82. HorngrenPeters’ Company manufacturers tires. Some of the company's data was misplaced. Use the following information to replace the lost data:

ActualResults

Flexible-Budget Variances

FlexibleBudget

Sales-Volume Variances

Static Budget

Units sold #225,000 #225,000 #206,250Revenues $84,160 $2,000 F (A) $2,800 U (B)Variable costs (C) $400 U $31,720 $4,680 F $36,400

Fixed costs $16,560 $1,720 F $18,280 0 $18,280Operating income $35,480 (D) $32,160 (E) $30,280

78. What amounts are reported for revenues in the flexible-budget (A) and the static-budget (B), respectively? (E)a. $82,160; $79,360 c. $84,960; $88,960b. $82,160; $84,960 d. $84,960; $83,360

79. What are the actual variable costs (C)? (E)a. $36,400 c. $31,320b. $32,120 d. $27,040

80. What is the total flexible-budget variance (D)? (E)a. $120 unfavorable c. $680 favorableb. $0 d. $3,320 favorable

81. What is the total sales-volume variance (E)? (E)a. $7,480 unfavorable c. $1,880 favorableb. $2,800 unfavorable d. $7,480 favorable

82. What is the total static-budget variance? (E)a. $5,200 favorable c. $1,880 unfavorableb. $3,320 favorable d. $1,880 favorable

STANDARD COSTUnit Cost*. Based on normal capacity operations, Sta. Ana Company employs 25 workers in its Refining

Department, working 8 hours a day, 20 days per month at a wage rate of P6 per hour. At normal capacity, production in the department is 5,000 units per month. Indirect materials average P0.25 per direct labor hour; indirect labor cost is 12½% of direct labor cost; and other overhead are P0.15 per direct labor hour.The flexible budget at the normal capacity activity level follows:

Direct materials P 4,000Direct labor 24,000Fixed factory overhead 1,200Indirect materials 1,000Indirect labor 3,000Other overhead 600Total P 33,800Cost per unit P 6.76

The cost per unit at 60% capacity is (M)a. P6.00 c. P6.82b. P6.50 d. P6.92 RPCPA 1082

Standard Total Cost6. Cascade Company, which has a $3 standard cost per unit and budgeted production at 1,000

units, actually produced 1,200 units. Total standard cost for the period isa. $3,000b. $3,600c. An amount that cannot be determined without knowing the variances for the period.d. None of the above. D, L & H 9E

MATERIALS VARIANCEStandard DM Cost per Unit21. RTW Co. manufactures a “one-size-fits-all” ready-to-wear outfit and uses a standard cost

system. Each unit of finished outfit contains two yards of fabric that cost P75 per yard. Based on experience, a 20% loss on fabric input is incurred. For each unit of outfit, the standard materials cost is (E)a. P150.00 c. P187.50b. P180.00 d. P200.00 RPCPA 1094

25. Dahl Co. uses a standard costing system in connection with the manufacture of a “one size fits all” article of clothing. Each unit of finished product contains 2 yards of direct material. However, a 20% direct material spoilage calculated on input quantities occurs during the

CMA EXAMINATION QUESTIONS Page 8 of 142

MANAGEMENT ADVISORY SERVICES STANDARD COSTS AND VARIANCE ANALYSIS

manufacturing process. The cost of the direct material is $3 per yard. The standard direct material cost per unit of finished product is (M)a. $4.80 c. $7.20b. $6.00 d. $7.50 AICPA adapted

26. Fleece Company uses a standard-costing system in relation to its manufacture of scarves. Each finished scarf contains 1.5 yards of direct materials. However, a 25% direct materials spoilage, which is calculated based on input quantities, occurs during the manufacturing process. The cost of the direct materials is $2.00 per yard. The standard direct materials cost per unit of finished product is (M)A. $2.25 C. $3.75B. $3.00 D. $4.00 Gleim

*. Hankies Unlimited has a signature scarf for ladies that is very popular. Certain production and marketing data are indicated below:

Cost per yard of cloth P36.00

10 . (92,000 units x $10) - (90,000 units x $10) = $20,000 F

11 . $450,800 - $432,000 = $18,800 U

12.Actual ResultsStatic BudgetStatic-budget VarianceUnits

sold92,00090,000Revenues$920,000$900,000$20,000 FVariable costs450,800432,00018,800 UContribution margin$469,200$468,000$1,200 FFixed costs95,000100,000(5,000) FOperating

income$374,200$368,000$6,200 F13 . (495,000 units x $10) - (500,000 units x $10) = $50,000 U

14 . $1,250,000 - $1,500,000= $250,000 F

15.Actual ResultsStatic BudgetStatic-budget VarianceUnits

sold495,000500,000Revenues$4,950,000$5,000,000$(50,000) UVariable costs1,250,0001,500,000(250,000) FContribution margin$3,700,000$3,500,000$200,000 FFixed

costs925,000900,00025,000 UOperating income$2,775,000$2,600,000$175,000 F16.

10,000 units20,000 unitsMaterials ($0.60)$ 6,000$12,000Machinery 9,000 9,000$15,000$21,000

17.10,000 units20,000 unitsMaterials ($1.50)$15,000$30,000Machinery 23,00023,000$38,000$53,000

Allowance for rejected scarf 5% of productionYards of cloth needed per scarf 0.475 yardAirfreight from supplier P0.60/yardMotor freight to customers P0.90 /scarfPurchase discounts from supplier 3%Sales discount to customers 2%

The allowance for rejected scarf is not part of the 0.475 yard of cloth per scarf. Rejects have no market value. Materials are used at the start of production.Calculate the standard cost of cloth per scarf that Hankies Unlimited should use in its cost sheets. (D)a. P16.87 c. P18.21b. P17.76 d. P17.30 RPCPA 0594

8. Alejo Company is a chemical manufacturer that supplies industrial users. The company plans to introduce a new chemical solution and needs to develop a standard product cost for this new solution.The new chemical solution is made by combining a chemical compound (nyclin) and a solution (salex), boiling the mixture, adding a second compound (protet), and bottling the resulting solution in 20-liter containers. The initial mix, which is 20 liters in volume consists of 24 kilograms of nyclyn and 19.2 liters of salex. A 20% reduction in volume occurs during the boiling process. The solution is then cooled slightly before 10 kilograms of protet are added: the addition of protet does not affect the total liquid volume.The purchase prices of the raw materials used in the manufacture of this new chemical solution are as follows:

Nyclyn P15.00 per kilogramSalex P21.00 per kilogramProtet P28.00 per kilogram

The total standard materials cost of 20 liters of the product is (D)A. P1,043,20 C. P1,304.00B. P834.56 D. P1,234.00 Pol Bobadilla

Material Price VarianceBased on Quantity Purchased27. Garland Company uses a standard cost system. The standard for each finished unit of

product allows for 3 pounds of plastic at $0.72 per pound. During December, Garland bought 4,500 pounds of plastic at $0.75 per pound, and used 4,100 pounds in the production of 1,300 finished units of product. What is the materials purchase price variance for the month of December? (E)a. $117 unfavorable. c. $135 unfavorable.b. $123 unfavorable. d. $150 unfavorable. CMA Samp Q3-11

CMA EXAMINATION QUESTIONS Page 9 of 142

MANAGEMENT ADVISORY SERVICES STANDARD COSTS AND VARIANCE ANALYSIS

18. The standard price of a material is $2 per pound. The company bought 2,000 pounds at $1.90 per pound and used 1,700 pounds. Standard use was 1,800 pounds. The material price variance was (E)a. $170 favorable. c. $200 favorable.b. $180 favorable. d. $400 favorable. D, L & H 9e

38. The standard usage for raw materials is 5 pounds at $4 per pound. ABC Company spent $13,940 in purchasing 3,400 pounds. ABC used 3,150 pounds to produce 600 units of finished product. The material price variance is (E)a. $340 unfavorable. c. $600 unfavorable.b. $400 unfavorable. d. $1,340 unfavorable. D, L & H 9e

40. Last month 75,000 pounds of direct material were purchased and 71,000 pounds were used. If the actual purchase price per pound was $0.50 more than the standard purchase price per pound, then the material price variance was: (E)a. $2,000 F. c. $37,500 U.b. $37,500 F. d. $35,500 U. G & N 9e

42. PRECISION Instruments established a standard cost for raw materials at P25 per unit. During the period just ended, a total of 10,000 units were purchased of which 50% was at P24.70 each, 20% was at P24.90 each, and the balance was at P25.60 each. The raw materials cost variance is a favorable (an unfavorable) (M)a. P100 c. P(100)b. P900 d. P(900) RPCPA 1097

43. The following materials standards have been established for a particular product:Standard quantity per unit of output .. 8.3 gramsStandard price ........................ $19.15 per gram

The following data pertain to operations concerning the product for the last month:Actual materials purchased ............ 7,500 gramsActual cost of materials purchased .... $141,375Actual materials used in production ... 7,100 gramsActual output ......................... 700 units

What is the materials price variance for the month? (E)a. $2,250 F c. $24,317 Ub. $7,540 U d. $7,660 U G & N 9e

38. Perkins Company, which has a standard cost system, had 500 pounds of raw material X in its inventory at June 1, purchased in May for $1.20 per pound and carried at a standard cost of $1.00 per pound. The following information pertains to raw material X for the month of June:

Actual pounds purchased 1,400Actual pounds used 1,500Standard pounds allowed for actual production 1,300Standard cost per pound $1.00Actual cost per pound $1.10

The unfavorable materials purchase price variance for raw material X for June was: A. $ 0. C. $140.B. $130. D. $150. G & N 10e

40. The following materials standards have been established for a particular product: Standard quantity per unit of output 5.3 metersStandard price $17.20 per meter

The following data pertain to operations concerning the product for the last month:Actual materials purchased 8,100 metersActual cost of materials purchased $141,345Actual materials used in production 7,600 metersActual output 1,400 units

What is the materials price variance for the month? A. $3,141 U C. $8,600 UB. $2,025 U D. $8,725 U G & N 10e

Questions 1 & 2 are based on the following information. H & MThe Max Company has developed the following standards for one of its products:

Direct materials: 15 pounds x $16 per poundDirect labor: 4 hours x $24 per hourVariable manufacturing overhead: 4 hours x $14 per hour

The following activity occurred during the month of October:Materials purchased: 10,000 pounds costing $170,000Materials used: 7,200 poundsUnits produced: 500 unitsDirect labor: 2,300 hours at $23.60/hourActual variable manufacturing overhead: $30,000

The company records materials price variances at the time of purchase.

28. The variable standard cost per unit is (E)a. $392 c. $296b. $336 d. $152

CMA EXAMINATION QUESTIONS Page 10 of 142

MANAGEMENT ADVISORY SERVICES STANDARD COSTS AND VARIANCE ANALYSIS

29. The direct materials price variance is (E)a. $50,000 favorable c. $10,000 unfavorableb. $50,000 unfavorable d. $10,000 favorable

Based on Quantity Purchased and Used30. The standard direct material cost to produce a unit of Lem is 4 meters of material at $2.50 per

meter. During May 2001, 4,200 meters of material costing $10,080 were purchased and used to produce 1,000 units of Lem. What was the material price variance for May 2001? (E)a. $400 favorable. c. $80 unfavorable.b. $420 favorable. d. $480 unfavorable. AICPA 1195

31. Data regarding Mill Company's direct materials costs is as follows:Actual unit cost $2.00Standard unit cost 2.20Actual quantity purchased and used 28,000 unitsStandard units of materials per unit of finished goods 3 unitsActual output of finished goods 9,000 units

What is the direct materials price variance? (E)A. $2,800 favorable. C. $5,600 favorable.B. $5,600 unfavorable. D. $2,200 unfavorable. Gleim

Actual Price47. ALPHA Co. uses a standard cost system. Direct materials statistics for the month of May,

19x7 are summarize below:Standard unit price P90.00Actual units purchased 40,000Standard units allowed for actual production 36,250Materials price variance- favorable P6,000

What was the actual purchase price per unit? (M)a. P75.00 c. P88.50b. P85.89 d. P89.85 RPCPA 0597

32. Information on Kennedy Company's direct material costs follows:Standard price per pound of raw materials ....... $3.60Actual quantity of raw materials purchased ...... 1,600 poundsStandard quantity allowed for actual production.. 1,450 poundsMaterials purchase price variance--favorable .... $ 240

What was the actual purchase price per unit, rounded to the nearest penny? (M)a. $3.06. c. $3.45.b. $3.11. d. $3.75. AICPA adapted

41. The Wright Company has a standard costing system. The following data are available for September:

Actual quantity of direct materials purchased 25,000 poundsStandard price of direct materials $2 per poundMaterial price variance $2,500 unfavorable

The actual price per pound of direct materials purchased in September is: A. $1.85. C. $2.10.B. $2.00. D. $2.15. G & N 10e

Actual Purchases17. Cascade Company bought 10,000 pounds of material and used 9,500. The material price

variance was $300 unfavorable and the standard price per pound is $3. The cost of materials purchased was (E)a. $28,200 c. $29,700b. $28,800 d. $30,300 D, L & H 9e

Standard Price33. During December, 6,000 pounds of raw materials were purchased at a cost of $16 per pound.

If there was an unfavorable direct materials price variance of $6,000 for December, the standard cost per pound must be (E)a. $17 c. $15b. $16 d. $14 H & M

42. The standard cost card for one unit of a certain finished product shows the following:Standard Quantity or

Hours Standard Price or Rate

Direct materials 10 pounds $ ? per poundDirect labor 2.5 hours $16 per hourVariable manufacturing overhead 1.5 hours $10 per hour

If the total standard variable cost for one unit of finished product is $85, then the standard price per pound for direct materials is: (M)A. $1.74. C. $5.90.B. $4.60. D. $3.00. G & N 10e

Material Quantity Variance37. Cox Company's direct material costs for the month of January were as follows:

Actual quantity purchased 18,000 kilogramsActual unit purchase price $ 3.60 per kilogramMaterials price variance – unfavorable (based on purchases) $ 3,600Standard quantity allowed for actual production 16,000 kilograms

CMA EXAMINATION QUESTIONS Page 11 of 142

MANAGEMENT ADVISORY SERVICES STANDARD COSTS AND VARIANCE ANALYSIS

Actual quantity used 15,000 kilogramsFor January there was a favorable direct material quantity variance of: (M)a. $3,360. c. $3,400.b. $3,375. d. $3,800. G & N 9e

Materials Quantity VarianceActual Quantity Used34. During April, 80,000 units were produced. The standard quantity of material allowed per unit

was 2 pounds at a standard cost of $5 per pound. If there was a favorable usage variance of $40,000 for April, the actual quantity of materials used must have been (M)a. 168,000 pounds c. 84,000 poundsb. 152,000 pounds d. 76,000 pounds H & M

35. During April, 20,000 units were produced. The standard quantity of material allowed per unit was 4 pounds at a standard cost of $6 per pound. If there was an unfavorable usage variance of $30,000 for April, the actual quantity of materials used must be (M)a. 85,000 pounds c. 21,250 poundsb. 75,000 pounds d. 18,750 pounds H & M

36. ChemKing uses a standard costing system in the manufacture of its single product. The 35,000 units of raw material in inventory were purchased for $105,000, and two units of raw material are required to produce one unit of final product. In November, the company produced 12,000 units of product. The standard allowed for material was $60,000, and there was an unfavorable quantity variance of $2,500. The units of material used to produce November output totaled (M)a. 12,000 units c. 23,000 unitsb. 12,500 units d. 25,000 units CMA 1293 3-23

Actual Price44. The Swenson Company has a standard costing system. The following data are available for

June:Actual quantity of direct materials purchased 35,000 poundsStandard price of direct materials $4 per poundMaterial price variance $7,000 unfavorableMaterial quantity variance $4,200 favorable

The actual price per pound of direct materials purchased in June is: (M)A. $3.92. C. $4.08.B. $4.32. D. $4.20. G & N 10e

Standard Quantity Allowed45. The Fletcher Company uses standard costing. The following data are available for October:

Actual quantity of direct materials used 23,500 poundsStandard price of direct materials $2 per poundMaterial quantity variance $1,000 favorable

The standard quantity of material allowed for October production is: (M)a. 23,000 lbs. c. 24,500 lbs.b. 24,000 lbs. d. 25,000 lbs. G & N 9e

43. The Cox Company uses standard costing. The following data are available for April:Actual quantity of direct materials used 12,200 gallonsStandard price of direct materials $4 per gallonMaterial quantity variance $2,000 unfavorable

The standard quantity of material allowed for April production is: (M)A. 14,200 gallons. C. 11,700 gallons.B. 12,700 gallons. D. 10,200 gallons. G & N 10e

Standard Unit Cost37. ChemKing uses a standard costing system in the manufacture of its single product. The

35,000 units of raw material in inventory were purchased for $105,000, and two units of raw material are required to produce one unit of final product. In November, the company produced 12,000 units of product. The standard allowed for material was $60,000, and there was an unfavorable quantity variance of $2,500. ChemKing’s standard price for one unit of material is (E)a. $2.50 c. $5.00b. $3.00 d. $6.00 CMA 1293 3-22

Materials Quantity Variance39. The standard usage for raw materials is 5 pounds at $4.00 per pound. ABC Company spent

$13,940 in purchasing 3,400 pounds. ABC used 3,150 pounds to produce 600 units of finished product. The material quantity variance is (E)a. $340 unfavorable. c. $600 unfavorable.b. $400 unfavorable. d. $1,340 unfavorable. D, L & H 9e

CMA EXAMINATION QUESTIONS Page 12 of 142

MANAGEMENT ADVISORY SERVICES STANDARD COSTS AND VARIANCE ANALYSIS

36. Home Company manufactures tables with vinyl tops. The standard material cost for the vinyl used per Type-R table is $7.80 based on six square feet of vinyl at a cost of $1.30 per square foot. A production run of 1,000 tables in January resulted in usage of 6,400 square feet of vinyl at a cost of $1.20 per square foot, a total cost of $7,680. The quantity variance resulting from the above production run was: (E)A. $120 favorable. C. $520 unfavorable.B. $480 unfavorable. D. $640 favorable. G & N 10e

38. A company uses a standard cost system to account for its only product. The materials standard per unit was 4 lbs. at $5.10 per lb. Operating data for April were as follows:

Material used 7,800 lbs.Cost of material used $40,950Number of finished units produced 2,000

The material usage variance for April was: (E)A. $1,020 favorable C. $1,170 unfavorableB. $1,050 favorable D. $1,200 unfavorable CIA adapted

41. During March, Younger Company's direct material costs for product T were as follows:Actual unit purchase price $6.50 per meterStandard quantity allowed for actual production 2,100 metersQuantity purchased and used for actual production 2,300 metersStandard unit price $6.25 per meter

Younger's material quantity variance for March was: (E)a. $1,250 unfavorable. c. $1,300 unfavorable.b. $1,250 favorable. d. $1,300 favorable. AICPA adapted

42. The following materials standards have been established for a particular product:Standard quantity per unit of output .. 1.7 metersStandard price ........................ $19.80 per meter

The following data pertain to operations concerning the product for the last month:Actual materials purchased ............ 5,800 metersActual cost of materials purchased .... $113,680Actual materials used in production ... 5,100 metersActual output ......................... 3,200 units

What is the materials quantity variance for the month? (E)a. $13,720 U c. $13,860 Ub. $6,732 F d. $6,664 F G & N 9e

39. The following materials standards have been established for a particular product: Standard quantity per unit of output 4.6gramsStandard price $15.05 per gram

The following data pertain to operations concerning the product for the last month:Actual materials purchased 3,100 gramsActual cost of materials purchased $44,020Actual materials used in production 2,400 gramsActual output 300 units

What is the materials quantity variance for the month? (E)A. $9,940 U C. $14,484 UB. $15,351 U D. $10,535 U G & N 10e

Direct Materials Price Variance39. ChemKing uses a standard costing system in the manufacture of its single product. The

35,000 units of raw material in inventory were purchased for $105,000, and two units of raw material are required to produce one unit of final product. In November, the company produced 12,000 units of product. The standard allowed for material was $60,000, and there was an unfavorable quantity variance of $2,500. The materials price variance for the units used in November was (M)a. $2,500 unfavorable. c. $12,500 unfavorable.b. $11,000 unfavorable. d. $3,500 favorable. CMA 1293 3-24

39. Information on Fleming Company's direct material costs follows:Actual amount of direct materials used ...... 20,000 poundsActual direct material costs ................ $40,000Standard price of direct materials .......... $2.10 per poundDirect material efficiency variance--favorable $3,000

What was the company's direct material price variance? (M)a. $1,000 favorable. c. $2,000 favorable.b. $1,000 unfavorable. d. $2,000 unfavorable. AICPA adapted

*. Information on the direct material costs of Bernal Manufacturing Corp. is as follows:Actual direct material costs P 44,000Actual units of direct material used 22,000Standard price per unit of direct material P2.20Direct material efficiency variance-unfavorable P2,800

What was Bernal’s direct material price variance? (M)a. P4,400 favorable. c. P5,600 favorable.b. P4,400 unfavorable. d. P5,600 unfavorable. RPCPA 1079

40. Information on Duke Co.'s direct material costs for May is as follows:CMA EXAMINATION QUESTIONS Page 13 of 142

MANAGEMENT ADVISORY SERVICES STANDARD COSTS AND VARIANCE ANALYSIS

Actual quantity of direct materials purchased and used 30,000lbs.Actual cost of direct materials $84,000Unfavorable direct materials usage variance 3,000Standard quantity of direct materials allowed for May production 29,000 lbs.

For the month of May, Duke's direct materials price variance was: (M)A. $2,800 favorable C. $6,000 unfavorableB. $2,800 unfavorable D. $6,000 favorable Carter & Usry

23. Information on Energy’s direct material costs for October is as follows:Actual quantity of direct materials purchased and used 30,000 lbs.Actual cost of direct materials P92,000Unfavorable direct materials usage variance P 3,000Standard quantity of direct materials allowed for May production 29,000 lbs

For the month of October, Energy’s direct materials price variance was:a. P3,000 favorable c. P2,000 unfavorableb. P2,000 favorable d. P2,000 favorable Pol Bobadilla

38. The Porter Company has a standard cost system. In July the company purchased and used 22,500 pounds of direct material at an actual cost of $53,000; the materials quantity variance was $1,875 Unfavorable; and the standard quantity of materials allowed for July production was 21,750 pounds. The materials price variance for July was: (D)a. $2,725 F. c. $3,250 F.b. $2,725 U d. $3,250 U. G & N 9e

Materials Price & Quantity Variance - GivenActual CostQuestions 114 thru 116 are based on the following information. Horngren

21 . REQUIRED: The true statement about the actual results.DISCUSSION: (C) The sales volume variance is the change in contribution margin caused by the difference between the actual and budgeted unit volume. It equals the budgeted unit contribution margin times the difference between actual and expected volume, or $8,000 [($10 - $6) x (12,000 – 10,000)]. The sales volume variance is favorable because actual sales exceeded budgeted sales.Answer (A) is incorrect because the flexible budget variance for actual results is $11,200 favorable ($29,200 actual operating income - $18,000 flexible budget operating income). Answer (B) is incorrect because the sales price variance is $12,000 [$132,000 actual sales - $10 x 12,000 units sold)]. Answer (D) is incorrect because the fixed cost budget variance is $2,000 unfavorable ($32,000 actual - $30,000 budgeted).

Hector’s Camera Shop has prepared the following flexible budget for September and is in the process of interpreting the variances. F denotes a favorable variance and U denotes an unfavorable variance.

VariancesFlexible Budget Price Efficiency

Material A $20,000 $1,000U $1,200FMaterial B 30,000 500F 800UMaterial C 40,000 1,400U 1,000F

41. The actual amount spent for Material A was (E)a. $18,800. c. $19,800.b. $20,200. d. $21,000.

42. The actual amount spent for Material B was (E)a. $29,700. c. $30,500.b. $30,800. d. $30,300.

116.The explanation that lower-quality materials were purchased is MOST likely for (M)a. Material A. c. Material C.b. Material B. d. both Material A and C.

Standard Quantity Allowed41. Acme has a standard price of $6 per pound for materials. July’s results showed an

unfavorable materials price variance of $44 and a favorable quantity variance of $228. If 1,066 pounds were used in production, what was the standard quantity allowed for materials? (M)a. 1,066 c. 1,294b. 1,104 d. Some other number. D, L & H 9e

Actual Units Produced. JKL Company has a standard of 15 parts of component X costing P1.50 each. JKL purchased

14,910 units of component X for P22,145. JKL generated a P220 favorable price variance and a P3,735 favorable quantity variance. If there were no changes in the component inventory, how many units of finished product were produced? (M)A. 994 units. C. 1,000 unitsB. 1,090 units. D. 1,160 units RPCPA 1001

40. Acme has a standard of 15 parts of component X costing $1.50 each. Acme purchased 14,910 units of X for $21, 950. Acme generated a $415 favorable price variance and a $3,735 favorable quantity variance. If there were no changes in the component inventory, how many units of finished product were produced? (M)a. 994 units. c. 1,160 units.

CMA EXAMINATION QUESTIONS Page 14 of 142

MANAGEMENT ADVISORY SERVICES STANDARD COSTS AND VARIANCE ANALYSIS

b. 1,000 units. d. Some other number. D, L & H 9e

Materials Price & Quantity Variances44. If the current material standard calls for the use of 100,000 units at P1.00 each, but the actual

usage was 105,000 units at P0.90 each, the variances to be explained are (E)RPCPA 0596 a. b. c. d.Price Variance P10,000 fav P10,000 unfav P10,500 fav P10,500 unfavQuantity Variance P 4,500 unfav P4,500 fav P5,000 unfav P5,000 fav

43. A company producing a single product employs the following direct material cost standard for each unit of output:

3 pounds of material x $4/pound = $12/output unitData regarding the operations for the current month are as follows:

Planned production 26,000 unitsActual production 23,000 unitsActual purchases of direct materials (75,000 pounds) $297,000Direct materials used in production 70,000 pounds

What would be the amount of the direct materials purchase price variance and direct materials quantity variance that the company would recognize for the month?(E) CIA 1191 IV-16 Purchase Price Variance Quantity VarianceA. $3,120 favorable $32,000 favorableB. $3,000 favorable $24,000 unfavorableC. $3,000 favorable $4,000 unfavorableD. $2,800 favorable $4,000 unfavorable

37. Throop Company's standards call for one kilogram of materials for each unit of output at a cost of $2 per kilogram for the raw materials. Actual output was 50,000 units of product requiring 45,000 kilograms of raw materials at a cost of $2.10 per kilogram. There were no beginning or ending inventories of raw materials. The direct material price variance and quantity variance were:

G & N 10e Price QuantityA. $ 4,500 unfavorable $10,000 favorableB. $ 5,000 favorable $10,500 unfavorableC. $ 5,000 unfavorable $10,500 favorableD. $10,000 favorable $ 4,500 unfavorable

Questions 17 and 18 are based on the following information. CMA 0695 3-23 & 24

Blaster Inc., a manufacturer of portable radios, purchases the components from subcontractors to use to assemble into a complete radio. Each radio requires three units each of Part XBEZ52 which has a standard cost of $1.45 per unit. During May 1995, Blaster experienced the following with respect to Part XBEZ52.

UnitsPurchases ($18,000) 12,000Consumed in manufacturing 10,000Radios manufactured 3,000

44. During May 1995, Blaster Inc. incurred a purchase price variance of (E)a. $450 unfavorable. c. $500 favorable.b. $450 favorable. d. $600 unfavorable.

45. During May 1995, Blaster Inc. incurred a materials price efficiency variance of (E)a. $1,450 unfavorable. c. $4,350 unfavorable.b. $1,450 favorable. d. $4,350 favorable.

Questions 13 and 14 are based on the following information. CIA 1192 IV-20 & 21A manufacturer has the following direct materials standard for one of its products.

Direct materials: 3 pounds @ $1.60/pound = $4.80The company records all inventory at standard cost. Data for the current period regarding the manufacturer's budgeted and actual production for the product as well as direct materials purchases and issues to production for manufacture of the product are presented as follows.

Budgeted production for the period 8,000 unitsActual production for the period 7,500 unitsDirect materials purchases: Pounds purchased 25,000 pounds

CMA EXAMINATION QUESTIONS Page 15 of 142

MANAGEMENT ADVISORY SERVICES STANDARD COSTS AND VARIANCE ANALYSIS

Total cost $38,750Direct materials issued to production 23,000 pounds

46. The direct materials price variance for the current period is (E)A. $1,125 favorable. C. $1,200 favorable.B. $1,150 favorable. D. $1,250 favorable.

47. The materials efficiency variance for the current period is (E)A. $775 unfavorable. C. $1,600 favorable.B. $800 unfavorable. D. $3,200 favorable.

Questions 15 and 16 are based on the following information. CIA 1196 III-80 & 81A company manufactures a product that has the direct materials, standard cost presented below. Budgeted and actual information for the current month for the manufacture of the finished product and the purchase and use of the direct materials is also presented.Standard cost for direct materials – 1.60 lbs. @ $2.50 per lb. = $4.00

Budget ActualFinished goods (in units) 30,000 32,000Direct materials usage (in pounds) 48,000 51,000Direct materials purchases (in pounds) 48,000 50,000Total cost of direct materials purchases $120,000 $120,000

48. The direct materials price variance for the current month is (E)a. $7,500 unfavorable. c. $5,000 favorable.b. Zero. d. $5,100 favorable.

49. The direct materials efficiency variance for the current month is (E)a. $500 favorable. c. $7,500 unfavorable.b. $3,000 favorable. d. $8,000 unfavorable.

CMA EXAMINATION QUESTIONS Page 16 of 142

MANAGEMENT ADVISORY SERVICES STANDARD COSTS AND VARIANCE ANALYSIS

Questions 19 and 20 are based on the following information. RPCPA 1083Valenzuela Plastics Inc. has set a standard cost, P5.25 per unit for Material D and P12.25 per unit for Material E. In June, Valenzuela bought 17,500 units of Material D and 8,750 units of Material E. All Material D, except 1,400 units were bought at the standard unit cost. The 1,400 units had a unit cost of P6.15. Valenzuela bought 7,875 units of Material E at standard cost and 875 units at a unit cost of P14.In accordance with the standard two units of Material D and one unit of Material E should be used to make each unit of Product F. In January, 7,000 units of Product F were made and 15,050 units of Material D were used and 7,175 units of Material E were used.*. The total materials price variance is (M)

a. P2,791.25 favorable c. P13,781.25 favorableb. P2,791,25 unfavorable d. P13,781.25 unfavorable

*. The total materials quantity variance is (M)a. P7,656.25 favorable c. P13,781.25 favorableb. P7,656.25 unfavorable d. P13,781.25 unfavorable

Questions 74-75 are based on the following information: G & N 10eThe following materials standards have been established for a particular product:

Standard quantity per unit of output 8.7 gramsStandard price $15.30 per gram

The following data pertain to operations concerning the product for the last month:Actual materials purchased 3,900 gramsActual cost of materials purchased $56,550Actual materials used in production 3,600 gramsActual output 400 units

74. What is the materials price variance for the month? A. $2,880 F C. $2,880 UB. $3,120 U D. $3,120 F

75. What is the materials quantity variance for the month? A. $1,740 U C. $4,590 U B. $4,350 U D. $1,836 U

Questions 76-77 are based on the following information: G & N 10eThe following materials standards have been established for a particular product:

Standard quantity per unit of output 3.2 feetStandard price $10.10 per feet

The following data pertain to operations concerning the product for the last month:Actual materials purchased 5,900 feetActual cost of materials purchased $60,475Actual materials used in production 5,400 feetActual output 1,400 units

76. What is the materials price variance for the month? A. $885 U C. $885 FB. $810 U D. $810 F

77. What is the materials quantity variance for the month? A. $5,050 U C. $9,292 UB. $5,125 U D. $9,430 U

Questions 85 & 86 are based on the following information. G & N 9eThe following materials standards have been established for a particular product:

Standard quantity per unit of output .. 4.4 poundsStandard price ........................ $13.20 per pound

The following data pertain to operations concerning the product for the last month:Actual materials purchased ............ 4,800 poundsActual cost of materials purchased .... $62,880Actual materials used in production ... 4,300 poundsActual output ......................... 700 units

85. What is the materials price variance for the month? (E)a. $480 F c. $430 Ub. $430 F d. $480 U

86. What is the materials quantity variance for the month? (E)a. $6,550 U c. $16,104 Ub. $15,982 U d. $6,600 U

Questions 87 thru 88 are based on the following information. G & N 9eThe following materials standards have been established for a particular product:

Standard quantity per unit of output .. 1.9 gramsStandard price ........................ $18.00 per gram

The following data pertain to operations concerning the product for the last month:Actual materials purchased ............ 5,800 gramsActual cost of materials purchased .... $108,460Actual materials used in production ... 5,200 gramsActual output ......................... 2,700 units

CMA EXAMINATION QUESTIONS Page 17 of 142

MANAGEMENT ADVISORY SERVICES STANDARD COSTS AND VARIANCE ANALYSIS

87. What is the materials price variance for the month? (E)a. $4,060 U c. $3,640 Ub. $3,640 F d. $4,060 F

88. What is the materials quantity variance for the month? (E)a. $1,260 U c. $11,220 Ub. $1,309 U d. $10,800 U

Questions 89 & 90 are based on the following information. G & N 9eThe following materials standards have been established for a particular product:

Standard quantity per unit of output .. 6.8 metersStandard price ........................ $17.10 per meter

The following data pertain to operations concerning the product for the last month:Actual materials purchased ............ 9,000 metersActual cost of materials purchased .... $156,600Actual materials used in production ... 8,500 metersActual output ......................... 1,200 units

89. What is the materials price variance for the month? (E)a. $2,700 F c. $2,550 Fb. $2,550 U d. $2,700 U

90. What is the materials quantity variance for the month? (E)a. $5,814 U c. $5,916 Ub. $8,700 U d. $8,550 U

Questions 1 & 2 are based on the following information. H & MAssume that the standard cost to make one widget includes 5 units of raw materials at a price of $3 per unit. In July, 17,000 units of raw materials were purchased for $50,400, and 10,400 units of raw materials were used to produce 2,000 units of finished product. The material price variance is recorded at the time of purchase.

50. What is the materials price variance for July? (E)a. $2,400 (U) d. $400 (U)b. $600 (F) e. $600 (U)c. $1,200 (U)

51. What is the materials usage variance? (E)a. $1,200 (U) d. $600 (U)b. $400 (U) e. $0

c. $600 (F)

Questions 1 and 2 are based on the following information. Flamholtz & DiamondGazarra Company has supplied you with the following data relating to the material required to manufacture mini-ring-nickers:

Standard price per ounce of material $1.25Standard quantity of material per mini-ring-nicker 4 oz.Actual material purchased 2,800 oz.Actual material used in production 2,200 oz.Actual mini-ring-nickers produced 520 unitsActual cost of material purchased $3,920

. If Gazarra had planned to produce a total of 550 mini-ring-nickers during the year, its material price variance was (E)A. $330 unfavorable. D. $78 unfavorable.B. $420 unfavorable. E. $0.15 unfavorable.C. $770 unfavorable.

. If Gazarra had planned to produce a total of 360 moni-ring-nickers during the year, its material quantity variance was (E)A. $200 unfavorable. D. $1,008 unfavorable.B. $900 unfavorable. E. $168 unfavorable.C. $150 unfavorable.

Questions 82 through 84 are based on the following information. BarfieldsKauai Mfg. Co. produces beach chairs. Chair frames are all the same size, but can be made from plastic, wood, or aluminum. Regardless of frame choice, the same sailcloth is used for the seat on all chairs. Kauai has set a standard for sailcloth of $9.90 per square yard and each chair requires 1 square yard of material. Kauai produced 500 plastic chairs, 100 wooden chairs, and 250 aluminum chairs during June. The total cost for 1,000 square yards of sailcloth during the month was $10,000. At the end of the month, 50 square yards of sailcloth remained in inventory.

82. The unfavorable material price variance for sailcloth purchases for the month was (E)a. $100. c. $1,090.b. $495. d. $1,585.

83. Assuming that there was no sailcloth in inventory at the beginning of June, the unfavorable material quantity variance for the month was (E)a. $495. c. $990.b. $500. d. $1,000.

CMA EXAMINATION QUESTIONS Page 18 of 142

MANAGEMENT ADVISORY SERVICES STANDARD COSTS AND VARIANCE ANALYSIS

84. Kauai Mfg. Co. could set a standard cost for which of the following? (E)a. b. c. d.

Frame cost Yes No Yes NoPredetermined OH rate Yes No No YesLabor rate Yes No No Yes

Standard Quantity & Materials Usage VarianceQuestions 1 & 2 are based on the following information. H & MSexson Corporation uses a standard cost system. The following information pertains to direct materials for the month of May.

Standard price per lb. $12.00Actual purchase price per lb. $11.00Quantity purchased 6,200 lbs.Quantity used 5,900 lbs.Standard quantity allowed for actual output 6,000 lbs.Actual output 1,000 units

Sexson reports its material price variances at the time of purchase.

52. What is the standard quantity of direct materials per unit for Sexson Corporation? (E)a. 7.00 lbs. d. 5.90 lbs.b. 6.50 lbs. e. 6.00 lbs.c. 6.20 lbs. H & M

53. What is the material usage variance for Sexson Corporation? (E)a. $1,200 (F) d. $1,200 (U)b. $2,600 (F) e. $6,200 (F)c. $3,800 (F)

Questions 1 thru 3 are based on the following information.The cost accountant for Hando Company has just informated you that the company’s material quantity variance was exactly equal to the company’s material price variance for the year. The company had expected to produce 450 units of its product. However, because of a slump in the economy, it was only able to justify the production of 400 units. Other cost information relating to the company’s raw materials activity is shown below:

Standard price per ton of material $ 50Actual tons purchased during the period 10Actual tons used in production during the period 8

Actual cost of material purchased during the period $600Number of pounds in 1 ton 2,000

. The standard quantity of materials allowed for the number of units produced by Hando totaledA. 6.4 tons. C. 8.4 tons.B. 8.0 tons. D. 6.0 tons

. Hando’s material quantity variance wasA. $80 unfavorable. C. $100 unfavorable.B. $80 favorable. D. $100 favorable.

. On the average, Hando used exceeded its standard materials allowed byA. 20 pounds per unit produced. D. 8 pounds per unit produced.B. 18 pounds per unit produced. E. 9 pounds per unit produced.C. 10 pounds per unit produced.

Materials Mix & Yield VarianceMaterials Quantity Variance*. The material mix variance for a product is P500 unfavorable, and the material yield variance is

P600 favorable. This means that the material (M)a. Price variance is P100 favorable.b. Quantity variance is P100 favorable.c. Price variance is unfavorable but the amount cannot be determined from the information

given.d. Quantity variance is P1,100. RPCPA 1092

Materials Yield Variance36. Slim Candle Co. manufactures candles in various shapes, sizes, colors, and scents.

Depending on the orders received, not all candles require the same amount of color, dye, or scent materials. Yields also vary, depending upon the usage of beeswax or synthetic wax. Standard ingredients for 1,000 pounds of candles are

Standard Mix Standard Cost per PoundInput: Beeswax 200 lbs. 1.00 Synthetic wax 840 lbs. 0.20 Colors 7 lbs. 2.00 Scents 3 lbs. 6.00

CMA EXAMINATION QUESTIONS Page 19 of 142

MANAGEMENT ADVISORY SERVICES STANDARD COSTS AND VARIANCE ANALYSIS

Totals 1,050 lbs. 9.20Standard output 1,000 lbs.

Price variances are charged off at the time of purchase. During January, the company was busy manufacturing red candles for Valentine’s Day. Actual production then was:

Input In pounds Beeswax 4,100 Synthetic wax 13,800 Colors 2,200 Scents 60 Total 20,160Actual output 18,500

The material yield variance isA. P280 unfavorable. C. P3,989 unfavorable.B. P280 favorable. D. P3,989 favorable. Pol Bobadilla

Materials Mix & Yield VarianceQuestions 22 and 23 are based on the following information. GleimThe information was presented as part of Question 5 on Part 4 of the June 1992 CMA examination.A company produces a gasoline additive. The standard costs and input for a 500-liter batch of the additive are presented below.

ChemicalStandard Input-

Quantity in LitersStandard Cost

per LiterTotalCost

Echol 200 $0.200 $ 40.00Protex 100 0.425 42.50Benz 250 0.150 37.50CT-40 50 0.300 15.00

600 $135.00The quantities purchased and used during the current period are shown below. A total of 140 batches were made during the current period.

ChemicalQuantity Pur-

chased (Liters)Total

Purchase PriceQuantity Used

(Liters)Echol 25,000 $ 5,365 26,600Protex 13,000 6,240 12,880Benz 40,000 5,840 37,800CT-40 7,500 2,220 7,140

85,500 $ 19,665 84,420

54. What is the materials mix variance for the operation? (M)a. $294 favorable. c. $94.50 unfavorable.b. $388.50 favorable. d. $219.50 favorable.

55. What is the materials yield variance for this operation? (M)a. $294.50 favorable. c. $94.50 unfavorable.b. $388.50 favorable. d. $219.50 favorable.

Questions xx thru xx are based on the following information. H & MHarrigan Corporation uses two materials in the production of their product. The materials, A and B, have the following standards:

Material Standard Mix Standard Unit Price Standard CostA 3,500 units $1.00 per unit $3,500B 1,500 units 3.00 per unit $4,500

Yield 4,000 unitsDuring January, the following actual production information was provided:

Material Actual MixA 30,000 unitsB 20,000 units

Yield 36,000 units

56. What is the materials mix variance? (M)a. $5,000 (F) c. $10,000 (F)b. $10,000 (U) d. $15,000 (F)

57. What is the materials yield variance? (M)a. $4,000 (F) c. $8,000 (U)b. $8,000 (F) d. $9,000 (U)

58. What is the materials usage variance? (M)a. $10,000 (U) c. $8,000 (F)b. $8,000 (U) d. $18,000 (U)

Materials Flexible Budget VarianceQuestions 24 and 25 are based on the following information. CMA 0697 3-22 & 23The controller of Durham Skates is reviewing the production cost report for July. An analysis of direct materials costs reflects an unfavorable flexible budget variance of $25. The plant manager believes this is excellent performance on a flexible budget for 5,000 units of direct materials. However, the production supervisor is not pleased with this result because he claims to have saved $1,200 in material cost on actual production using 4,900 units of direct materials. The standard materials cost is $12 per unit. Actual materials used for the month amounted to $60,025.

59. The actual average cost per unit for materials was (M)a. $12.00 c. $12.24

CMA EXAMINATION QUESTIONS Page 20 of 142

MANAGEMENT ADVISORY SERVICES STANDARD COSTS AND VARIANCE ANALYSIS

b. $12.01 d. $12.25

60. If the direct materials variance is investigated further, it will reflect a price variance of (M)a. Zero. c. $1,225 unfavorable.b. $1,200 favorable. d. $2,500 favorable.

ComprehensiveTHE FOLLOWING INFORMATION APPLIES TO QUESTIONS 66 THROUGH 68. HorngrenJJ White planned to use $82 of material per unit but actually used $80 of material per unit, and planned to make 1,200 units but actually made 1,000 units.

61. The flexible-budget amount is (E)a. $80,000. c. $96,000.b. $82,000. d. $98,400.

62. The flexible-budget variance is (E)a. $2,000 favorable. c. $16,400 unfavorable.b. $14,000 unfavorable. d. $2,400 favorable.

63. The sales-volume variance is (E)a. $2,000 favorable. c. $16,400 unfavorable.b. $14,000 unfavorable. d. $2,400 favorable.

THE FOLLOWING INFORMATION APPLIES TO QUESTIONS 70 THROUGH 72. HorngrenMcKenna Incorporated planned to use $24 of material per unit but actually used $25 of material per unit, and planned to make 1,000 units but actually made 1,200 units.

64. The flexible-budget amount is (E)a. $24,000. c. $28,800.b. $25,000. d. $30,000.

65. The flexible-budget variance is (E)a. $4,800 favorable. c. $5,000 unfavorable.b. $1,200 unfavorable. d. $6,000 favorable.

66. The sales-volume variance is (E)a. $4,800 favorable. c. $5,000 unfavorable.b. $1,200 unfavorable. d. $6,000 favorable.

Questions 26 through 28 are based on the following information. Gleim

A manufacturer of radios purchases components from subcontractors for assembly into complete radios. Each radio requires three units each of Part X, which has a standard cost of $2.90 per unit. During June, the company had the following experience with respect to Part X:

UnitsPurchases ($36,000) 12,000Consumed in manufacturing 10,000Radios manufactured 3,000

67. During June, the company incurred a materials purchase-price variance of (E)a. $900 unfavorable. c. $1,200 unfavorable.b. $900 favorable. d. $1,200 favorable.

68. During June, the company incurred a materials efficiency variance of (E)a. $2,900 unfavorable. c. $8,700 unfavorable.b. $2,900 favorable. d. $8,700 favorable.

69. The amount that will be shown on a flexible budget for Part X usage during the month of June is (E)a. $26,100 c. $29,000b. $27,000 d. $36,000

LABOR VARIANCESStandard DL Time per Unit84. Hansen Company is a chemical manufacturer that supplies various products to industrial

users. The company plans to introduce a new chemical solution called Bysap, for which it needs to develop a standard product cost. The following labor information is available on the production of Bysap. The product, which is bottled in 10-liter containers, is primarily a mixture of Byclyn,

Salex, and Protex. The finished product is highly unstable, and one 10-liter batch out of six if rejected at

final inspection. Rejected batches have no commercial value and are thrown out. It takes a worker 35 minutes to process one 10-liter batch of Bysap. Employees work

eight hours a day, including one hour per day for rest breaks and cleanups.What is the standard labor time to produce one 10-liter batch of Bysap?A. 35 minutes. C. 48 minutes.B. 40 minutes. D. 45 minutes. Pol Bobadilla

Standard DL Cost per Unit70. The following direct labor information pertains to the manufacture of product Glu:

Time required to make one unit 2 direct labor hoursNumber of direct workers 50

CMA EXAMINATION QUESTIONS Page 21 of 142

MANAGEMENT ADVISORY SERVICES STANDARD COSTS AND VARIANCE ANALYSIS

Number of productive hours per week, per worker 40Weekly wages per worker $500Workers’ benefits treated as direct labor costs 20% of wages

What is the standard direct labor cost per unit of product Glu? (M)a. $30. c. $15.b. $24. d. $12. AICPA 0592 II-46

62. The following direct labor information pertains to the manufacturer of Part SARS:Number of hours required to make a part 2.5 DLHNumber of direct workers 75Number of total productive hours per week 3,000Weekly wages per worker P1,000Laborer’s fringe benefits treated as direct labor costs 25% of wages

What is the standard direct labor cost per unit of Parts SARS? (M)A. P62.500 C. P41.670B. P78.125 D. P84.125 Pol Bobadilla

71. Media Co. manufactures televisions. The following direct labor information relates to the manufacture of televisions.

Number of workers 60Number of productive hours per week, per worker 40Hours required to make 1 unit 3Weekly wages per worker $600Employee benefits treated as direct labor costs 20% of wages

What is the standard direct labor cost per unit? (M)A. $54 C. $30B. $36 D. $18 Gleim

*. Each unit of Product 8 in 1 requires two direct labor hours. Employee benefit costs are treated as direct labor costs. Data on direct labor are as follows:

Number of direct employees 25Weekly productive hours per employee 30Estimated weekly wages per employee $240Employee benefits (related to weekly wages) 25%

The standard direct labor cost per unit of Product 8in1 is: (M)A. $8.00 C. $12.00B. $10.00 D. $20.00 CMA adapted

72. Each unit of Product XK-46 requires three direct labor hours. Employee benefit costs are treated as direct labor costs. Data on direct labor are

Number of direct employees 25Weekly productive hours per employee 35Estimated weekly wages per employee $245Employee benefits (related to weekly wages) 25%

The standard direct labor cost per unit of Product XK-46 is (M)A. $21.00. C. $29.40.B. $26.25. D. $36.75. CMA 0684 4-26

Standard DLH Allowed28. Hart Company's labor standards call for 500 direct labor hours to produce 250 units of product.

During October the company worked 625 direct labor hours and produced 300 units. The standard hours allowed for October would be: (E)a. 625 hours. c. 600 hours.b. 500 hours. d. 250 hours. G & N 9e

Total DL Variance73. The following is a standard cost variance analysis report on direct labor cost for a division of a

manufacturing company.Job Actual Hours at

Actual WagesActual Hours at

Standard WagesStandard Hours atStandard Wages

213 $3,243 $3,700 $3,100215 15,345 15,675 15,000217 6,754 7,000 6,600219 19,788 18,755 19,250221 3,370 3,470 2,650Totals $48,500 $48,600 $46,600

What is the total flexible budget direct labor variance for the division? (M)a. $1,00 unfavorable. c. $1,900 favorable.b. $1,900 unfavorable. d. $2,000 unfavorable. CIA 0592 IV-18

DL Rate Variance*. Below are FLX Corporation’s standard costs to produce one concrete table:

Direct raw materials 2 kgs. P375 per kg.Direct labor 30 minutes P31.25 per hour

In September, FLX produced 250 concrete tables. 520 kgs. of raw materials were used at a total cost of P193,440. A total of 128 direct labor hours were used at a cost of P4,096. The direct labor rate variance is (E)a. P22.50 c. P64.75b. P93.00 d. P96.00 RPCPA 1097

CMA EXAMINATION QUESTIONS Page 22 of 142

MANAGEMENT ADVISORY SERVICES STANDARD COSTS AND VARIANCE ANALYSIS

. JKL Company is using a direct labor cost standard of 4 hours and a P12 wage rate per hour for one of its products. Planned production was 300 units, but actual production was 250 units, using for each unit 3 labor hours at a P13 wage rate. What is the labor price variance? (M)A. P750 unfavorable. C. P1,200 favorable.B. P900 unfavorable. D. P750 favorable. RPCPA 1001

42. Genco paid $39,400 to direct labor for the production of 1,500 units. Standards allow 2 labor hours per unit at a rate of $12.50 per hour. Actual hours totaled 2,900. The direct labor rate variance was (E)a. $1,250 favorable. c. $3,150 unfavorable.b. $3,150 favorable. d. $1,900 unfavorable. D, L & H 9e

42. Genco paid $78,800 to direct labor for the production of 1,500 units. Standards allow 2 labor hours per unit at a rate of $25.00 per hour. Actual hours totaled 2,900. The direct labor rate variance was (E)a. $2,050 favorable c. $6,300 unfavorableb. $3,800 favorable d. some other number L & H 10e

46. Chippewa paid $32,225 to direct labor for the production of 1,700 units. Standards allow 3 labor hours per unit at a rate of $6.50 per hour. Actual hours totaled 5,150. The direct labor rate variance was (E)a. $1,250 favorable. c. $325 favorable.b. $925 favorable. d. $325 unfavorable. D, L & H 9e

. Blasto Company produces bug-bombs. Direct labor standards for the firm and actual data for the month of April are shown below:

Standard labor rate per hour $6Standard hours allowed per bug-bomb 0.05Actual bug-bombs produced in April 50,000Actual labor costs for April $17,400Actual labor hours recorded for April 3,000

Blasto’s labor rate variance for April was (E)A. $640 unfavorable. C. $560 unfavorable.B. $600 favorable. D. $500 favorable. Flamholtz & Diamond

48. The following labor standards have been established for a particular product:Standard labor hours per unit of output .. 1.7 hoursStandard labor rate ...................... $14.05 per hour

The following data pertain to operations concerning the product for the last month:Actual hours worked ...................... 3,700 hours

Actual total labor cost .................. $50,690Actual output ............................ 2,300 units

What is the labor rate variance for the month? (E)a. $1,295 F c. $4,246 Fb. $2,877 F d. $4,246 U G & N 9e

47. The following labor standards have been established for a particular product: Standard labor hours per unit of output 9.0 hoursStandard labor rate $15.10 per hour

The following data pertain to operations concerning the product for the last month:Actual hours worked 8,100 hoursActual total labor cost $119,880Actual output 800 units

What is the labor rate variance for the month? A. $11,160 F C. $11,160 UB. $13,320 U D. $2,430 F G & N 10e

33. Operational statistics generated for the period just ended for APEX Manufacturing Co., maker of a line of furniture, follow:

Standards per set: Materials 2.0 yards @ P100 Direct labor 0.5 hour @ P200Actual results: Production 20,000 sets Materials used 37,000 yards Price per yard P102

CMA EXAMINATION QUESTIONS Page 23 of 142

MANAGEMENT ADVISORY SERVICES STANDARD COSTS AND VARIANCE ANALYSIS

Direct labor hours used 9,000 hours Direct labor cost P1,764,000

The direct labor rate variance was (E)a. P36,000 favorable c. P40,000 favorableb. P36,000 unfavorable d. P40,000 unfavorable RPCPA 1094

74. The flexible budget for the month of May 1993 was for 9,000 units at a direct materials cost of $15 per unit. Direct labor was budgeted at 45 minutes per unit for a total of $81,000. Actual output for the month was 8,500 units with $127,500 indirect materials and $77,775 in direct labor expense. The direct labor standard of 45 minutes was maintained throughout the month. Variance analysis of the performance for the month of May would show a(n) (E)a. Favorable materials usage variance of $7,500.b. Favorable direct labor efficiency variance of $1,275.c. Unfavorable direct labor efficiency variance of $1,275.d. Unfavorable direct labor price variance of $1,275. CMA 0693 3-15

Actual Direct Labor Hours. STA Company uses a standard cost system. The following information pertains to direct labor

costs for the month of June:Standard direct labor rate per hour P10.00Actual direct labor rate per hour P 9.00Labor rate variance P12,000 FActual output 2,000 unitsStandard hours allowed for actual production 10,000 hours

How many actual labor hours were worked during March for STA Company? (M)A. 10,000 C. 8,000B. 12,000 D. 10,500 Pol Bobadilla

Actual Direct Labor Rate52. The Reedy Company uses a standard costing system. The following data are available for

November:Actual direct labor hours worked ... 5,800 hoursStandard direct labor rate ......... $9 per hourLabor rate variance ................ $1,160 favorable

The actual direct labor rate for November is: (M)a. $8.80. c. $9.00.b. $8.90. d. $9.20. G & N 9e

51. The Hanson Company employs a standard costing system. The following data are available for February:

Actual direct labor hours worked 6,500Standard direct labor rate $8 per hourLabor rate variance $2,600 favorable

The actual direct labor rate for February is: (M)A. $7.60. C. $8.00.B. $8.40. D. $2.50. G & N 10e

75. During October, 14,000 direct labor hours were worked at a standard cost of $40 per hour. If the direct labor rate variance for October was $70,000 favorable, the actual cost per direct labor hour must be (M)a. $35 c. $45b. $40 d. $50 H & M

76. During October, 20,000 direct labor hours were worked at a standard cost of $5 per hour. If the direct labor rate variance for October was $4,000 unfavorable, the actual cost per direct labor hour must be (M)a. $5.20 c. $4.80b. $5.00 d. $4.60 H & M

Actual Direct Labor Hours & Actual Direct Labor RateQuestions 87-88 are based on the following information: G & N 10eThe auto repair shop of Empire Motor Sales uses standards to control labor time and labor cost in the shop. The standard time for a motor tune-up is 2.5 hours. The record showing time spent in the shop last week on tune-ups has been misplaced; however, the shop supervisor recalls that 50 tune-ups were completed during the week and the controller recalls that the labor rate variance on tune-ups was $87, favorable. The shop has a set standard labor rate of $9 per hour for tune-up work. The total labor variance for the week on tune-up work was $93, unfavorable.

87. The number of actual hours spent on tune-up work last week was: A. 125 hours. B. 105 hours. C. 145 hours. D. Cannot be computed without further information.

CMA EXAMINATION QUESTIONS Page 24 of 142

MANAGEMENT ADVISORY SERVICES STANDARD COSTS AND VARIANCE ANALYSIS

88. The actual hourly rate of pay for tune-up work last week was: A. $8.40 per hour. B. $9.00 per hour. C. $9.60 per hour. D. Cannot be computed without further information.

Actual Direct Labor Cost77. Sullivan Corporation’s direct labor costs for the month of March were as follows:

Standard direct labor hours 42,000Actual direct labor hours 40,000Direct labor rate variance – favorable $8,400Standard direct labor rate per hour $6.30

What was Sullivan’s total direct labor payroll for the month of March? (M)a. $243,600 c. $264,600b. $252,000 d. $260,400 AICPA 1180 I-35

78. Daniel Corporation's direct labor costs for June were as follows:Actual direct labor hours 32,000Standard direct labor hours 33,600Direct labor rate variance – favorable $6,720Standard direct labor rate per hour $5.04

Compute Daniel's total direct labor payroll for the month of June. (M)A. $154,560 C. $167,680B. $154,880 D. $168,000 Gleim

*. The following information related to the direct labor costs of Valley Mfg. Co., for the month of April 19x9:

Actual direct labor hours 57,000 hoursStandard direct labor hours 58,000 hoursDirect labor rate variance – favourable P8,550Standard direct labor rate per hour P5.50

What was the total direct labor payroll of Valley Mfg. for the month of April 19x9? (M)a. P310,300 c. P319,000b. P322,050 d. P304,950 RPCPA 1089

Standard Direct Labor Rate 79. Information on Hanley’s direct labor costs for the month of January is as follows:

Actual direct labor rate $7.50Standard direct labor hours allowed 11,000Actual direct labor hours 10,000Direct labor rate variance – favorable $5,500

The standard direct labor rate in January was (E)a. $6.95 c. $8.00b. $7.00 d. $8.05 CPA 0582 I-25

80. Lake's direct labor costs for the month of May are as follows:Standard direct labor hours allowed 12,500Actual direct labor rate $8.25Actual direct labor hours 10,000Direct labor rate variance – favorable $5,600

What was Lake's standard direct labor rate in May? (E)A. $7.69 C. $8.25B. $7.80 D. $8.81 Gleim

81. Information on ABC Company’s direct labor costs for the month of August is as follows:Actual direct labor rate P7.50Standard direct labor hours allowed 11,000Actual direct labor hours 10,000Direct labor price variance – unfavorable P5,000

The standard direct labor rate in January was (E)a. P8,05 c. P7.00b. P6.95 d. P8.00 RPCPA 1001

CMA EXAMINATION QUESTIONS Page 25 of 142

MANAGEMENT ADVISORY SERVICES STANDARD COSTS AND VARIANCE ANALYSIS

49. Information on Hanley's direct labor costs for the month of January follows:Actual direct labor rate $7.50Standard direct labor hours allowed 11,000Actual direct labor hours 10,000Direct labor rate variance—favorable $5,500

What was the standard direct labor rate in effect for the month of January? A. $6.95 C. $8.00 B. $7.00 D. $8.05 G & N 10e

DL Efficiency Variance82. Yola Co. manufactures one product with a standard labor cost of 4 hours at $12.00 per hour.

During June 1,000 units were produced using 4,100 hours at $12.20 per hour. The unfavorable direct labor efficiency variance was (E)a. $1,220 c. $820b. $1,200 d. $400 AICPA 1192 II-21

46. Yola Company manufactures a product with standards for direct labor of 4 direct labor-hours per unit at a cost of $12.00 per direct labor-hour. During June, 1,000 units were produced using 4,100 hours at $12.20 per hour. The direct labor efficiency variance was: (E)a. $1,200 favorable. c. $2,020 favorable.b. $1,200 unfavorable. d. $2,020 unfavorable. AICPA adapted

45. Palo Corp. manufactures one product with a standard direct labor cost of 2 hours at $6.00 per hour. During March, 500 units were produced using 1,050 hours at $6.10 per hour. The unfavorable direct labor efficiency variance is: A. $100. C. $300.B. $105. D. $305. G & N 10e

83. Bell Co. manufactures a single product with a standard direct labor cost of 2 hours at $10.00 per hour. During November, 1,500 units were produced requiring 3,200 hours at $10.25 per hour. What was the unfavorable direct labor efficiency variance? (E)A. $2,050 C. $1,250B. $2,000 D. $1,200 Gleim

84. The direct labor standards for producing a unit of a product are two hours at $10 per hour. Budgeted production was 1,000 units. Actual production was 900 units, and direct labor cost was $19,000 for 2,000 direct labor hours. The direct labor efficiency variance was: (E)A. $1,000 favorable C. $2,000 favorableB. $1,000 unfavorable D. $2,000 unfavorable CIA adapted

43. DIGITAL Products produces a product, Digit, and uses standard costing methods. The standard direct labor cost of Digit is one and one-half hours at P180 per hour. During October, 19x7, 500 Digit units were produced in 1,000 hours at P176 per hour. The direct labor efficiency variance is a favorable (an unfavorable) (E)a. P30,000 c. P(30,000)b. P45,000 d. P(45,000) RPCPA 1097

*. Alice & Co. has this selected information: standard direct labor hours of 2,000 at P7.00/hour and actual labor hours worked are 2,400 at an actual rate of P7.60/hour. The labor efficiency variance is (E)a. P1,400 c. P2,800b. P2,400 d. none of these RPCPA 1087

CMA EXAMINATION QUESTIONS Page 26 of 142

MANAGEMENT ADVISORY SERVICES STANDARD COSTS AND VARIANCE ANALYSIS

43. Genco paid $39,400 to direct labor for the production of 1,500 units. Standards allow 2 labor hours per unit at a rate of $12.50 per hour. Actual hours totaled 2,900. The direct labor efficiency variance was (E)a. $1,250 favorable. c. $3,150 unfavorable.b. $3,150 favorable. d. $1,900 unfavorable. D, L & H 9e

43. Genco paid $78,800 to direct labor for the production of 1,500 units. Standards allow 2 labor hours per unit at a rate of $25.00 per hour. Actual hours totaled 2,900. The direct labor efficiency variance was (E)a. $2,500 favorable c. $6,300 unfavorableb. $3,800 favorable d. some other number L & H 10e

47. Chippewa paid $32,225 to direct labor for the production of 1,700 units. Standards allow 3 labor hours per unit at a rate of $6.50 per hour. Actual hours totaled 5,150. The direct labor efficiency variance was (E)a. $1,250 favorable. c. $325 favorable.b. $925 favorable. d. $325 unfavorable. D, L & H 9e

85. The total budgeted direct labor cost of a company for the month was set at $75,000 when 5,000 units were planned to be produced. The following standard cost, stated in terms of direct labor hours (DLH), was used to develop the budget for direct labor cost:

1.25 DLH x $12.00/DLH = $15.00/unit producedThe actual operating results for the month were as follows:

Actual units produced 5,200Actual direct labor hours worked 6,600Actual direct labor cost $77,220

The direct labor efficiency variance for the month would be (E)A. $4,200 unfavorable. C. $2,220 unfavorable.B. $3,000 unfavorable. D. $1,200 unfavorable. CIA 1191 IV-15

50. Cameron Company has standard variable costs as follows:Materials, 3 pounds at $4.00 per pound $12.00

Labor, 2 hours at $10.00 per hour 20.00Variable overhead, $7.50 per labor hour 15.00

$47.00 During September, Chetek produced 5,000 units, using 9,640 labor hours at a total wage of $94,670 and incurring $78,600 in variable overhead. The direct labor efficiency variance is (E)a. $1,730 favorable. c. $3,600 favorable.b. $2,700 favorable. d. $5.330 favorable. D, L & H 9e

. Blasto Company produces bug-bombs. Direct labor standards for the firm and actual data for the month of April are shown below:

Standard labor rate per hour $6Standard hours allowed per bug-bomb 0.05Actual bug-bombs produced in April 50,000Actual labor costs for April $17,400Actual labor hours recorded for April 3,000

Blasto’s labor efficiency variance for April was (M)A. $2,900 unfavorable. C. $3,000 favorable.B. $2,820 favorable. D. $3,000 unfavorable. Flamholtz & Diamond

46. The following labor standards have been established for a particular product: Standard labor hours per unit of output 8.7 hoursStandard labor rate $18.10 per hour

The following data pertain to operations concerning the product for the last month:Actual hours worked 3,800 hoursActual total labor cost $67,640Actual output 500 units

What is the labor efficiency variance for the month? A. $9,790 F C. $9,955 FB. $11,095 U D. $11,095 F G & N 10e

86. A manager prepared the following table by which to analyze labor costs for the month:Actual Hours at

Actual RateActual Hours at Standard Rate

Standard Hours atStandard Rate

$10,000 $9,800 $8,820What variance was $980? (M)A. Labor efficiency variance. C. Volume variance.B. Labor rate variance. D. Labor spending variance. CIA 0590 IV-15

16. A product requires 0.60 standard labor hours, the actual labor rate is $10 per hour, and production was 300 units. Actual labor cost was $1,862 at $9.80. Which of the following is true? (M)

CMA EXAMINATION QUESTIONS Page 27 of 142

MANAGEMENT ADVISORY SERVICES STANDARD COSTS AND VARIANCE ANALYSIS

a. The labor rate variance was $98 favorable.b. The labor rate variance was $62 unfavorable.c. The labor efficiency variance was $62 unfavorable.d. The labor efficiency variance was $100 unfavorable. D, L & H 9e

47. The following labor standards have been established for a particular product:Standard labor hours per unit of output .. 8.3 hoursStandard labor rate ...................... $12.10 per hour

The following data pertain to operations concerning the product for the last month:Actual hours worked ...................... 6,100 hoursActual total labor cost .................. $71,370Actual output ............................ 900 units

What is the labor efficiency variance for the month? (E)a. $19,017 F c. $16,029 Fb. $19,017 U d. $16,577 F G & N 9e

Actual Hours Worked50. The standards for direct labor for a product are 2.5 hours at $8 per hour. Last month, 9,000

units of the product were made and the labor efficiency variance was $8,000 F. The actual number of hours worked during the past period was: (M)a. 23,500. c. 20,500.b. 22,500. d. 21,500. G & N 9e

66. Rainbow Company uses a standard cost system. Information about its direct labor costs for Product Lux for the month of January follows:

Standard hours allowed for actual production 1,500Actual hourly rate paid P61.00Standard hourly rate P60.00Labor efficiency variance – favorable P6,000

How many direct labor hours were actually worked during the month of January?A. 1,400 C. 1,498B. 1,402 D. 1,600 Pol Bobadilla

48. BINGO Co. uses a standard cost system. Direct labor statistics for the month of May, 19x7 follows:

Actual rate per hour P152.50Standard rate per hour P150.00Labor efficiency variance – unfavorable P15,000Standard hours allowed for actual production 37,500

What was the actual number of hours worked? (M)a. 35,700 c. 37,000

b. 36,700 d. 37,600 RPCPA 0597

50. Harper Company uses a standard cost system. Data relating to direct labor for the month of August follows:

Direct labor efficiency variance—favorable $5,250Standard direct labor rate $7.00Actual direct labor rate $7.50Standard hours allowed for actual production 9,000

What are the actual hours worked for the month of August? A. 9,750 C. 8,300B. 8,400 D. 8,250 G & N 10e

20. Information on Ulan Company’s direct labor costs is as follows:Standard direct labor rate P7.50Actual direct labor rate P7.00Standard direct labor hours 20,000Direct labor usage variance – unfavorable P8,400

What were the actual hours worked, rounded to the nearest hour?a. 21,914 c. 21,120B. 20,714 d. 21,200 Pol Bobadilla

12. Information on Rita Company’s direct labor costs is as follows:Standard direct labor rate P 3.75Actual direct labor rate P 3.50Standard direct labor hours 10,000Direct labor usage variance – unfavorable P 4,200

What were the actual hours worked, rounded to the nearest hour?a. 11,914 c. 11,120b. 10,714 d. 11,200 Pol Bobadilla

87. Tub Co. uses a standard cost system. The following information pertains to direct labor for product B for the month of October:

Standard hours allowed for actual production 2,000Actual rate paid per hour $8.40Standard rate per hour $8.00Labor efficiency variance $1,600 U

What were the actual hours worked? (M)a. 1,800 c. 2,190b. 1,810 d. 2,200 AICPA 1186 II-21

88. Bolt Co. uses a standard-cost system. Bolt's direct labor information for July is as follows:

CMA EXAMINATION QUESTIONS Page 28 of 142

MANAGEMENT ADVISORY SERVICES STANDARD COSTS AND VARIANCE ANALYSIS

Standard hours allowed for actual production 3,000Actual rate paid per hour $9.35Standard rate per hour $8.50Labor efficiency variance $1,870 U

The actual hours worked equaled (M)A. 2,780 C. 3,200B. 2,800 D. 3,220 Gleim

89. Information on Orman Company's direct labor costs is as follows:Standard direct labor rate $3.75Actual direct labor rate $3.50Standard direct labor hours 10,000Direct labor usage (efficiency) variance—unfavorable $ 4,200

What were the actual hours worked, rounded to the nearest hour?A. 11,914 C. 11,120B. 10,714 D. 11,200 Carter & Usry

49. Lab Corp. uses a standard cost system. Direct labor information for Product CER for the month of October follows:

Standard direct labor rate ................. $6.00 per hourActual direct labor rate paid .............. $6.10 per hourStandard hours allowed for actual production 1,500 hoursLabor efficiency variance--unfavorable ..... $600

What are the actual hours worked? (M)a. 1,400. c. 1,598.b. 1,402. d. 1,600. AICPA adapted

Standard Hours Allowed48. The Fischer Company uses a standard costing system. For the month of December, the

following data have been assembled:Actual direct labor hours worked 5,800 hoursStandard direct labor rate $9 per hourLabor efficiency variance $1,800 unfavorable

The standard hours allowed for December production is: A. 5,400 hours. C. 5,800 hours.B. 5,600 hours. D. 6,000 hours. G & N 10e

28. ACE Company’s operations for the month just ended originally set up a 60,000 direct labor hour level, with budgeted direct labor of P960,000 and budgeted variable overhead of P240,000. The actual results revealed that direct labor incurred amounted to P1,148,000 and that the unfavorable variable overhead variance was P40,000. Labor trouble caused an

unfavorable labor efficiency variance of P120,000, and new employees hired at higher rates resulted in an actual average wage rate of P16.40 per hour. The total number of standard direct labor hours allowed for the actual units produced is (M)a. P52,500 c. P62,500b. P60,000 d. P70,000 RPCPA 1094

9. The Taxi Corporation makes a variety of leather goods. It uses standard costs and a flexible budget to aid planning and control. Budgeted variable overhead at a 45,000 direct labor hour level is P27,000.During April material purchases were P241,900. Actual direct-labor costs incurred were P140,700. The direct-labor usage variance was P5,100 unfavorable. The actual average wage rate was P0.20 lower than the average standard wage rate.The company uses a variable overhead rate of 20% of standard direct-labor cost for flexible budgeting purposes. Actual variable overhead for the month was P30,750.What were the standard hours allowed during the month of April? (D)A. 50,250 C. 48,550B. 58,625 D. 37,520 Pol Bobadilla

DL Rate Variance90. Pane Company's direct labor costs for April are as follows:

Standard direct labor hours 42,000Actual direct labor hours 41,200Total direct labor payroll $247,200Direct labor efficiency variance – favorable $3,840

What is Pane's direct labor rate variance? (M)A. $44,496 unfavorable. C. $49,440 favorable.B. $49,440 unfavorable. D. $50,400 favorable. Gleim

54. Borden Enterprises uses standard costing. For the month of April, the company reported the following data:

Standard direct labor rate: $10 per hourStandard hours allowed for actual production: 8,000Actual direct labor rate: $9.50 per hourLabor efficiency variance: $4,800 F

The labor rate variance for April is: (M)a. $3,760 U. c. $2,850 F.b. $3,760 F. d. $2,850 U. G & N 9e

66. Rainbow Company uses a standard cost system. Information about its direct labor costs for Product Lux for the month of January follows:

Standard hours allowed for actual production 1,500

CMA EXAMINATION QUESTIONS Page 29 of 142

MANAGEMENT ADVISORY SERVICES STANDARD COSTS AND VARIANCE ANALYSIS

Actual hourly rate paid P61.00Standard hourly rate P60.00Labor efficiency variance – favorable P6,000

How much was the direct labor rate variance? (M)A. 1,400 F C. 1,400 UB. 1,600 F D. 1,600 U Pol Bobadilla

DL Rate & Efficiency Variance - GivenActual Hours37. Acme Company produced 500 units with a $50 unfavorable labor rate variance. The labor use

variance was $180 favorable. Actual labor cost was $17,870. The standard wage rate was $9. Actual hours were (M)a. 1,520 c. 2,000b. 1,980 d. 2,020 D, L & H 9e

51. In a certain standard costing system the following results occurred last period: labor rate variance, $1,000 U; labor efficiency variance, $2,800 F; and the actual labor rate was $0.20 more per hour than the standard labor rate. The number of actual direct labor hours used last period was: (M)a. 9,000. c. 5,000.b. 5,400. d. 4,800. G & N 9e

Actual Wage Rate45. Jeter’s Company had a $510 unfavorable direct labor rate variance and a $1,000 favorable

efficiency variance. Jeter’s standard payroll was $11,200 at a standard wage of $10 per hour. What was the actual direct labor wage rate? (M)a. $9.56 c. $10.50b. $10.00 d. Some other number. D, L & H 9e

44. SUPERIOR Mfg. Co., using a standard cost system, furnished information on direct labor cost as follows:

Standard direct labor hours 75,000Actual direct labor hours 72,500Total payroll P275,500Unfavorable rate variance 14,500Favorable efficiency variance 10,000

What was SUPERIOR’s actual direct labor rate per hour? (E)a. P3.80 c. P4.20b. P4.00 d. P4.25 RPCPA 1097

Standard Hours19. A company made 1,200 units with a $550 favorable labor use variance. There was no labor

rate variance and actual labor cost was $19,250. The actual wage rate was $11. Standard labor time per unit is (M)a. 0.5 hours. c. 1.5 hours.b. 1.0 hour d. 2.0 hours. D, L & H 9e

Standard Direct Labor Rate44. Danner had a $550 favorable direct labor rate variance and a $720 unfavorable efficiency

variance. Danner paid $6,650 for 800 hours of labor. What was the standard direct labor wage rate? (M)a. $8.10 c. $9.00b. $8.31 d. Some other number. D, L & H 9e

53. For the month of April, Thorp Co.'s records disclosed the following data relating to direct labor:Actual cost ............... $10,000Rate variance ............. $ 1,000 favorableEfficiency variance ....... $ 1,500 unfavorable

For the month of April, actual direct labor hours amounted to 2,000. In April, Thorp's standard direct labor rate per hour was: (M)a. $5.50. c. $4.75.b. $5.00. d. $4.50. AICPA adapted

51. X’OR Co. uses a standard cost system, and data for its direct labor costs are summarized below:

Actual direct labor hours 72,500Standard direct labor hours 75,000Total direct labor payroll P275,500Direct labor rate variance – favorable 14,500

CMA EXAMINATION QUESTIONS Page 30 of 142

MANAGEMENT ADVISORY SERVICES STANDARD COSTS AND VARIANCE ANALYSIS

Direct labor efficiency variance - favorable 10,000The standard direct labor rate per hour is (M)a. P3.60 c. P4.00b. P3.80 d. P4.20 RPCPA 1096

Rate Variance & Budget VarianceActual Direct Labor Rate31. TAMARAW, Inc. has a maintenance shop where repairs to its motor vehicles are done. During

last month’s labor strike, certain records were lost. The actual input of direct labor hours was 1,000, and the resulting direct labor budget variance was a favorable P3,400. The standard direct labor rate was P28.00 per hour, but an unexpected labor shortage necessitated the hiring of higher-paid workers for some jobs and had resulted in a rate variance of P800. The actual direct labor rate was (M)a. P27.20 per hour c. P30.25 per hourb. P28.80 per hour d. P31.40 per hour RPCPA 0595 II-3

DL Rate & Efficiency VarianceQuestions 50 and 51 are based on the following information. CIA 1189 IV-17 & 18One of the items produced by a manufacturer of lawn and garden tools is a chain saw. The direct labor standard for assembling and testing a chain saw is 2.5 hours at $8 per hour. Budgeted production for October was 1,200 units. Actual production during the month was 1,000 units, and direct labor cost was $27,840 for 3,200 hours.

91. Using a two-variance system, what was the direct labor price (rate) variance for October? (E)A. $2,240 favorable. C. $3,840 favorable.B. $2,240 unfavorable. D. $5,600 unfavorable.

92. Using a two-variance system, what is the direct labor efficiency variance? (E)A. $2,240 unfavorable. C. $5,600 unfavorable.

B. $5,600 favorable. D. $6,090 favorable.

Questions 106 and 107 are based on the following information.The standard direct labor cost to produce one pound of output for a company is presented below. Related data regarding the planned and actual production activities for the current month for the company are also given below: CIA 0597 III-89 & 90, 0R98 IV-67 & 68NOTE: DLH = Direct labor hours

Direct labor standard: 0.4 DLH @ $12.00 per DLH = $4.80Planned production 15,000 poundsActual production 15,500 poundsActual direct labor costs (6,250 DLH) $75,250

93. The company's direct labor rate variance for the current month is A. $10 unfavorable. C. $248 unfavorable.B. $240 unfavorable. D. $250 unfavorable.

94. The company's direct labor efficiency variance for the current month is A. $600 unfavorable. C. $2,400 unfavorable.B. $602 unfavorable. D. $3,000 unfavorable.

Questions 91 & 92 are based on the following information. G & N 9eThe following labor standards have been established for a particular product:

Standard labor hours per unit of output .. 7.5 hoursStandard labor rate ...................... $15.25 per hour

The following data pertain to operations concerning the product for the last month:Actual hours worked ...................... 9,600 hoursActual total labor cost .................. $144,480Actual output ............................ 1,200 units

91. What is the labor rate variance for the month? (E)a. $1,920 F c. $1,920 Ub. $240 U d. $240 F

92. What is the labor efficiency variance for the month? (E)a. $7,230 U c. $7,230 Fb. $9,030 U d. $9,150 U

Questions 83-84 are based on the following information: G & N 10eThe following labor standards have been established for a particular product:

Standard labor hours per unit of output 8.1 hoursStandard labor rate $14.40 per hour

CMA EXAMINATION QUESTIONS Page 31 of 142

MANAGEMENT ADVISORY SERVICES STANDARD COSTS AND VARIANCE ANALYSIS

The following data pertain to operations concerning the product for the last month:Actual hours worked 8,700 hoursActual total labor cost $129,195Actual output 1,000 units

83. What is the labor rate variance for the month? A. $450 F C. $3,915 UB. $3,915 F D. $450 U

84. What is the labor efficiency variance for the month? A. $8,910 U C. $12,555 UB. $12,555 F D. $8,640 U

Questions 85-86 are based on the following information: G & N 10eThe following labor standards have been established for a particular product:

Standard labor hours per unit of output 9.8 hoursStandard labor rate $16.40 per hour

The following data pertain to operations concerning the product for the last month:Actual hours worked 7,900 hoursActual total labor cost $127,980Actual output 700 units

85. What is the labor rate variance for the month? A. $140 U C. $140 FB. $1,580 U D. $1,580 F

86. What is the labor efficiency variance for the month? A. $17,056 U C. $15,476 F B. $15,476 U D. $16,848 U

Total DL Variance & Actual DLHQuestions 1 & 2 are based on the following information. H & MSmith Company uses a standard cost system. The following information pertains to direct labor costs for the month of June.

Standard direct labor rate per hour $10.00Actual direct labor rate per hour $9.00Labor rate variance $12,000 favorableActual output 2,000 unitsStandard hours allowed for actual production 10,000 hours

95. What is the total labor budget variance for Smith Company?a. $12,000(F) d. $8,000(U)b. $8,000(F) e. $20,000(U)c. $12,000(U)

96. How many actual labor hours were worked during March for Smith Company?a. 10,000 d. 12,000b. 2,000 e. 1,000c. 1,200

DL Mix and Yield VarianceQuestions 54 and 55 are based on the following information. GleimLandeau Manufacturing Company has a process cost accounting system. A monthly analysis compares actual results with both a monthly plan and a flexible budget. Standard direct labor rates used in the flexible budget are established at the time the annual plan is formulated and held constant for the entire year. Standard direct labor rates in effect for the fiscal year ending June 30 and standard hours allowed for the output in April are

Standard DL Rate per Hour Standard DLH Allowed for OutputLabor class III $8.00 500Labor class II 7.00 500Labor class I 5.00 500

The wage rates for each labor class increased on January 1 under the terms of a new union contract negotiated in December of the previous fiscal year. The standard wage rates were not revised to reflect the new contract. The actual direct labor hours (DLH) worked and the actual direct labor rates per hour experienced for the month of April were as follows:

Actual Direct Labor Actual Rate per Hour Direct Labor HoursLabor class III $8.50 550Labor class II 7.50 650Labor class I 5.40 375

97. What is the labor yield variance for Landeau in April (rounded)? (M)A. $500 unfavorable. C. $825 favorable.B. $750 unfavorable. D. $1,500 favorable.

98. What is the labor mix variance for Landeau in April? (M)A. $325.00 unfavorable. C. $180.00 favorable.B. $66.67 unfavorable. D. $50.00 favorable.

CMA EXAMINATION QUESTIONS Page 32 of 142

MANAGEMENT ADVISORY SERVICES STANDARD COSTS AND VARIANCE ANALYSIS

Questions 56 through 58 are based on the following information. GleimThe information was presented as part of Question 4 on Part 4 of June 1978 CMA exam. A company’s standard direct labor rates in effect for the fiscal year ending June 30 and standard hours allowed for the output in April are

Standard DLRate per Hour

Standard DLHAllowed for Output

Labor class III $8.00 500Labor class II 7.00 500Labor class I 5.00 500

The wage rates for each labor class increased on January 1, under the terms of a new union contract. The standard wage rates were not revised.The actual direct labor hours (DLH) and the actual direct labor rates for April were as follows:

Actual Rate Actual DLHLabor class III $8.50 550Labor class II 7.50 650Labor class I 5.40 375

99. What is the labor yield variance (rounded)? (M)a. $500 c. $820b. $320 d. $515

100. What is the labor mix variance (rounded)? (M)a. $50.00 c. $66.67b. $320.00 d. $500.00

101. The labor mix and labor yield variances together equal the (E)a. Total labor variance.b. Labor rate variance.c. Labor efficiency variance.d. Sum of the labor efficiency and overhead efficiency variances.

Questions 1 thru 3 are based on the following information. H & MHarrigan Corporation uses two different types of labor to manufacture its product. The types of labor, Mixing and Finishing, have the following standards:

Labor Type Standard Mix Standard Unit Price Standard CostMixing 500 hours $10.00 per unit $5,000Finishing 250 hours 5.00 per unit $1,250Yield 4,000 units

During January, the following actual production information was provided:Labor Type Actual Mix

Mixing 4,500 unitsFinishing 3,000 units

Yield 36,000 units

102. What is the labor mix variance?a. $2,500 (F) c. $5,000 (F)b. $5,000 (U) d. $2,500 (U)

103. What is the labor yield variance?a. $6,250 (F) c. $6,250 (U)b. $4,000 (F) d. $4,000 (U)

104. What is the labor efficiency variance?a. $2,500 (F) c. $3,750 (F)b. $6,250 (U) d. $3,750 (U)

Questions 59 through 61 are based on the following information. GleimMountain View Hospital (MVH) has adopted a standard cost accounting system for evaluation and control of nursing labor. Diagnosis Related Groups (DRGs), instituted by the U.S. government for health insurance reimbursement, are used as the output measure in the standard cost system. A DRG is a patient classification scheme in which hospitals are regarded as multiproduct firms with inpatient treatment procedures related to the numbers and types of patient ailments treated. MVH has developed standard nursing times for the treatment of each DRG classification, and nursing labor hours are assumed to vary with the number of DRGs treated within a time period.The nursing unit on the fourth floor treats patients with four DRG classifications. The unit is staffed with registered nurses (RNs), licensed practical nurses (LPNs), and aides. The standard nursing hours and salary rates and actual numbers of patients for the month of May were as follows.

DRGClassification

Actual No.of Patients

Standard Hours per DRG Total Standard HoursRN LPN Aide RN LPN Aide

1 250 6 4 5 1,500 1,000 1,2502 90 26 16 10 2,340 1,440 900

CMA EXAMINATION QUESTIONS Page 33 of 142

MANAGEMENT ADVISORY SERVICES STANDARD COSTS AND VARIANCE ANALYSIS

3 240 10 5 4 2,400 1,200 9604 140 12 7 10 1,680 980 1,400

7,920 4,620 4,510

Standard Hourly RatesRN $12.00LPN 8.00Aide 6.00

The results of operations during May for the fourth floor nursing unit are presented below: RN LPN AideActual hours 8,150 4,300 4,400Actual salary $100,245 $35,260 $25,300Actual hourly rate $12.30 $8.20 $5.75

Because MVH does not have data to calculate variances by DRG, it uses a flexible budgeting approach to calculate labor variances for each reporting period by labor classification (RN, LPN, Aide). Labor mix and labor yield variances are also calculated because one labor input can be substituted for another. The variances are used by nursing supervisors and hospital administration to evaluate the performance of nurses.

105. What is the total flexible budget variance? (M)A. $2,205 favorable. C. $1,745 favorable.B. $2,205 unfavorable. D. $1,745 unfavorable.

106. What is the labor mix variance? (M)A. $2,205 unfavorable. C. $1,406 unfavorable.B. $2,205 favorable. D. $1,406 favorable.

107. What is the labor yield variance? (M)A. $1,908 favorable. C. $1,733 favorable.B. $1,866 favorable. D. $460 favorable.

Bonus Computation25. To improve productivity, ST. MICHAEL Corp. instituted a bonus plan where employees are

paid 75% of the time saved when production performance exceeds the standard level of production. The company computes the bonus on the basis of four-week periods. The standard production is set at 3 units per hour. Each employee works 37 hours per week, and the wage rate is P24 per hour. Below are data for one 4-week period:

Weekly Production (Units)Employee 1st 2nd 3rd 4th TotalALAN 107 100 110 108 425JOEL 104 110 115 115 444ROMY 108 112 112 133 465TONY 123 120 119 124 486

The employee who had the inconsistent performance (sometimes performing below standard) but got a bonus is (M)a. Alan = P36 bonus. c. Romy = P126 bonus.b. Joel = P54 bonus. d. Tony = P252 bonus. RPCPA 0594

Questions 62 and 63 are based on the following information. RPCPA 0584Ipil-ipil Woods Inc. grants bonus to its plant employees equal to 50% pay for the time saved in production. The company has set up a standard rate of production of 200 units of cutting board per hour. The standard pay per labor hour is P8. Factory overhead varies at the rate of P2.50 per hour.During the month of June, the employees worked a total of 25,000 direct labor hours and produced 6,000,000 units of cutting boards. The total variable factory overhead amounted to P62,500. Bonus checks are issued to employees in the month following the month in which the standards are exceeded.*. The labor bonus for the production in June is (M)

a. P6,250 c. P20,000b. P12,500 d. P38,000

*. The total net savings to the company for the month of June after deducting the bonus is (M)a. P25,000 c. P52,500b. P32,500 d. P60,000

CMA EXAMINATION QUESTIONS Page 34 of 142

MANAGEMENT ADVISORY SERVICES STANDARD COSTS AND VARIANCE ANALYSIS

OVERHEAD VARIANCESetting Predetermined RatesPlant CapacityTHE FOLLOWING INFORMATION APPLIES TO QUESTIONS 121 AND 122. HorngrenA manufacturing firm is able to produce 1,000 pairs of shoes per hour, at maximum efficiency. There are three eight-hour shifts each day. Due to unavoidable operating interruptions, production averages 800 units per hour. The plant actually operates only 27 days per month.

108. What is the theoretical capacity for the month of April? (E)a. 1,000,000 units c. 518,400 unitsb. 720,000 units d. 240,000 units

109. What is the practical capacity for the month of April? (E)a. 1,000,000 units c. 518,400 unitsb. 720,000 units d. 240,000 units

Budgeted Overhead17. Machine hours used to set the predetermined overhead rate were 50,000, actual hours were

48,000, and overhead applied was $120,000. Budgeted overhead for the year wasa. $115,200 c. $120,000b. $118,000 d. $125,000 D, L & H 9e

42. Machine hours used to set the predetermined overhead rate were 80,000, actual hours were 90,000, and overhead applied was $117,000. Budgeted overhead for the year wasa. $104,000 c. $131,625b. $117,000 d. Some other number. D, L & H 9e

Budgeted Fixed Factory Overhead34. The predetermined overhead rate (variable and fixed) is $7.50 per machine hour and the

denominator activity level is 135,000 machine hours. If the variable portion of the predetermined overhead rate is $3.00 per machine hours, then the budgeted fixed factory overhead for the year is: (M)a. $30,000. c. $405,000.b. $607,500. d. $1,012,500. G & N 9e

. ABC Company applies overhead at P8 per direct labor hour of which P3 was variable overhead. Budgeted directed labor hours were 80,000. Budgeted fixed overhead was (E)A. P200,000 C. P400,000B. P640,000 D. P240,000 RPCPA 1001

Denominator Hours77. Nevada Company uses a predetermined overhead application rate of $.30 per direct labor

hour. During the year it incurred $345,000 dollars of actual overhead, but it planned to incur $360,000 of overhead. The company applied $363,000 of overhead during the year. How many direct labor hours did the company plan to incur?a. 1,150,000 c. 1,200,000b. 1,190,000 d. 1,210,000 Barfields

110. In connection with a standard cost system being developed by Flint Co., the following information is being considered with regard to standard hours allowed for output of one unit of product:

HoursAverage historical performance for the past 3 years 1.85Production level to satisfy average consumer demand over a seasonal

time span 1.60Engineering estimates based on attainable performance 1.50Engineering estimates based on ideal performance 1.25

To measure controllable production inefficiencies, what is the best basis for Flint to use in establishing standard hours allowed? (E)a. 1.25 c. 1.60b. 1.50 d. 1.85 AICPA 1192

Standard Variable Overhead Rate9. Filter Company’s budget for overhead cost is: total overhead cost = $50,000 + ($4 x direct

labor hours). Standard direct labor time is 1.5 hours per unit of product. The standard wage rate is $6 per hour. Standard variable overhead cost for a unit of product isa. $4.00 c. $9.00b. $6.00 d. $10.00 D, L & H 9e

18. PALOS Manufacturing Co. has an expected production level of 175,000 product units for 19x7. Fixed factory overhead is P450,000 and the company applies factory overhead on the basis of expected production level at the rate of P5.20 per unit. The variable overhead cost per unit is (E)a. P2.57 c. P2.93b. P2.63 d. P3.02 RPCPA 1097

Standard Overhead Rate2. If annual overhead costs are expected to be P1,000,000 and 200,000 total labor hours are

anticipated (80% direct, 20% indirect), the overhead rate based on direct labor hours isA. P6.25 C. P25.00B. P5.00 D. P4.00 Pol Bobadilla

CMA EXAMINATION QUESTIONS Page 35 of 142

MANAGEMENT ADVISORY SERVICES STANDARD COSTS AND VARIANCE ANALYSIS

32. At the end of the year, actual manufacturing overhead costs were $110,000 and applied manufacturing overhead costs were $118,800. If the denominator activity for the year was 20,000 machine-hours, and if 22,000 standard machine-hours were allowed for the year's production, the predetermined overhead rate per machine-hour was: A. $5.00. C. $5.50.B. $5.94. D. $5.40. G & N 10e

34. Dean Company uses a standard cost system in which it applies manufacturing overhead to units of product on the basis of direct labor-hours. The company is preparing a flexible budget for next year and the following data are available:

At capacityDirect labor-hours 60,000Variable factory overhead $150,000Fixed factory overhead $240,000

Assume that Dean's denominator activity for the year is set at 80% of capacity. What would be the total predetermined overhead rate, based on direct labor-hours, for the year? A. $6.00. C. $7.50.B. $6.50. D. $8.13. G & N 10e

25. ABC Company is preparing a flexible budget for 2004 and the following maximum capacity estimates for the manufacturing division are available:

Direct labor hours 60,000 hoursVariable factory overhead P600,000Fixed manufacturing overhead P300,000

Assume that ABC’s expected capacity is 80% of maximum capacity. What would be the total factory overhead rate, based on direct labor hours, in a flexible budget at expected capacity?a. P18.75 C. P16.25b. P14.25 D. P15.00 Pol Bobadilla

5. Sales Company is preparing a flexible budget for 2004 and the following maximum capacity estimates for Assembly Department are available:

Direct labor hours 80,000 hoursVariable factory overhead P640,000Fixed manufacturing overhead P300,000

Assume that Sales’ expected capacity is 75% of maximum capacity. What would be the total factory overhead rate, based on direct labor hours, in a flexible budget at expected capacity?A. P13.00 C. P11.75B. P15.67 D. P11.00 Pol Bobadilla

35. At the end of the year, a company's Manufacturing Overhead account contained the following data:

Manufacturing OverheadActual $82,140 $78,260 Applied

$3,880If the denominator activity for the year was 40,000 machine-hours, and if 36,400 machine-hours were allowed for the year's production, then the predetermined overhead rate per machine-hour was: A. $2.15. C. $2.26.B. $1.96. D. $2.05. G & N 10e

Denominator Hours & Standard Variable Overhead Rate4. The MPG Reyes Co. owns 6 machines, each of which is run on a 48-hour week basis.

Annually, 100 working hours are allotted for each machine for periodic cleansing and greasing.During the year, the company was closed for 250 hours due to a strike triggered by labor-management disputes. With the intervention of the Labor Ministry, the company was able to resume operations. However, it suffered unexpected delays in obtaining the needed raw materials, thus resulting in additional loss of machine hours of 950 . Manufacturing expenses which vary with productive hours totaled P38,600 for the 6 machines.Assuming that the company normally closes for 3 weeks annually due to slack season, what is the normal number of machine hours worked and how much is the variable expense per working hours? (D)RPCPA 1086 a. b. c. d.Normal Number of Machine Hours 13,176 12,312 14,376 13,512Variable Expense per Working Hour P2.92 P3.14 P2.92 P3.14

ONE-WAY VARIANCEFlexible Budget Based on Actual Output79. Ashley Co. has developed the following flexible budget formula for monthly overhead:

For output of less than 200,000 units: $36,600 + $.80(units)For output of 200,000 units or more: $43,000 + $.80(units)

How much overhead should Ashley expect if the firm plans to produce 200,000 units?a. $52,600 c. $196,600b. $59,000 d. $203,000 Barfields

111. Scott Company uses the following flexible budget formula for annual maintenance costs:Total cost = $6,000 + $0.70 per machine hour

The current month's budget is based on planned machine time of 30,000 hours. Monthly maintenance cost included in this flexible budget is (M)

CMA EXAMINATION QUESTIONS Page 36 of 142

MANAGEMENT ADVISORY SERVICES STANDARD COSTS AND VARIANCE ANALYSIS

A. $20,500 C. $21,500B. $21,000 D. $27,000 Gleim

112. A company has the following budget formula for annual electricity expense in its shop:Expense = $7,200 + ($0.60 x units produced)

If management expects to produce 20,000 units during February, the appropriate monthly flexible budget allowance for the purpose of performance evaluation should be (M)a. $7,200 c. $12,600b. $12,000 d. $19,200 Gleim

22. Molds Corporation has developed the following flexible budget formula for annual indirect labor costs:

Total Cost = P300,000 + P5.00 per machine hourOperating budgets for the current month are based upon 18,000 machine hours of planned machine time. Indirect labor costs included in this planning budget are: (M)a. P300,000 c. P 90,000b. P390,000 d. P115,000 Pol Bobadilla

5. Saldua Co. uses a monthly cost formula for overhead of P50,000 + P30 for each direct labor hour worked. For the coming year, Saldua plans to manufacture 200,000 units. Each unit requires five minutes of direct labor. Saldua’s total budgeted overhead for the coming year isA. P550,000 C. P1,200,000B. P1,100,000 D. P650,000 Pol Bobadilla

113. A company prepares a flexible budget each month for manufacturing costs. Formulas have been developed for all costs within a relevant range of 5,000 to 15,000 units per month. The budget for electricity (a semivariable cost) is $19,800 at 9,000 units per month, and $21,000 at 10,000 units per month. How much should be budgeted for electricity for the coming month if 12,000 units are to be produced? (E)A. $26,400 C. $23,400B. $25,200 D. $22,200 CIA 0586 IV-12

*. Assuming the flexible budget of Dept. 1 is as follows:Hours of activity 1,500 2,000Fixed costs P7,500 P7,500Variable costs 5,250 7,000Total costs P12,750 P14,500

The budgeted total cost for this department at the 1,700 hour level of activity is (E)a. P13,450 c. P19,757b. P13,625 d. P27,250 RPCPA 1077

2. WORD PROCESSORS, Inc. provides computer processing services, and relevant data set up by the firm’s management are shown below:

No. of pages per hour 20No. of hours per month 500Variable costs per hour P30Fixed costs per month P10,000

For the month of May, 19x4, 12,000 pages are generated in 450 hours. The actual variable costs totaled P13,200, while the actual fixed costs equaled the estimated amount. The total standard cost for May wasa. P25,000 c. P30,000b. P27,500 d. P31,500 RPCPA 0594

114. Based on past experience, a company has developed the following budget formula for estimating its shipping expenses. The company’s shipments average 12 lbs. per shipment:

Shipping costs = $16,000 + ($0.50 x lbs. shipped)The planned activity and actual activity regarding orders and shipments for the current month are given in the following schedule:

Plan ActualSales orders 800 780Shipments 800 820Units shipped 8,000 9,000Sales $120,000 $144,000Total pounds shipped 9,600 12,300

The actual shipping costs for the month amounted to $21,000. The appropriate monthly flexible budget allowance for shipping costs for the purpose of performance evaluation would be (M)a. $20,680. c. $20,800.b. $20,920. d. $22,150. CMA 1295 3-24

*. Premised on past experience, Mayo Corp. adopted the following budgeted formula for estimating shipping expenses. The company’s shipments average 12 kilos per shipment.

Shipping costs = P8,000 + (0.25 x kgs. shipped)Planned Actual

Sales order 800 780Shipments 800 820Units shipped 8,000 9,000Sales 240,000 288,000Total kilograms shipped 9,600 12,300

The actual shipping costs for the month amounted to P10,500. The appropriate monthly flexible budget allowance for shipping costs for purposes of performance evaluation would be (M)

CMA EXAMINATION QUESTIONS Page 37 of 142

MANAGEMENT ADVISORY SERVICES STANDARD COSTS AND VARIANCE ANALYSIS

a. P10,250 c. P10,340b. P11,075 d. P10,400 RPCPA 0596

115. A company has developed the budget formula below for estimating its shipping expenses. Shipments have historically averaged 12 pounds per shipment.

Shipping costs = $18,000 + ($0.60 x Pounds shipped)The planned activity and actual activity regarding orders and shipments for the current month are given in the following schedule:

Plan ActualSales orders 800 780Shipments 800 820Units shipped 8,000 9,000Sales $120,000 $144,000Total pounds shipped 9,600 12,500

The actual shipping costs for the month amounted to $21,000. The appropriate monthly flexible budget allowance for shipping costs for the purpose of performance evaluation should be (M)a. $18,000 c. $23,760b. $18,492 d. $25,500 Gleim

Actual Output116. At the beginning of the year, Smith Inc. budgeted the following:

Units 10,000Sales $100,000Minus:Total variable expenses 60,000Total fixed expenses 20,000 Net income $ 20,000

Factory overhead:Variable $ 30,000Fixed 10,000

There were no beginning inventories. At the end of the year, no work was in process, total factory overhead incurred was $39,500, and underapplied factory overhead was $1,500. Factory overhead was applied on the basis of budgeted unit production. How many units were produced this year? (D)A. 10,250. C. 9,875.B. 10,000. D. 9,500. Gleim

Applied OverheadTotal Overhead Applied48. Machine hours used to set the predetermined overhead rate were 68,000, actual hours were

64,000, and budgeted overhead was $142,800. Overhead applied for the year wasa. $134,400 c. $142,800b. $136,500 d. $151,725 D, L & H 9e

86. One unit requires 2 direct labor hours to produce. Standard variable overhead per unit is $1.25 and standard fixed overhead per unit is $1.75. If 330 units were produced this month, what total amount of overhead is applied to the units produced?a. $990b. $1,980c. $660d. cannot be determined without knowing the actual hours worked Barfield

31. The Rowe Company uses a standard cost system in which it applies manufacturing overhead to units of product on the basis of machine-hours. During January, the company budgeted to incur $225,000 in manufacturing overhead cost and to operate at a denominator activity level of 25,000 machine-hours. At standard, each unit of finished product requires 3 machine-hours. The following cost and activity were recorded during January:

Total actual manufacturing overhead cost incurred $217,750Units of product completed 8,000Actual machine-hours worked 23,000

The amount of overhead cost that the company applied to Work in Process for January was: A. $217,750. C. $221,600.B. $225,000. D. $216,000. G & N 10e

30. The Ammon Company uses a standard cost system in which manufacturing overhead is applied to units on the basis of machine-hours. During July, the company budgeted $350,000 in manufacturing overhead cost at a denominator activity of 25,000 machine-hours. At standard, each unit of finished product requires 5 machine-hours. The follow cost and activity were recorded during July:

Total actual manufacturing overhead cost incurred $325,000Units of product completed 4,500Actual machine-hours worked 23,000

The amount of overhead cost that the company applied to work in process for July was: A. $292,500. C. $322,000.B. $315,000. D. $325,000. G & N 10e

33. The Adlake Company makes and sells a single product and uses a standard cost system. During October, the company budgeted $300,000 in manufacturing overhead cost at a

CMA EXAMINATION QUESTIONS Page 38 of 142

MANAGEMENT ADVISORY SERVICES STANDARD COSTS AND VARIANCE ANALYSIS

denominator activity of 20,000 machine-hours. At standard, each unit of finished product requires 5 machine-hours. The following cost and activity were recorded during October:

Total actual manufacturing overhead cost incurred $294,000Units of product completed 3,800Actual machine-hours worked 19,422

The amount of overhead cost that the company applied to work in process for October was: (M)a. $279,300. c. $294,000.b. $291,330. d. $285,000. G & N 9e

117. Union Company uses a standard cost accounting system. The following factory O/H and production data are available for August?

Standard fixed O/H rate per DLH $1Standard variable O/H rate per DLH $4Budgeted monthly DLH 40,000Actual DLH worked 39,500Standard DLH allowed for actual production 39,000Overall O/H variance – favorable $2,000

The applied factory O/H for August should be (E)a. $195,000 c. $197,500b. $197,000 d. $199,500 AICPA 1181 I-24

118. River Company uses a standard-cost accounting system. It applies overhead based on direct labor hours. The following overhead costs and production data are available for March:

Standard fixed overhead rate per DLH $1.50Standard variable overhead rate per DLH $5.00Budgeted monthly DLH 30,000Actual DLH worked 28,000Standard DLH allowed for actual production 27,500Overall overhead variance -- favorable $2,500

What is the applied factory overhead for March? (M)A. $137,500 C. $178,750B. $176,250 D. $182,000 Gleim

27. Overhead cost is applied to units based on direct labor hours. For April, total overhead cost was budgeted at $80,000 based on a denominator activity level of 20,000 direct labor hours for the month. The standard cost card indicates that each unit of finished product requires 2 direct labor-hours. The following data are available for April's activity:

Number of units produced 9,500Direct labor hours worked 19,500Actual total overhead cost incurred $79,500

What amount of total overhead cost would have been applied to production for the month of April? (E)a. $76,000. c. $79,500.b. $78,000. d. $80,000. AICPA, Adapted

33. The Judd Company uses a standard cost system in which it applies manufacturing overhead to units of product on the basis of machine-hours. During May, the company budgeted $320,000 in manufacturing overhead cost at a denominator activity of 20,000 machine-hours. At standard, each unit of finished product requires 4 machine-hours. The following cost and activity was recorded during May:

Total actual manufacturing overhead cost incurred $335,500Units of product completed 4,700Actual machine-hours worked 21,000

The amount of overhead cost that the company applied to Work in Process for May was: A. $336,000. C. $300,800.B. $335,500. D. $315,370. G & N 10e

Applied Fixed Overhead85. Actual fixed overhead is $33,300 (12,000 machine hours) and fixed overhead was estimated at

$34,000 when the predetermined rate of $3.00 per machine hour was set. If 11,500 standard hours were allowed for actual production, applied fixed overhead isa. $33,300.b. $34,000.c. $34,500.

CMA EXAMINATION QUESTIONS Page 39 of 142

MANAGEMENT ADVISORY SERVICES STANDARD COSTS AND VARIANCE ANALYSIS

d. not determinable without knowing the actual number of units produced. Barfield

Over- (Under-) Applied OverheadOver-applied Overhead24. Palo applies overhead based on direct labor cost. It had budgeted manufacturing overhead of

$500,000 and budgeted direct labor of $250,000. Actual overhead was $525,000, actual labor cost was $270,000. Overhead wasa. Over-applied by $15,000. c. Over-applied by $25,000.b. Over-applied by $20,000. d. Under-applied by $20,000. D, L & H 9e

7. ABC Company has prepared the following flexible budget for production costs: total production costs = $340,000 + $9x, where x is the number of units produced. ABC produced 20,000 units at a total cost of $490,000. The variance of actual costs from budgeted costs was a. $150,000 favorable. c. $30,000 unfavorable.b. $30,000 favorable. d. $90,000 unfavorable. D, L & H 9e

47. Monroe Company has prepared the following flexible budget for production costs: total production costs = $840,000 + $16x, where X is the number of units produced. Monroe produced 20,000 units at a total cost of $1,290,000. The variance of actual costs from budgeted costs wasa. $450,000 favorable. c. $130,000 unfavorable.b. $130,000 favorable. d. $450,000 unfavorable. D, L & H 9e

43. Cooke Company uses the equation $450,000 + $1.50 per direct labor hour to budget manufacturing overhead. Cooke has budgeted 150,000 direct labor hours for the year. Actual results were 156,000 direct labor hours and $697,500 total manufacturing overhead. The total overhead variance for the year isa. $4,500 favorable. c. $4,500 unfavorable.b. $18,000 favorable. d. $18,000 unfavorable. D, L & H 9e

*. The following data are presented:Budgeted Actual

Production in units 50,000 55,000Manufacturing overhead P750,000 P800,000Sales in units No data 47,000No beginning inventories

The under-applied or over-applied overhead is: (M)a. P25,000 under-applied. c. P75,000 over-applied.b. P25,000 over-applied. d. P75,000 under-applied. RPCPA 1097

91. Air Inc. uses a standard cost system. Overhead cost information for October is as follows:

Total actual overhead incurred $12,600Fixed overhead budgeted $3,300Total standard overhead rate per MH $4Variable overhead rate per MH $3Standard MHs allowed for actual production 3,500

What is the total overhead variance?a. $1,200 F c. $1,400 Fb. $1,200 U d. $1,400 U Barfield

119. Pane Company uses a job costing system and applies overhead to products on the basis of direct labor cost. Job No. 75, the only job in process on January 1, had the following costs assigned as of that date: direct materials, $40,000; direct labor, $80,000; and factory overhead, $120,000. The following selected costs were incurred during the year:

Traceable to jobs: Direct materials $178,000 Direct labor 345,000 $523,000Not traceable to jobs: Factory materials and supplies 46,000 Indirect labor 235,000 Plant maintenance 73,000 Depreciation on factory equipment 29,000 Other factory costs 76,000 459,000

Pane's profit plan for the year included budgeted direct labor of $320,000 and factory overhead of $448,000. Assuming no work-in-process on December 31, Pane's overhead for the year was A. $11,000 overapplied. C. $11,000 underapplied.B. $24,000 overapplied. D. $24,000 underapplied.

CMA Samp Q3-5

Under-applied Overhead*. Information on Bustos Manufacturing Company’s overhead costs is as follows:

Budgeted overhead based on standard direct-labor hours allowed P90,000Budgeted overhead based on actual direct-labor hours allowed P89,000Standard applied overhead P86,000Actual overhead P92,000

What is the total overhead variance (M)a. P4,000 favorable. c. P8,000 unfavorable.b. P6,000 unfavorable. d. P9,000 favorable. RPCPA 1079

120. Anderson Company prepared the following information using a flexible budget system. Percentage of total capacity

CMA EXAMINATION QUESTIONS Page 40 of 142

MANAGEMENT ADVISORY SERVICES STANDARD COSTS AND VARIANCE ANALYSIS

75% 90%Direct labor hours 30,000 36,000Variable factory overhead $52,500 $63,000Fixed factory overhead $144,000 $144,000Total factory overhead rate per DLH $6.55 $5.75

Anderson operated at 75% of capacity during the year. However, Anderson applied factory overhead based on 90% of capacity. If actual factory overhead was equal to the factory overhead budgeted for 75% of capacity, what is the amount of overhead variance for the year? (M)A. $28,500 underabsorbed. C. $24,000 underabsorbed.B. $28,500 overabsorbed. D. $24,000 overabsorbed. Gleim

121. Peters Company uses a flexible budget system and prepared the following information for the year

Percentage of total capacity80% 90%

Direct labor hours 24,000 27,000Variable factory O/H $48,000 $54,000Fixed factory O/H $108,000 $108,000Total factory O/H rate per DLH $6.50 $6.00

Peters operated at 80% capacity during the year but applied factory overhead based on the 90% capacity level. Assuming that actual factory O/H was equal to the budgeted amount for the attained capacity, what is the amount of O/H variance for the year? (M)a. $6,000 over-absorbed. c. $12,000 over-absorbed.b. $6,000 under-absorbed. d. $12,000 under-absorbed. AICPA 0581 I-

25

Actual Overhead25. Markham applies overhead at $4 per machine hour. During March it worked 10,000 hours and

over-applied overhead by $3,000. Actual overhead was a. $43,000 c. $37,000b. $40,000 d. None of the above. D, L & H 9e

122. The following were among Gage Co.’s 2000 costs:Normal spoilage $ 5,000Freight out 10,000Excess of actual manufacturing costs over standard costs 20,000Standard manufacturing costs 100,000Actual prime manufacturing costs 80,000

Gage’s 2000 actual manufacturing overhead was (M)

a. $40,000 c. $55,000b. $45,000 d. $120,000

Standard Rate & Applied OverheadQuestions 106-107 are based on the following information: G & N 10eA manufacturer of industrial equipment has a standard costing system based on direct labor-hours (DLHs) as the measure of activity. Data from the company's flexible budget for manufacturing overhead are given below:

Denominator level of activity 1,400 DLHsOverhead costs at the denominator activity level:Variable overhead cost $5,670Fixed overhead cost $27,860

The following data pertain to operations for the most recent period:Actual hours 2,000 DLHsStandard hours allowed for the actual output 1,710 DLHsActual total variable overhead cost $8,000Actual total fixed overhead cost $27,210

106.What is the predetermined overhead rate to the nearest cent? A. $23.95 C. $25.15B. $16.77 D. $17.61

107.How much overhead was applied to products during the period to the nearest dollar? A. $33,530 C. $47,900B. $40,955 D. $35,210

Standard Rate, Applied Overhead & Total Overhead VarianceQuestions 116 through 118 are based on the following information. L.J. McCarthyPatie Company uses a standard FIFO, process-cost system to account for its only product, Mituea. Patie has found that direct machine hours (DMH) provide the best estimate of the application of O/H. Four (4) standard direct machine hours are allowed for each unit.Using simple linear regression analysis in the form y = a + b(DMH), given that (A) equals fixed costs and (B) equals variable costs, Patie has developed the following O/H budget for a normal activity level of 100,000 direct machine hours:

ITEM (y) a bSupplies $ 0.50

CMA EXAMINATION QUESTIONS Page 41 of 142

MANAGEMENT ADVISORY SERVICES STANDARD COSTS AND VARIANCE ANALYSIS

Indirect Labor $ 54,750 6.50Depreciation -- Plant and Equipment 27,000Property Taxes and Insurance 32,300Repairs and Maintenance 14,550 1.25Utilities 3,400 4.75Total O/H $132,000 $13.00

Actual fixed O/H incurred was $133,250, and actual variable O/H was $1,225,000. Patie produced 23,500 equivalent units during the year using 98,700 direct machine hours.

123. What is the standard O/H rate? A. $13.00 per DMH. C. $14.32 per DMH.B. $1.32 per DMH D. $13.76 per DMH.

124. How much O/H should be applied to production? A. $1,413,384 C. $1,358,250B. $1,432,000 D. $1,346,080

125. What is the total O/H variance? A. $12,170 unfavorable. C. $55,134 favorable.B. $55,134 unfavorable. D. $73,750 favorable.

Flexible Budget based on Standard Input and Standard Variable Overhead RateQuestions 80 and 81 are based on the following information.Standard costs and budgetary control methods should be closely related. This relationship is especially important for factory overhead. Better control over factory overhead can be achieved if a flexible budget, rather than a fixed budget is used. The flexible budget for Kupang Corporation is summarized below:

Percent of Normal Operating Capacity80% 90% 100%* 110%

Variable overhead P21,000 P23,000 P25,000 P27,000Fixed overhead 50,000 50,000 50,000 50,000Total factory overhead P71,000 P73,000 P75,000 P77,000

* normal capacityIn accordance with the standards established, 100,000 units of product should be manufactured when the company operates at its normal capacity. The standard labor time per unit of product is 15 minutes. Actual production in 1980 was 90,000 units of product in 44,000 hours.*. What is the standard variable factory overhead rate per hour?(E)

a. P1.00 c. P2.00b. P1.50 d. P2.50 RPCPA 1081

*. What is the budgeted factory overhead adjusted to standard hours? (E)a. P67,500 c. P75,000b. P72,500 d. P90,500 RPCPA 1081

Questions 43 thru 46 are based on the following information. G & N 9ePollitt Potato Packers has a flexible budget for manufacturing overhead that is based on direct labor hours. The following overhead costs appear on the flexible budget at the 200,000 hour level of activity:

Variable overhead costs (total):CMA EXAMINATION QUESTIONS Page 42 of 142

MANAGEMENT ADVISORY SERVICES STANDARD COSTS AND VARIANCE ANALYSIS

Packing supplies .......... $120,000 Indirect labor ............ $180,000Fixed overhead costs (total): Utilities ................. $100,000 Insurance ................. $ 40,000 Rent ...................... $ 20,000

44. The flexible budget would show total variable overhead cost in dollars per direct labor hour as: (E)a. $0.60. c. $1.50.b. $0.90. d. $1.80.

43. At an activity level of 180,000 direct labor hours, the flexible budget would show indirect labor cost of: (E)a. $180,000. c. $144,000.b. $108,000. d. $162,000.

45. At an activity level of 180,000 direct labor hours, the flexible budget would show total budgeted fixed costs to be: (E)a. $100,000. c. $150,000.b. $144,000. d. $160,000.

46. At an activity level of 160,000 direct labor hours, the flexible budget would show the budgeted amount for utilities to be: (E)a. $80,000. c. $120,000.b. $100,000. d. $160,000.

TWO-WAY VARIANCEBudget Variance126. Samuel Company provided the following data for June production activity. Samuel uses a two-

way analysis of overhead variances.Actual variable factory overhead incurred $294,000Variable factory overhead rate per DLH $6.00Standard DLH allowed 49,500Actual DLH 48,000

The budget (controllable) variance for June, assuming that budgeted fixed overhead costs equal actual fixed costs, is (M)A. $3,000 favorable. C. $9,000 favorable.B. $6,000 unfavorable. D. $9,000 unfavorable. Gleim

127. Universal Company uses a standard cost system and prepared the following budget at normal capacity for the month of January:

Direct labor hours 24,000Variable factory O/H $48,000Fixed factory O/H $108,000Total factory O/H per DLH $6.50

Actual data for January were as follows:Direct labor hours worked 22,000Total factory O/H $147,000Standard DLH allowed for capacity attained 21,000

Using the two-way analysis of O/H variances, what is the budget (controllable) variance for January? (M)a. $3,000 favorable. c. $9,000 favorable.b. $13,500 unfavorable. d. $10,500 unfavorable. CPA 0583 I-39

128. Wheeler Company uses a standard-cost system. Wheeler prepared the following budget using normal capacity for the month of May:

Direct labor hours 36,000Variable factory overhead $72,000Fixed factory overhead $162,000

Actual results were as follows:Direct labor hours worked 33,000Total factory overhead $220,500Standard DLH allowed for capacity attained 31,500

What is the budget (controllable) variance for May using the two-way analysis of overhead variances? (M)A. $4,500 favorable. C. $7,500 unfavorable.B. $7,500 favorable. D. $13,500 unfavorable. Gleim

90. Martin Company uses a two-way analysis of overhead variances. Selected data for the April production activity are as follows:

Actual variable OH incurred $196,000Variable OH rate per MH $6Standard MHs allowed 33,000Actual MHs 32,000

Assuming that budgeted fixed overhead costs are equal to actual fixed costs, the controllable variance for April isa. $2,000 F. c. $4,000 F.

CMA EXAMINATION QUESTIONS Page 43 of 142

MANAGEMENT ADVISORY SERVICES STANDARD COSTS AND VARIANCE ANALYSIS

b. $4,000 U. d. $6,000 F. Barfield

34. GMA Company employs a standard absorption system for product costing. The standard cost of its product is as follows:

Direct materials P14.50Direct labor (2 direct labor hours at P8) 16.00Manufacturing overhead ( 2 DLH at P11) 22.00

The manufacturing overhead rate is based upon a normal activity level of 600,000 direct labor hours. Joker planned to produce 25,000 units each month during the year. The budgeted annual manufacturing overhead is:

Variable P3,600,000Fixed 3,000,000

During November, GMA produced 26,000 units. GMA used 53,500 direct labor hours in November at a cost of P433,350. Actual manufacturing overhead for the month was P250,000 fixed and P325,000 variable.The manufacturing overhead controllable variance for November isa. P13,000 unfavorable c. P3,000 favorable b. P10,000 favorable d. P4,000 favorable Pol Bobadilla

Questions 1 & 2 are based on the following information. CIA adapted.A company's only service department provides the following data:

Service Center Monthly Budget Service Hours Available Actual Monthly ExpenseCarpenter Shop $40,000 1,600 $47,800

It serves three producing departments that show the following budgeted and actual cost and service-hours data:

Carpenter ShopDepartment No. Estimated Services Required Actual Services Used

1 350 hrs. 600 hrs.2 800 hrs. 750 hrs.3 450 hrs. 650 hrs.

129. The sold-hour rate for the carpenter shop is:A. $29.88 C. $25.00B. $20.00 D. $23.90

130. The spending variance for the carpenter shop, assuming that 80% of the budgeted expense is fixed, is:A. $5,800 unfav. C. $5,800 fav.B. $7,800 unfav. D. $7,800 fav.

Volume Variance39. Gamma Corporation has total budgeted fixed costs of $150,000. Actual production was 8,000

units; normal capacity is 7,500 units. What was the volume variance? (E)a. $10,000 favorable c. $15,000 unfavorableb. $15,000 favorable d. $10,000 unfavorable D, L & H 9e

42. Monona Corporation has total budgeted fixed costs of $64,000. Actual production was 15,000 units; normal capacity is 16,000 units. What was the volume variance? (E)a. $4,000 favorable c. $4,267 unfavorableb. $4,267 favorable d. $4,000 unfavorable D, L & H 9e

13. ABC had $400,000 budgeted fixed overhead costs and based its standard on normal activity of 40,000 units. Actual fixed overhead costs were $430,000, actual production was 36,000 units, and sales were 30,000 units. The volume variance was (E)a. $30,000. c. $70,000.b. $40,000. d. $77,777. D, L & H 9e

13. ABC had $200,000 budgeted fixed overhead costs and based its standard on normal activity of 20,000 units. Actual fixed overhead costs were $215,000, actual production was 18,000 units, and sales were 15,000 units. The volume variance was (E)a. $15,000 c. $35,000b. $20,000 d. $38,888 D, L & H 9e

43. QUEEN Processing Co. has set its normal capacity at 24,000 hours for the current year. Fixed overhead was budgeted for P18,000 and variable overhead was budgeted for P72,000. If actual hours worked for the current year were 22,000, the idle capacity variance would be (E)a. P0 c. P6,000b. P1,500 d. P7,500 RPCPA 1095

*. TYD, Inc. reported the following data for 1996:Actual hours 120,000Denominator hours 150,000Standard hours allowed for output 140,000Fixed predetermined overhead rate P6 per hourVariable predetermined overhead rate P4 per hour

TYD’s 1996 volume variance was (M)a. P60,000 which is neither favorable nor under-applied.b. P60,000 favorable.c. No volume variance.d. P60,000 under-applied. RPCPA 1097

CMA EXAMINATION QUESTIONS Page 44 of 142

MANAGEMENT ADVISORY SERVICES STANDARD COSTS AND VARIANCE ANALYSIS

6. The following standards were developed based on a capacity of 180,000 direct labor hours as follows:

Standard costs per unit: Variable portion 2 hours @ P3 = P6 Fixed portion 2 hours @ P5 = P10

During April, 85,000 units were scheduled for production, but only 80,000 units were actually produced. The following data relate to April: Actual direct labor cost incurred was P644,000 for 165,000 actual hours worked. Actual overhead incurred totaled P1,378,000 (P518,000 variable plus P860,000

fixed). All inventories are carried at standard cost.The volume variances for April were (M)A. P100,000 U C. P50,000 FB. P100,000 F D. P50,000 U Pol Bobadilla

*. The following data were gathered from the Paliwas Company’s overhead costs for the January, 1983 production activity:

Actual total overhead incurred P112,500Budgeted fixed overhead P 40,500Standard direct-labor hours allowed for actual production 15,000Standard fixed overhead rate per direct-labor hour P2.25Standard variable overhead rate per direct-labor hour P3.25

Paliwas Company has been maintaining a standard absorption and flexible budgeting system. It also uses the two-variance method (two-way) for overhead variances. What is Paliwas volume (denominator) variance for January, 1983? (M)a. P6,750 favorable c. P8,250 favorableb. P6,750 unfavorable d. P8,250 unfavorable RPCPA 1083

38. Patridge Company uses a standard cost system in which it applies manufacturing overhead to units of product on the basis of direct labor hours. The information below is taken from the company's flexible budget for manufacturing overhead:

Percent of capacity 70% 80% 90%Direct labor hours 21,000 24,000 27,000Variable overhead $ 42,000 $ 48,000 $ 54,000Fixed overhead 108,000 108,000 108,000Total overhead $150,000 $156,000 $162,000

During the year, the company operated at exactly 80% of capacity, but applied manufacturing overhead to products based on the 90% level. The company's fixed overhead volume variance for the year was: (M)a. $6,000 unfavorable. c. $12,000 unfavorable.

b. $6,000 favorable. d. $12,000 favorable. AICPA, Adapted

28. Hero Company uses a flexible budget system and prepared the following information for the year:

Percent of Capacity 80 Percent 90 PercentDirect labor hours 24,000 27,000Variable factory overhead P 54,000 P 60,750Fixed factory overhead P108,000 P 108,000Total factory overhead rate per DLH P6.75 P6.25

Hero operated at 80 percent of capacity during the year, but applied factory overhead based on the 90 percent capacity level. Assuming that actual factory overhead was equal to the budgeted amount of overhead, how much was the overhead volume variance for the year? (M)a. P12,000 unfavorable c. P16,750 unfavorableb. P12,000 favorable d. P16,750 favorable Pol Bobadilla

28. Fidelity Company uses a flexible budget system and prepared the following information for the year:

Percent of Capacity 80 Percent 90 PercentDirect labor hours 24,000 27,000Variable factory overhead P 54,000 P 60,750Fixed factory overhead P81,000 P 81,000Total factory overhead rate per DLH P5.625 P5.25

Fidelity operated at 80 percent of capacity during the year, but applied factory overhead based on the 90 percent capacity level. Assuming that actual factory overhead was equal to the budgeted amount of overhead, how much was the overhead volume variance for the year? (M)a. P9,000 unfavorable c. P9,000 favorableb. P15,750 unfavorable d. P15,750 favorable Pol Bobadilla

21. Meteor Company employs a standard absorption system for product costing. The standard cost of its product is as follows:

Direct materials P14.50Direct labor (2 direct labor hours at P9) 18.00Manufacturing overhead ( 2 DLH at P12) 24.00

The manufacturing overhead rate is based upon the annual normal activity level of 600,000 direct labor hours. Meteor planned to produce 25,000 units each month during the year. The budgeted annual manufacturing overhead is:

Variable P4,200,000Fixed 3,000,000

CMA EXAMINATION QUESTIONS Page 45 of 142

MANAGEMENT ADVISORY SERVICES STANDARD COSTS AND VARIANCE ANALYSIS

During November, Meteor produced 26,000 units. Meteor used 53,500 direct labor hours in November at a cost of P433,350. Actual manufacturing overhead for the month was P250,000 fixed and P325,000 variable.The manufacturing overhead volume variance for November is (M)a. P10,000 unfavorable c. P5,000 unfavorableb. P10,000 favorable d. P5,000 favorable Pol Bobadilla

26. Lord Industries manufactures a single product. Variable production costs are P10 and fixed production costs are P75,000. Lord uses a normal activity of 10,000 units to set its standard costs. Lord began the year with no inventory, produced 11,000 units and sold 10,500 units. The volume variance under each product costing are: (M)Pol Bobadilla A. B. C. D.Absorption Costing P3,750 P3,750 P7,500 P7,500Variable P 0 P7,500 P3,750 P 0

13. Atlas Company employs a standard absorption system for product costing. The standard cost of its product is as follows:

Direct materials P14.50Direct labor (2 direct labor hours at P8) 16.00Manufacturing overhead ( 2 DLH at P11) 22.00

The manufacturing overhead rate is based upon a normal activity level of 600,000 direct labor hours. Atlas planned to produce 25,000 units each month during the year. The budgeted annual manufacturing overhead is:

Variable P3,600,000Fixed 3,000,000

During November, Atlas produced 26,000 units. Atlas used 53,500 direct labor hours in November at a cost of P433,350. Actual manufacturing overhead for the month was P250,000 fixed and P325,000 variable.The manufacturing overhead volume variance for November isa. P12,000 favorable c. P3,000 favorableb. P10,000 favorable d. P9,000 favorable Pol Bobadilla

Budgeted Capacity*. JKL Co. has total budgeted fixed costs of P75,000. Actual production of 19,500 units resulted

in a $3,000 favorable volume variance. What normal capacity was used to determine the fixed overhead rate? (M)a. 18,750 c. 17,590b. 20,313 d. 16,500 RPCPA 1001

*. Eastern Co. has total budgeted fixed costs of $150,000. Actual production of 39,000 units resulted in a $6,000 favorable volume variance. What normal capacity was used to determine the fixed overhead rate? (M)a. 33,000 c. 40,560b. 37,500 d. 40,625 D, L & H 9e

49. Western Co. has total budgeted fixed costs of $72,000. Actual production of 5,500 units resulted in a $6,000 unfavorable volume variance. What normal capacity was used to determine the fixed overhead rate? (M)a. 5,000 c. 6,000b. 5,500 d. 5,077 D, L & H 9e

Budgeted Fixed Overhead Costs41. Western Company has a standard fixed cost of $8 per unit. At an actual production of 8,000

units a favorable volume variance of $12,000 resulted. What were total budgeted fixed costs? (E)a. $52,000 c. $76,000b. $64,000 d. $80,000 D, L & H 9e

37. Alpha Company has a standard fixed cost of $10 per unit. At an actual production of 16,000 units an unfavorable volume variance of $20,000 resulted. What were total budgeted fixed costs? (E)a. $140,000 c. $180,000b. $160,000 d. $150,000 D, L & H 9e

32. Web Company uses a standard cost system in which manufacturing overhead is applied to units of product on the basis of machine hours. During February, the company used a denominator activity of 80,000 machine hours in computing its predetermined overhead rate. However, only 75,000 standard machine hours were allowed for the month's actual production. If the fixed overhead volume variance for February was $6,400 unfavorable, then the total budgeted fixed overhead cost for the month was: (M)a. $96,000. c. $100,000.b. $102,400. d. $98,600. G & N 9e

Actual Output38. Beta Company has a standard fixed cost of $10 per unit using a normal capacity of 11,000

units. An unfavorable volume variance of $12,000 resulted. What was the volume produced? (E)a. 9,800 c. 12,200b. 11,000 d. 10,000 D, L & H 9e

CMA EXAMINATION QUESTIONS Page 46 of 142

MANAGEMENT ADVISORY SERVICES STANDARD COSTS AND VARIANCE ANALYSIS

48. Sigma Company has a standard fixed cost of $18 per unit using a normal capacity of 9,000 units. A favorable volume variance of $18,000 resulted. What was the volume produced? (E)a. 8,000 c. 10,000b. 9,000 d. 9,500 D, L & H 9e

Budget and Volume VarianceApplied Overhead40. OPAL Co. uses the two-way variance analysis for overhead performance. The budgeted

factory overhead includes monthly fixed costs of P1,200,000 plus variable costs of P96 per direct labor hour. Total standard direct labor hours allowed for the May, 19x8 production was 43,200 hours. Analysis showed that, in May, 19x8, the controllable variance of P24,000 is unfavorable while the volume variance of P12,000 is favorable. How much was the applied factory overhead in May 19x8? (M)a. P5,335,200 c. P5,359,200b. P5,347,200 d. P5,371,200 RPCPA 0598

Actual and Applied OverheadProblems 90 and 91 are based on the following information RPCPA 0589The MABINI CANDY FACTORY has the following budgeted factory overhead costs:

Budgeted fixed monthly factory overhead costs P85,000Variable factory overhead P4.00 per direct labor hour

For the month of January, the standard direct labor hours allowed were 25,000. An analysis of the factory overhead shows that in January, the factory had an unfavorable budget (controllable) variance of P3,500 and a favorable volume variance of P1,200. The factory uses a two-way analysis of factory overhead variances.*. The actual factory overhead incurred in January was (M)

a. P186,200 c. P181,500b. P188,500 d. P103,500

*. The applied factory overhead in January was (M)a. P188,500 c. P186,200b. P183,800 d. P103,500

Budget and Volume VarianceQuestions 92 and 93 are based on the following information. RPCPA 1078Assuming actual factory overhead is P7,250; budgeted fixed overhead is P3,600; variable overhead rate is P2.00 per hour and the standard hours in the product are 2,000 hours –*. The controllable variance is (E)

a. unfavorable at P350 c. favorable at P350b. unfavorable at P3,250 d. favorable at P3,250

*. Using data given above, and assuming that the fixed factory overhead rate is P1.20 per hour – the volume variance is (E)a. P1,000 c. P1,400b. P1,200 d. P1,600

Questions 94 and 95 are based on the following information. RPCPA 0581Roig Enterprises manufactures different types of aluminum products for different industries. Standard cost accounting systems are used. The following data are available:

Actual total overhead P40,000Budgeted fixed costs P10,200Total overhead application rate per standard direct labor hour P 2.50Actual hours used 13,980Normal activity in hours 12,000Standard hours allowed 15,000

The company uses a two-way analysis of overhead variances.

*. The controllable variance of Roig Enterprises is (M)a. P2,500 favorable c. P5,050 favorableb. P2,550 unfavorable d. P5,050 unfavorable

*. The volume variance of Roig Enterprises is (M)a. P2,550 favorable c. P9,990 favorableb. P2,550 unfavorable d. P9,990 unfavorable

Questions 96 and 97 are based on the following information. RPCPA 1080Beacon Company manufactures various types of plastic and rubber coated tubing products for various industries. Standard cost accounting system is used. The following are available:

Actual total overhead P 44,000Budgeted fixed costs P 12,600

CMA EXAMINATION QUESTIONS Page 47 of 142

MANAGEMENT ADVISORY SERVICES STANDARD COSTS AND VARIANCE ANALYSIS

Total overhead application rate per standard direct labor hour P 2.50Actual hours used 16,000Standard hours allowed 17,000Normal activity in hours 14,000

The company uses a two-way analysis of overhead variances.

*. The controllable variance of Beacon Company is (M)a. P1,500 favorable. c. P4,200 favorable.b. P1,500 unfavorable. d. P4,200 unfavorable.

*. The volume variance of Beacon Company is (M)a. P2,700 favorable. c. P7,500 favorable.b. P2,700 unfavorable. d. P7,500 unfavorable.

Questions thru are based on the following information. CMA adaptedJ. R. Richard Company employs a standard absorption system for product costing. The standard cost of its product is as follows:

Direct materials $14.50Direct labor (2 direct labor hours x $8) 16.00Manufacturing overhead (2 direct labor hours x $11) 22.00Total standard cost $52.50

The manufacturing overhead rate is based upon a normal activity level of 600,000 direct labor hours. Richard planned to produce 25,000 units each month during the year. The budgeted annual manufacturing overhead is:

Variable $3,600,000Fixed 3,000,000

$6,600,000During November, Richard produced 26,000 units. Richard used 53,500 direct labor hours in November at a cost of $433,350. Actual manufacturing overhead for the month was $250,000 fixed and $325,000 variable.

131. The manufacturing overhead controllable variance for November is (M)A. $9,000 unfavorable C. $9,000 favorableB. $13,000 unfavorable D. $4,000 favorable

132. The manufacturing overhead volume variance for November is:(M)A. $12,000 unfavorable D. $9,000 unfavorableB. $10,000 unfavorable E. $1,000 favorableC. $3,000 unfavorable

ComprehensiveQuestions 116 through 118 are based on the following information. L.J. McCarthyPatie Company uses a standard FIFO, process-cost system to account for its only product, Mituea. Patie has found that direct machine hours (DMH) provide the best estimate of the application of O/H. Four (4) standard direct machine hours are allowed for each unit.Using simple linear regression analysis in the form y = a + b(DMH), given that (A) equals fixed costs and (B) equals variable costs, Patie has developed the following O/H budget for a normal activity level of 100,000 direct machine hours:

ITEM (y) a bSupplies $ 0.50Indirect Labor $ 54,750 6.50Depreciation -- Plant and Equipment 27,000Property Taxes and Insurance 32,300Repairs and Maintenance 14,550 1.25Utilities 3,400 4.75Total O/H $132,000 $13.00

Actual fixed O/H incurred was $133,250, and actual variable O/H was $1,225,000. Patie produced 23,500 equivalent units during the year using 98,700 direct machine hours.

CMA EXAMINATION QUESTIONS Page 48 of 142

MANAGEMENT ADVISORY SERVICES STANDARD COSTS AND VARIANCE ANALYSIS

133. What is the standard O/H rate? (M)A. $13.00 per DMH. C. $14.32 per DMH.B. $1.32 per DMH D. $13.76 per DMH.

134. How much O/H should be applied to production? (M)A. $1,413,384 C. $1,358,250B. $1,432,000 D. $1,346,080

135. What is the total O/H variance? (M)A. $12,170 unfavorable. C. $55,134 favorable.B. $55,134 unfavorable. D. $73,750 favorable.

Predetermined Overhead Rate and Applied OverheadQuestions 81 & 82 are based on the following information. G & N 9eA manufacturer of industrial equipment has a standard costing system based on machine-hours (MHs) as the measure of activity. Data from the company's flexible budget for manufacturing overhead are given below:

Denominator level of activity ................. 3,900 MHsOverhead costs at the denominator activity level: Variable overhead cost ...................... $33,345 Fixed overhead cost ......................... $61,425

The following data pertain to operations for the most recent period:Actual hours 3,900 MHsStandard hours allowed for the actual output 3,952 MHsActual total variable overhead cost $32,565Actual total fixed overhead cost $60,675

81. What is the predetermined overhead rate to the nearest cent? (M)a. $23.91 c. $24.30b. $24.30 d. $23.91

82. How much overhead was applied to products during the period to the nearest dollar? (M)a. $93,240 c. $96,034b. $94,770 d. $94,770

Questions 83 & 84 are based on the following information. G & N 9eA manufacturer of industrial equipment has a standard costing system based on direct labor-hours (DLHs) as the measure of activity. Data from the company's flexible budget for manufacturing overhead are given below:

Denominator level of activity 8,000 DLHsOverhead costs at the denominator activity level:

Variable overhead cost $56,400 Fixed overhead cost $100,800

The following data pertain to operations for the most recent period:Actual hours 7,800 DLHsStandard hours allowed for the actual output 7,735 DLHsActual total variable overhead cost $54,210Actual total fixed overhead cost $100,200

83. What is the predetermined overhead rate to the nearest cent? (M)a. $19.30 c. $19.80b. $19.65 d. $20.15

84. How much overhead was applied to products during the period to the nearest dollar? (M)a. $151,993 c. $157,200b. $154,410 d. $153,270

Volume Variance & Over- (Under-) Applied OverheadQuestions 22 and 23 are based on the following information. RPCPA 1079Factory overhead for the Cabanatuan Co. has been estimated as follows:

Fixed overhead P30,000Variable overhead 90,000Estimated direct labor hours 40,000

Production for the month reached 75% of the budget, and actual factory overhead totaled P86,000.

22. The over- (under-) applied factory overhead was (E)a. P34,000 c. P4,000b. (P4,000) d. None of the above.

23. The favorable (unfavorable) idle capacity variance was (E)a. P4,000 c. P7,500b. (P7,500) d. None of the above

THREE-WAY VARIANCE - CMASpending Variance136. Cara Williams, a supervisor, controls her department's costs. The following data relate to her

department for the month of June:Factory Overhead Budgeted ActualVariable $100,000 $106,250Fixed 31,250 33,750

What was the department's total spending variance for June? (E)

CMA EXAMINATION QUESTIONS Page 49 of 142

MANAGEMENT ADVISORY SERVICES STANDARD COSTS AND VARIANCE ANALYSIS

A. $8,750 U. C. $3,750 F.B. $6,250 U. D. $2,500 U. Gleim

70. Using the information presented below, calculate the total overhead spending variance. (E)Budgeted Actual

Units produced 5,000 4,500Variable overhead (2 DLH at P2 per DLH) P4/ unit P19,500Fixed overhead P10,000 P10,300Direct labor hours 10,000 9,500

A. P500 U C. P1,000 UB. P800 U D. P1,300 U Pol Bobadilla

137. The following information is available from the Tyro Company:Actual factory O/H $15,000Fixed O/H expenses, actual $7,200Fixed O/H expenses, budgeted $7,000Actual hours 3,500Standard hours 3,800Variable O/H rate per DLH $2.50

Assuming that Tyro uses a three-way analysis of O/H variances, what is the spending variance? (E)a. $750 favorable. c. $950 favorable.b. $750 unfavorable. d. $200 unfavorable. AICPA 0578 I-32

138. Coleman Company compiled the following information:Actual factory overhead $22,500Fixed overhead expenses, actual $10,800

Fixed overhead expenses, budgeted $10,500Actual hours 5,250Standard hours 5,700Variable overhead rate per DLH $3.80

What is the spending variance assuming Coleman uses a three-way analysis of overhead? (E)A. $9,660 unfavorable. C. $7,950 favorable.B. $8,250 favorable. D. $7,950 unfavorable. Gleim

CMA EXAMINATION QUESTIONS Page 50 of 142

MANAGEMENT ADVISORY SERVICES STANDARD COSTS AND VARIANCE ANALYSIS

Variable Overhead Efficiency Variance139. The following information pertains to Roe Co.’s 2000 manufacturing operations:

Standard direct manufacturing labor hours per unit 2Actual direct manufacturing labor hours 10,500Number of units produced 5,000Standard variable overhead per standard direct labor hour $3Actual variable overhead $28,000

Roe’s 2000 unfavorable variable overhead efficiency variance was (E)a. $0 c. $2,000b. $1,500 d. $3,500 AICPA 1192

140. The following data relate to Tray Co.'s manufacturing operations: Standard direct labor hours per unit 3Actual direct labor hours 24,500Number of units produced 8,000Standard variable overhead per standard direct labor hour $2Actual variable overhead $46,000

Tray's variable overhead efficiency variance is (E)A. $0 C. $2,000 F.B. $1,000 U. D. $3,000 F. Gleim

35. The following data are the actual results for Bustos Company for the month of May:Actual output 4,500 unitsActual variable overhead P360,000Actual fixed overhead P108,000Actual machine time 14,000 MH

Standard cost and budget information for Bustos Company follows:Standard variable overhead rate P6.00 per MHStandard quantity of machine hours 3 hours per unitBudgeted fixed overhead P777,600 per yearBudgeted output 4,800 unit per month

The overhead efficiency variance is (M)a. P3,000 Favorable c. P5,400 Favorableb. P3,000 Unfavorable d. P5,400 Unfavorable Pol Bobadilla

Spending & Efficiency VarianceQuestions 1 and 2 are based on the following information. Carter & UsryThe following information relates to Department 1 of Ruiz Company for the fourth quarter. The total overhead variance is divided into three variances: spending, variable efficiency, and volume.

Actual total overhead (fixed plus variable) $178,500Budget formula $110,000+ $.50 per hourTotal overhead application rate $1.50 per hourActual hours worked 121,000

141. What was the spending variance in this department during the quarter? (E)A. $8,000 favorable C. $8,000 unfavorableB. $4,500 favorable D. $4,500 unfavorable

142. What was the variable efficiency variance in this department during the quarter?A. $4,500 favorable C. $4,500 unfavorableB. $8,000 favorable D. $8,000 unfavorable

Actual Fixed oOerhead and Overhead Efficiency VarianceQuestions 1 and 2 are based on the following information. Flamholtz & DiamondMeldouville Company has just finished reviewing the results of its operations for the current period. The company’s chief accountant told management that the total overhead spending variance was unfavorable by $1,000. However, the variable overhead portion of this variance was favorable by $200. The company’s volume variance, according to the accountant, was favorable by $300. During the period Medouville applied $1,800 of fixed overhead to production. The adjustment at the end of the period to allocate under-or-overapplied overhead included a debit of $200 to the Factory Overhead Control account.

. Meldouville’s actual fixed overhead costs for the period amounted to (M)A. $3,300 C. $900B. $300 D. $2,700

. Meldouville’s overhead efficiency variance for the period was (M)A. $900 favorable. C. $1,500 favorable.B. $1,100 favorable. D. $500 unfavorable.

ComprehensiveQuestions 1 thru 5 are based on the following information.Beebo Company uses a standard cost system as a means of control for their manufacturing business. The firm has done an analysis of its overhead cost behavior patterns relative to its activity base of direct labor hours (DLH), and reports the following findings:

Variable overhead Indirect material $1.20 per DLH Indirect labor 2.15 per DLH Other indirect costs 3.45 per DLH

CMA EXAMINATION QUESTIONS Page 51 of 142

MANAGEMENT ADVISORY SERVICES STANDARD COSTS AND VARIANCE ANALYSIS

Annual Fixed Costs Salaries $100,000 Depreciation 14,000 Rent 30,000

Beebo’s annual budget is based on total production of 240,000 units, for which 2.5 DLH per unit is required. It may be assumed that Beebo’s fixed costs and production activity occur evenly throughout the year.During its first month of operations in the current year Beebo produced 22,000 units, logged 57,200 direct labor hours,and reported the following cost figures.

Actual variable overhead costs $360,000Total fixed costs 14,000

. Beebo’s predetermined overhead application rate is (M)A. $6.80 per DLH D. $9.40 per DLHB. $7.04 per DLH E. $9.76 per DLHC. $8.14 per DLH

. Beebo’s overhead speanding variance for the month is (M)A. $10,000 unfavorable. D. $28,688 favorable.B. $22,000 unfavorable. E. $40,688 favorable.C. $26,960 favorable.

. Beebo’s overhead efficiency variance for the month is (M)A. $14,960 unfavorable. D. $48,960 favorable.B. $15,488 unfavorable. E. $50,688 favorable.C. $27,200 favorable.

. Beebo’s overhead volume variance for the month is (M)A. $528 unfavorable. D. $34,000 favorable.B. $1,200 favorable. E. $35,200 favorable.C. $1,728 unfavorable.

. Beebo’s overhead for the month was (M)A. Overapplied by $28,688 D. Overapplied by $14,960B. Underapplied by $22,000 E. Underapplied by $34,000C. Overapplied by $13,200

Questions 106 through 109 are based on the following information. RPCPA 1084Standard costs and budgetary control methods must be closely related. This relationship is particularly applicable for factory overhead. A flexible budget allows better control over factory overhead than a fixed budget. The flexible budget for Sta. Maria Corporation is presented below:

Percent of Normal Operating Capacity80% 90% 100%* 110%

Variable overhead P 72,000 P 81,000 P 90,000 P 99,000Fixed overhead 45,000 45,000 45,000 45,000Total factory overhead

P117,000 P126,000 P135,000 P144,000

* Normal capacityIn accordance with standards established, 90,000 units of product should be manufactured when the company operates its normal capacity. The standard labor time per unit of product is 20 minutes.Actual production in 1983 was 75,000 units of product in 24,000 hours.

CMA EXAMINATION QUESTIONS Page 52 of 142

MANAGEMENT ADVISORY SERVICES STANDARD COSTS AND VARIANCE ANALYSIS

*. What is the standard variable factory overhead rate per hour? (M)a. P1.00 c. P3.00b. P2.00 d. P4.50

*. What is the budgeted factory overhead adjusted to standard hours allowed for units actually produced? (M)a. P112,500 c. P120,000b. P117,000 d. P135,000

*. What is the budgeted factory overhead adjusted to actual hours worked. (M)a. P112,500 c. P120,000b. P117,000 d. P135,000

*. What is the factory overhead applied to production (based on standard hours? (M)a. P112,500 c. P120,000b. P117,000 d. P135,000

THREE-WAY VARIANCE – Spending, Idle Capacity & Efficiency VarianceQuestions 1 through 7 are based on the following information. RPCPA 0593The U. R. Good Company manufactures a product, using standard costs as follows:

1. Standard costs per unit: Material - 7 kilos at P3.50 per kilo Labor - 8 hours at P1.75 per hour Overhead: Fixed - P1.15 per hour or P9.20 per unit Variable - P0.85 per hour or P6.80 per unit2. Overhead applied on direct labor hours3. Actual performance (one month) a) Volume produced - 800 b) Labor hours - 6,300 c) Overhead - P13,200 d) Material cost - P3.45 per kilo e) Labor cost - P1.80 per hour f) Material used - 4,800 kilos

1. Material price variance is (E)a. P 240 favorable. c. P240 unfavorable.b. P2,800 unfavorable. d. P360 favorable.

2. Labor rate variance is (E)a. P400 favorable. c. P175 favorable.b. P315 unfavorable. d. P500 unfavorable

3. Total material variance is (E)a. P2,800 favorable. c. P3,000 unfavorable.b. P2,800 unfavorable. d. P3,040 favorable.

4. Total overhead variance is (E)a. P200 favorable. c. P400 unfavorable.b. P500 favorable. d. P400 favorable.

5. Material quantity variance is (E)a. P200 favorable. c. P240 favorable.b. P250 unfavorable. d. P2,800 favorable.

6. Overhead idle capacity variance isa. P435 favorable. c. P1,035 unfavorable.b. P435 unfavorable. d. P1,035 favorable.

7. Overhead efficiency variance isa. P435 favorable. c. P435 unfavorable.b. 200 favorable. d. 200 unfavorable.

FOUR-WAY VARIANCEFlexible Variable Budget (Based on Actual Output)*. Pranic Corp. uses flexible budgeting for cost control. It produced 5,400 units of product for the

month just ended incurring an indirect materials cost of P26,000. Its master budget for the year showed an indirect materials cost of P360,000 at a production volume of 72,000 units. A flexible budget for the month’s production would show indirect material cost of (E)a. P27,000 c. P27,950b. P26,000 d. P23,400 RPCPA 0597

143. RedRock Company uses flexible budgeting for cost control. RedRock produced 10,800 units of product during March, incurring an indirect materials cost of $13,000. Its master budget for the year reflected an indirect materials cost of $180,000 at a production volume of 144,000 units. A flexible budget for March production should reflect indirect materials costs of (E)a. $13,000 c. $13,975b. $13,500 d. $11,700 CMA 1291 3-26

5. For the period just ended, LAMBDA Co. budgeted its variable overhead at P40 per direct labor hour and fixed overhead at P480,000. Budgeted production volume was 8,000 units and the production time, which was the basis for allocation of variable and fixed overhead, was budgeted at 0.80 hour per unit. The actual results for the period were: fixed overhead,

CMA EXAMINATION QUESTIONS Page 53 of 142

MANAGEMENT ADVISORY SERVICES STANDARD COSTS AND VARIANCE ANALYSIS

P552,000; variable overhead, P283,480; units produced, 7,460; direct labor hours used, 5,595. What was the budgeted variable overhead for the actual volume attained? (M)a. P223,800 c. P238,720b. P226,784 d. P283,480 RPCPA 1094

Variable Overhead Spending Variance45 Antaya Company uses the equation $375,000 + $1.20 per direct labor hour to budget

manufacturing overhead. Antaya has budgeted 75,000 direct labor hours for the year. Actual results were 81,000 direct labor hours, $388,000 fixed overhead, and $98,600 variable overhead. The variable overhead spending variance for the year is (E)a. $1,400 c. $37,200.b. $8,600 d. $15,600. L & H 10e

49. Barron Company has a standard variable costs as follows:Materials, 3 pounds at $4.00 per pound $12.00Labor, 2 hours at $10.00 per hour 20.00Variable overhead, $7.50 per labor hour 15.00

$47.00During September, Barron produced 5,000 units, using 9,640 labor hours at a total wage of $94,670 and incurring $78,600 in variable overhead. The variable overhead spending variance is (E)a. $6,300 unfavorable. c. $2,700 favorable.b. $3,600 unfavorable. d. Some other number. D, L & H 9e

56. The following standards for variable manufacturing overhead have been established for a company that makes only one product:

Standard hours per unit of output 5.6 hoursStandard variable overhead rate $12.00 per hour

The following data pertain to operations for the last month:Actual hours 2,600 hoursActual total variable overhead cost $31,330Actual output 400 units

What is the variable overhead spending variance for the month? (E)a. $112 F c. $4,450 Ub. $130 U d. $4,338 U G & N 9e

53. The following standards for variable manufacturing overhead have been established for a company that makes only one product:

Standard hours per unit of output 1.2 hoursStandard variable overhead rate $19.80 per hour

The following data pertain to operations for the last month:Actual hours 2,100 hoursActual total variable overhead cost $40,740Actual output 1,600 units

What is the variable overhead spending variance for the month? A. $2,724 U C. $840 F B. $3,492 U D. $768 U G & N 10e

31. Tyro Company has a standard cost system in which it applies manufacturing overhead to units of product on the basis of direct labor hours (DLHs). The following information is available:

Actual total overhead costs $15,000Actual fixed overhead costs $ 7,200Budgeted fixed overhead costs $ 7,000Actual hours worked 3,500 DLHsStandard hours allowed for the output 3,800 DLHsVariable overhead rate $2.50 per DLH

Based on these data, what is the variable overhead spending variance? (M)a. $1,700 favorable. c. $950 favorable.b. $750 unfavorable. d. $1,500 unfavorable. AICPA, Adapted

33. Baltimore, Inc. analyzes manufacturing overhead in the production of its only one product, Blu. The following set of information applies to the month of May, 2003:

Budgeted ActualUnits produced 40,000 38,000Variable manufacturing overhead P4/DLH P16,400Fixed manufacturing overhead P20/DLH P88,000Direct labor hours 6 minutes/unit 4,200 hours

How much was the variable overhead spending variance?a. P400 Favorable c. P1,200 Favorableb. P400 Unfavorable d. P1,200 Unfavorable Pol Bobadilla

Standard Variable Overhead Rate30. At Overland Company, maintenance cost is exclusively a variable cost that varies directly with

machine-hours. The performance report for July showed that actual maintenance costs totaled $9,800 and that the associated spending variance was $200 unfavorable. If 8,000 machine-hours were actually worked during July, the budgeted maintenance cost per machine-hour was: (M)

CMA EXAMINATION QUESTIONS Page 54 of 142

MANAGEMENT ADVISORY SERVICES STANDARD COSTS AND VARIANCE ANALYSIS

a. $1.20. c. $1.275.b. $1.25. d. $1.225. G & N 9e

29. At Jacobson Company, indirect labor is a variable cost that varies with direct labor hours. Last month's performance report showed that actual indirect labor cost totaled $5,780 for the month and that the associated spending variance was $245 F. If 24,100 direct labor hours were actually worked last month, then the flexible budget cost formula for indirect labor must be (per direct labor hour): (M)a. $0.20. c. $0.30.b. $0.25. d. $0.35. G & N 9e

28. At Eady Company, maintenance is a variable cost that varies directly with machine-hours. The performance report for July showed that actual maintenance costs totaled $8,650 and that the associated spending variance was $250 unfavorable. If 5,000 machine-hours were actually worked during July, the budgeted maintenance cost per machine-hour was: (M)A. $1.73. C. $1.68.B. $1.78. D. $1.83. G & N 10e

Variable Overhead Spending Variance & Overhead RateQuestions 93-94 are based on the following information: G & N 10eThe Richie Company employs a standard costing system in which variable manufacturing overhead is assigned to production on a basis of number of machine setups. Data for the month of October include the following: Variable manufacturing overhead cost incurred: $42,750 Total variable overhead variance: $5,430 favorable Standard machine setups allowed for actual production: 2,920 setups Actual machine setups incurred: 2,850 setups

93. The standard variable overhead rate per machine setup is: (M)A. $16.91. C. $15.00. B. $12.78. D. $16.50.

94. The variable overhead spending variance is: (M)A. $4,275 favorable. C. $1,050 unfavorable.B. $4,275 unfavorable. D. $1,050 favorable.

Variable Overhead Efficiency Variance144. Compute the variable efficiency variance, using the following data: (E)

Standard labor hours per good unit produced 2Good units produced 1,000Actual labor hours used 2,100Standard variable overhead per standard labor hour $3Actual variable overhead $6,500

A. $200 favorable C. $300 favorableB. $200 unfavorable D. $300 unfavorable CIA adapted

49. Barron Company has a standard variable costs as follows:Materials, 3 pounds at $4.00 per pound $12.00Labor, 2 hours at $10.00 per hour 20.00Variable overhead, $7.50 per labor hour 15.00

$47.00During September, Barron produced 5,000 units, using 9,640 labor hours at a total wage of $94,670 and incurring $78,600 in variable overhead. The variable overhead efficiency variance is (E)a. $6,300 unfavorable. c. $2,700 favorable. b. $3,600 unfavorable. d. Some other number. D, L & H 9e

55. The following standards for variable manufacturing overhead have been established for a company that makes only one product:

Standard hours per unit of output ...... 7.8 hoursStandard variable overhead rate ........ $12.55 per hour

The following data pertain to operations for the last month:Actual hours ........................... 2,900 hoursActual total variable overhead cost .... $36,975Actual output .......................... 200 units

What is the variable overhead efficiency variance for the month? (E)a. $17,397 U c. $312 Fb. $16,817 U d. $17,085 U G & N 9e

52. The following standards for variable manufacturing overhead have been established for a company that makes only one product:

Standard hours per unit of output 3.5 hoursStandard variable overhead rate $15.20 per hour

The following data pertain to operations for the last month:Actual hours 3,800 hoursActual total variable overhead cost $59,090

CMA EXAMINATION QUESTIONS Page 55 of 142

MANAGEMENT ADVISORY SERVICES STANDARD COSTS AND VARIANCE ANALYSIS

Actual output 800 unitsWhat is the variable overhead efficiency variance for the month? A. $15,550 U C. $16,530 U B. $15,200 U D. $980 F G & N 10e

Actual Direct Labor Hours65. The following information relates to Orc Company’s 2003 manufacturing activities:

Standard direct labor hours per unit 2Number of units produced 5,000Standard variable overhead per standard direct labor hours P3Actual variable overhead P28,000Unfavorable overhead efficiency variance P1,500

The number of actual direct labor hours are (M)A. 10,500 C. 11,000B. 10,000 D. Indeterminate Pol Bobadilla

Variable Overhead Budget Variance145. A company's flexible budget shows an expected variable delivery expense of $160,000 when

sales are 50,000 units. If sales total 52,000 units, and the actual delivery expense is $163,000, what will be the flexible budget variance for delivery expense? (E)A. $3,000 unfavorable. C. $3,000 favorable.B. $3,400 unfavorable. D. $3,400 favorable. Gleim

146. Baxter Corporation's master budget calls for the production of 5,000 units of product monthly. The master budget includes indirect labor of $144,000 annually; Baxter considers indirect labor to be a variable cost. During the month of April, 4,500 units of product were produced, and indirect labor costs of $10,100 were incurred. A performance report utilizing flexible budgeting would report a budget variance for indirect labor of (E)A. $1,900 unfavorable. C. $1,900 favorable.B. $700 favorable. D. $700 unfavorable. CMA 0687 4-18

49. Barron Company has a standard variable costs as follows:Materials, 3 pounds at $4.00 per pound $12.00Labor, 2 hours at $10.00 per hour 20.00Variable overhead, $7.50 per labor hour 15.00

$47.00During September, Barron produced 5,000 units, using 9,640 labor hours at a total wage of $94,670 and incurring $78,600 in variable overhead. The variable overhead budget variance is (E)a. $6,300 unfavorable. c. $2,700 favorable.b. $3,600 unfavorable. d. Some other number. D, L & H 9e

Variable Overhead Spending Variance. Given for the variable factory overhead of GHI Products, Inc.: P39,500 actual input at

budgeted rate, P41,500 flexible budget based on standard input allowed for actual output, P2,500 favorable flexible budget variance. Compute the spending variance. (M)a. P500 unfavorable. c. P500 favorable.b. P2,000 favorable. d. P2,000 unfavorable. RPCPA 1001

Variable Overhead Spending Variance & Budget VarianceStandard Hours54. The Waters Company has a standard costing system. Variable manufacturing overhead is

assigned to production on the basis of machine hours. The following data are available for July: Actual variable manufacturing overhead cost incurred: $45,240 Actual machine hours worked: 3,200 Variable overhead spending variance: $6,840 unfavorable Total variable overhead variance: $9,240 unfavorableThe standard number of machine hours allowed for July production is: A. 3,200 hours. C. 3,400 hours.B. 3,000 hours. D. 4,540 hours. G & N 10e

ComprehensiveVariable Overhead Flexible Budget Variance & Spending VarianceTHE FOLLOWING INFORMATION APPLIES TO QUESTIONS 57 AND 58. HorngrenKellar Corporation manufactured 1,500 chairs during June. The following variable overhead data pertain to June.

Budgeted variable overhead cost per unit $ 12.00Actual variable manufacturing overhead cost $16,800Flexible-budget amount for variable manufacturing overhead $18,000Variable manufacturing overhead efficiency variance $360 unfavorable

147. What is the variable overhead flexible-budget variance? (E)a. $1,200 favorable c. $1,560 favorableb. $360 unfavorable d. $1,200 unfavorable

148. What is the variable overhead spending variance? (E)a. $840 unfavorable c. $1,200 unfavorableb. $1,200 favorable d. $1,560 favorable

Variable Overhead Flexible Budget Variance & Efficiency VarianceQuestions xx thru xx are based on the following information. H & M

CMA EXAMINATION QUESTIONS Page 56 of 142

MANAGEMENT ADVISORY SERVICES STANDARD COSTS AND VARIANCE ANALYSIS

Gem Company’s standard variable overhead rate is $3 per direct labor hour, and each unit requires 2 standard direct labor hours. During October, Gem recorded 12,000 actual direct labor hours, $37,000 actual variable overhead costs, and 5,800 units of product manufactured.

149. What is the total variable overhead budget variance for October for Gem? (E)a. $1,200 (U) d. $2,200 (F)b. $600 (U) e. $2,200 (U)c. $1,000 (U)

150. What is the variable overhead efficiency variance for October for Gem? (E)a. $2,200 (U) d. $1,200 (U)b. $2,200 (F) e. $1,000 (U)c. $600 (U)

THE FOLLOWING INFORMATION APPLIES TO QUESTIONS 59 AND 60. HorngrenPatel Corporation manufactured 1,000 coolers during October. The following variable overhead data pertain to October.

Budgeted variable overhead cost per unit $ 9.00Actual variable manufacturing overhead cost $8,400Flexible-budget amount for variable manufacturing overhead $9,000Variable manufacturing overhead efficiency variance $180 unfavorable

151. What is the variable overhead flexible-budget variance? (E)a. $600 favorable c. $780 favorableb. $420 unfavorable d. $600 unfavorable

152. What is the variable overhead spending variance? (E)a. $420 unfavorable c. $600 unfavorableb. $600 favorable d. $780 favorable

Variable Overhead Spending & Efficiency VarianceQuestions 10 and 11 are based on the following information. Pol BobadillaThe Huber Company produces a pesticide. At the beginning of the year, Huber had the following standard cost sheet:

Direct materials (5 lbs. @ P11.60) P 80.00Direct labor (1.5 hrs @ P90.00) 135.00Fixed overhead (1.5 hrs @ P20.00) 30.00Variable overhead (1.5 hrs. @ P15.00) 22.00Standard cost per unit P267.50

The Huber Company computes its overhead rates using practical volume, which is 36,000 units. The actual results for the year are: Units produced, 35,000 units. Materials purchased, 186,000 pounds @ P15. Materials used, 180,000 pounds. Direct labor, 53,000 hours @ P89.50 Fixed overhead, P725,000. Variable overhead, P802,500

10. The amount of variable overhead spending variance is: (E)A. P15,000 U C. P15,000 FB. P7,500 U D. P7,500 F

11. The amount of variable overhead efficiency variance is (E)A. P7,500 F C. P7,500 UB. P15,000 F D. P15,000 U

Questions 91-92 are based on the following information: G & N 10eThe following standards for variable manufacturing overhead have been established for a company that makes only one product:

Standard hours per unit of output 7.0 hoursStandard variable overhead rate $11.35 per hourThe following data pertain to operations for the last month:Actual hours 7,500 hoursActual total variable overhead cost $87,000Actual output 1,000 units

91. What is the variable overhead spending variance for the month? (E)A. $7,550 F C. $1,875 FB. $7,550 U D. $1,875 U

92. What is the variable overhead efficiency variance for the month? (E)A. $5,800 F C. $1,750 FB. $5,800 U D. $5,675 U

Questions 89-90 are based on the following information: G & N 10eThe following standards for variable manufacturing overhead have been established for a company that makes only one product:

Standard hours per unit of output 8.1 hoursStandard variable overhead rate $14.85 per hour

The following data pertain to operations for the last month:

CMA EXAMINATION QUESTIONS Page 57 of 142

MANAGEMENT ADVISORY SERVICES STANDARD COSTS AND VARIANCE ANALYSIS

Actual hours 8,600 hoursActual total variable overhead cost $130,720Actual output 1,000 units

89. What is the variable overhead spending variance for the month? (E)A. $3,010 F C. $10,435 UB. $3,010 U D. $10,435 F

90. What is the variable overhead efficiency variance for the month? (E)A. $7,600 F C. $7,600 UB. $2,835 F D. $7,425 U

Questions 74-75 are based on the following information: G & N 10eA manufacturing company that has only one product has established the following standards for its variable manufacturing overhead. The company uses direct labor-hours (DLHs) as its measure of activity.

Standard hours per unit of output 3.6 DLHsStandard variable overhead rate $15.35 per DLH

The following data pertain to operations for the last month:Actual direct labor-hours 7,300 DLHsActual total variable overhead cost $112,785Actual output 1,800 units

74. What is the variable overhead spending variance for the month? (E)A. $13,317 U C. $730 FB. $730 U D. $13,317 F

75. What is the variable overhead efficiency variance for the month? (E)A. $12,669 F C. $12,669 U B. $648 F D. $12,587 U

Questions 98 & 99 are based on the following information. G & N 9eThe following standards for variable manufacturing overhead have been established for a company that makes only one product:

Standard hours per unit of output ...... 1.6 hoursStandard variable overhead rate ........ $11.55 per hour

The following data pertain to operations for the last month:Actual hours ........................... 4,900 hours

Actual total variable overhead cost .... $58,310Actual output .......................... 3,000 units

98. What is the variable overhead spending variance for the month? (E)a. $2,870 U c. $1,715 Ub. $2,870 F d. $1,715 F

99. What is the variable overhead efficiency variance for the month? (E)a. $1,680 F c. $1,155 Ub. $1,190 U d. $1,190 F

Questions 65 & 66 are based on the following information. G & N 9eA manufacturing company that has only one product has established the following standards for its variable manufacturing overhead. The company uses machine-hours as its measure of activity.

Standard hours per unit of output 8.1 machine-hours Standard variable overhead rate $14.30 per machine-hour

The following data pertain to operations for the last month:Actual hours ........................... 1,700 machine-hoursActual total variable overhead cost .... $24,905Actual output .......................... 200 units

65. What is the variable overhead spending variance for the month? (E)a. $1,739 U c. $595 Ub. $595 F d. $1,739 F

66. What is the variable overhead efficiency variance for the month? (E)a. $1,172 F c. $1,172 Ub. $567 F d. $1,144 U

Questions 67 & 68 are based on the following information. G & N 9eA manufacturing company that has only one product has established the following standards for its variable manufacturing overhead. The company uses direct labor-hours (DLHs) as its measure of activity.

Standard hours per unit of output 7.2 DLHsStandard variable overhead rate $14.20 per DLH

The following data pertain to operations for the last month:Actual direct labor-hours 5,100 DLHsActual total variable overhead cost $72,165Actual output 600 units

CMA EXAMINATION QUESTIONS Page 58 of 142

MANAGEMENT ADVISORY SERVICES STANDARD COSTS AND VARIANCE ANALYSIS

67. What is the variable overhead spending variance for the month?(E)a. $10,821 U c. $10,821 Fb. $255 U d. $255 F

68. What is the variable overhead efficiency variance for the month?(E)a. $11,076 U c. $11,037 Ub. $11,037 F d. $216 U

Fixed Overhead Flexible Budget26. Aurora applies overhead at $8 per direct labor hour of which $3 is variable overhead.

Budgeted direct labor hours were 90,000. Budgeted fixed overhead was (E)a. $270,000 c. $720,000b. $450,000 d. None of the above. D, L & H 9e

*. Broker Corp.’s budget shows straight-line depreciation on machinery of P516,000 based on the annual production volume of 103,200 units of product. In July, it produced 8,170 units of product, and the accounts had actual depreciation on machinery of P41,000. It controls manufacturing costs with a flexible budget. The flexible budget for the depreciation of the machine for July is (M)a. P38,950 c. P41,000b. P43,000 d. P40,850 RPCPA 0596

153. Simson Company's master budget shows straight-line depreciation on factory equipment of $258,000. The master budget was prepared at an annual production volume of 103,200 units of product. This production volume is expected to occur uniformly throughout the year. During September, Simson produced 8,170 units of product, and the accounts reflected actual depreciation on factory machinery of $20,500. Simson controls manufacturing costs with a flexible budget. The flexible budget amount for depreciation on factory machinery for September would be (M)A. $19,475. C. $20,500.B. $20,425. D. $21,500. CMA 0686 4-23

Fixed Overhead Spending (Budget) Variance154. A company's flexible budget shows an expected fixed cost of $100,000 for straight-line

depreciation when sales total 50,000 units. If sales total 52,000 units, and the actual cost of depreciation is $103,000, what will be the budget variance? (E)A. $1,000 favorable. C. $1,000 unfavorable.B. $3,000 favorable. D. $3,000 unfavorable. Gleim

155. Selo Imports uses flexible budgeting for the control of costs. The company’s annual master budget includes $324,000 for fixed production supervisory salaries at a volume of 180,000

units. Supervisory salaries are expected to be incurred uniformly through the year. During September, 15,750 units were produced and production supervisory salaries incurred were $28,000. A performance report for September should reflect a budget variance of (E)a. $350 F c. $1,000 Ub. $350 U d. $1,000 F CMA 0687 4-17

36. Jaune Company uses a standard cost system in which it applies manufacturing overhead to units of product on the basis of direct labor hours (DLHs). The following data pertain to last month's operations:

Budgeted fixed overhead costs $5,000Actual fixed overhead costs $5,500Standard hours allowed for output 2,400 DLHsPredetermined overhead rate ($2 variable+ $3 fixed) $5 per DLH

The fixed overhead budget variance is: (E)a. $500 U. c. $2,200 U.b. $500 F. d. $1,700 U. G & N 9e

35. Mauve Company uses a standard cost system in which it applies manufacturing overhead to units of product on the basis of direct labor hours (DLHs). The following data pertain to last month:

Actual hours worked 2,400 DLHsBudgeted fixed overhead costs $10,000Actual fixed overhead costs $10,400Standard hours allowed 2,500 DLHsPredetermined overhead rate $5 per DLH

The fixed overhead budget variance is: (E)a. $400 U. c. $300 F.b. $500 F. d. $300 U. G & N 9e

36. Long Company analyzes its manufacturing overhead in the production of its only one product, Shorts. The following set of information applies to the month of January 2004:

Budgeted ActualUnits produced 40,000 38,000Variable manufacturing overhead P4/DLH P16,400Fixed manufacturing overhead P20/DLH P85,000Direct labor hours 6 minutes/unit 4,000 hours

What is the fixed overhead spending variance? (M)a. P5,000 Favorable c. P1,600 Unfavorableb. P1,600 Favorable d. P5,000 Unfavorable Pol Bobadilla

CMA EXAMINATION QUESTIONS Page 59 of 142

MANAGEMENT ADVISORY SERVICES STANDARD COSTS AND VARIANCE ANALYSIS

Volume VarianceFixed Overhead Rate & Volume VarianceQuestions 76-77 are based on the following information: G & N 10eThe Forkes Company uses a standard cost system in which overhead costs are applied to products on the basis of direct labor-hours (DLHs). The following data applied to the company's activities for the month of June:

Actual fixed overhead cost incurred $161,450Denominator activity 50,000 DLHsNumber of units completed 21,000 UnitsFixed overhead budget variance $11,450 UnfavorableStandard direct labor-hours per unit 3 DLHs

76. The fixed portion of the predetermined overhead rate for June is: (E)A. $3.00. C. $3.78.B. $3.23. D. $3.46.

77. The volume variance for June is: (E)A. $44,954 Unfavorable. C. $39,000 Unfavorable.B. $39,000 Favorable. D. $44,954 Favorable.

Questions 78-79 are based on the following information: G & N 10eThe Malcolm Company uses a standard cost system in which manufacturing overhead costs are applied to products on the basis of direct labor-hours (DLHs). The standards call for 3 hours of direct labor per unit produced. The following data pertain to the company's manufacturing overhead for the month of July:

Actual fixed overhead cost incurred $28,450Denominator activity 8,400 DLHsNumber of units produced 2,900 unitsBudget variance $3,250 Unfavorable

78. The fixed portion of the predetermined overhead rate for June is: (E)A. $3.11. C. $3.77.B. $3.39. D. $3.00.

79. The volume variance for July is: (E)A. $1,131 F. C. $1,131 U.B. $900 F. D. $900 U.

Budgeted Fixed Overhead & Applied Fixed Overhead

Questions 94 & 95 are based on the following information. G & N 9eThe Tate Company uses a standard costing system in which manufacturing overhead is applied to units of product on the basis of direct labor hours (DLHs). The company recorded the following costs and activity for September:

Cost: Actual fixed overhead costs incurred $61,400 Volume variance $2,850 Unfavorable Fixed portion of the predetermined overhead rate $0.95 per DLHActivity: Number of units completed 22,800 Standard direct labor hours allowed per unit of product 2.5 DLHs Denominator activity 60,000 DLHs

94. The amount of fixed manufacturing overhead cost applied to work in process during September was: (E)a. $61,400. c. $54,150.b. $57,000. d. $59,850.

95. The amount of fixed overhead cost contained in the company’s flexible budget for manufacturing overhead for September was: (E)a. $61,400. c. $60,000.b. $57,000. d. $58,550.

Budgeted Fixed Overhead & Denominator HoursQuestions 79 & 80 are based on the following information. G & N 9eThe Claus Company makes and sells a single product and uses standard costing. During January, the company actually used 8,700 direct labor-hours (DLHs) and produced 3,000 units of product. The standard cost card for one unit of product includes the following:

Variable factory overhead: 3.0 DLHs @ $4.00 per DLH.Fixed factory overhead: 3.0 DLHs. @ $3.50 per DLH.

For January, the company incurred $22,000 of actual fixed overhead costs and recorded a $875 favorable volume variance.

79. The budgeted fixed factory overhead cost for January is: (E)a. $31,500. c. $32,375.b. $30,625. d. $33,250.

80. The denominator level of activity in direct labor-hours (DLHs) used by Claus in setting the predetermined overhead rate for January is: (E)a. 9,500 DLHs. c. 8,750 DLHs.

CMA EXAMINATION QUESTIONS Page 60 of 142

MANAGEMENT ADVISORY SERVICES STANDARD COSTS AND VARIANCE ANALYSIS

b. 9,250 DLHs. d. 10,500 DLHs.

Budgeted Fixed Overhead, Applied Fixed Overhead & Fixed Overhead Budget VarianceQuestions 40 thru 42 are based on the following information. G & N 9eThe Murray Company makes and sells a single product. The company recorded the following activity and cost data for May:

Number of units completed 45,000 unitsStandard direct labor-hours allowed per unit of product 1.5 DLHSBudgeted direct labor-hours (denominator activity) 72,000 DLHSActual fixed overhead costs incurred $66,000Volume variance $4,275 U

The fixed portion of the predetermined overhead rate is $0.95 per direct labor-hour.

40. The amount of fixed overhead contained in the company's overhead flexible budget for May was: (E)a. $64,125. c. $68,400.b. $67,500. d. $70,275.

41. The amount of fixed manufacturing overhead cost applied to work in process during May was: (E)a. $61,725. c. $42,750.b. $62,700. d. $64,125.

42. The fixed overhead budget variance for May was: (E)a. $2,400 U. c. $6,000 U.b. $2,400 F. d. $6,000 F.

Questions 103-105 are based on the following information: G & N 10eThe Murray Company uses a standard cost system in which it applies manufacturing overhead on the basis of direct labor-hours (DLHs). The standard calls for 1.5 direct labor-hours per unit of product. The company recorded the following activity and cost data for May:

Activity: Number of units produced 40,100 UnitsDenominator activity 64,000 DLHsCost:Actual fixed overhead costs incurred $56,000Volume variance $3,465 UnfavorableFixed portion of the predetermined overhead rate $0.90 per DLH

103.The amount of fixed overhead contained in the company's budget for May was: (E)A. $54,135. C. $59,465. B. $60,150. D. $57,600.

104.The amount of fixed manufacturing overhead cost applied during May was: (E)A. $52,535. C. $36,090.B. $54,135. D. $50,400.

105.The fixed overhead budget variance for May was: (E)A. $8,000 F. C. $1,600 F.B. $8,000 U. D. $1,600 U.

Questions 100-102 are based on the following information: G & N 10eThe Chase Company has a standard cost system in which manufacturing overhead is applied to units of product on the basis of direct labor-hours (DLHs). The company recorded the following activity and cost data relating to manufacturing overhead for October:

CMA EXAMINATION QUESTIONS Page 61 of 142

MANAGEMENT ADVISORY SERVICES STANDARD COSTS AND VARIANCE ANALYSIS

Activity: Number of units completed 31,200 UnitsStandard direct labor-hours per unit 1.6 DLHsDenominator activity 54,000 DLHsCost:Actual fixed overhead costs incurred $51,300Volume variance $3,468 UnfavorableFixed portion of the predetermined overhead rate $0.85 per DLH

100.The amount of fixed overhead cost contained in the company's overhead budget for September was: (E)A. $45,900. C. $49,920. B. $54,768. D. $47,703.

101.The amount of fixed manufacturing overhead cost applied to work in process during September was: (E)A. $47,832. C. $42,432. B. $26,520. D. $43,605.

102.The fixed overhead budget variance for September was: (E)A. $2,700 favorable. C. $5,400 favorable.B. $2,700 unfavorable. D. $5,400 unfavorable.

Fixed Overhead Spending Variance & Volume Variance69. STA Company’s standard fixed overhead cost is P3 per direct labor hours based on budgeted

fixed costs of P300,000. The standard allows 2 direct labor hours per unit. During 2001, STA produced 55,000 units of product, incurred P315,000 of fixed overhead costs, and recorded 106,000 actual hours of direct labor. What are the fixed overhead variances? (E)Pol Bobadilla A. B. C. D.Fixed OH Spending Variance P15,000 U P33,000 U P15,000 U P33,000 UFixed OH Volume Variance P30,000 F P30,000 F P18,000 F P18,000 F

Questions 63 & 64 are based on the following information. Pol BobadillaBaltimore, Inc. analyzes manufacturing overhead in the production of its only one product, Blu. The following set of information applies to the month of May, 2003:

Budgeted ActualUnits produced 40,000 38,000Variable manufacturing overhead P4/DLH P16,400

Fixed manufacturing overhead P20/DLH P88,000Direct labor hours 6 minutes/unit 4,200 hours

63. What is the fixed overhead spending variance? (E)A. P4,000 Favorable C. P8,000 UnfavorableB. P8,000 Favorable D. P4,000 Unfavorable

64. What is the volume variance?(E)A. P4,000 Favorable C. P8,000 FavorableB. P4,000 Unfavorable D. P8,000 Unfavorable

Questions 88 & 89 are based on the following information. G & N 9eJessep Corporation has a standard cost system in which manufacturing overhead is applied to units of product on the basis of direct labor hours. The company has provided the following data concerning its fixed manufacturing overhead costs in March:

Denominator hours ...................... 15,000 hoursActual hours worked .................... 14,000 hoursStandard hours allowed for the output .. 12,000 hoursFlexible budget fixed overhead cost .... $45,000Actual fixed overhead costs ............ $48,000

88. The fixed overhead budget variance is: (E)a. $1,000 U. c. $2,000 U.b. $3,000 U. d. $2,000 F.

89. The fixed overhead volume variance is (E)a. $3,000 U. c. $9,000 U.b. $3,000 F. d. $6,000 U.

Questions 108-109 are based on the following information: G & N 10eAn outdoor barbecue grill manufacturer has a standard costing system based on machine-hours (MHs) as the measure of activity. Data from the company's flexible budget for manufacturing overhead are given below:

CMA EXAMINATION QUESTIONS Page 62 of 142

MANAGEMENT ADVISORY SERVICES STANDARD COSTS AND VARIANCE ANALYSIS

Denominator level of activity 6,900 MHsFixed overhead cost $95,565

The following data pertain to operations for the most recent period:Actual hours 7,300 MHsStandard hours allowed for the actual output 7,084 MHsActual total fixed overhead cost $94,565

108.What was the fixed overhead budget variance for the period to the nearest dollar? (E)A. $2,798 F C. $3,548 UB. $6,540 U D. $1,000 F

109.What was the fixed overhead volume variance for the period to the nearest dollar? (E)A. $2,548 F C. $5,540 FB. $2,992 U D. $2,456 F

Questions 90 & 91 are based on the following information. G & N 9eAn outdoor barbecue grill manufacturer has a standard costing system based on direct labor-hours (DLHs) as the measure of activity. Data from the company's flexible budget for manufacturing overhead are given below:

Denominator level of activity 3,300 DLHsFixed overhead cost $26,895

The following data pertain to operations for the most recent period:Actual hours 3,400 DLHsStandard hours allowed for the actual output 3,420 DLHsActual total fixed overhead cost $28,295

90. What was the fixed overhead budget variance for the period to the nearest dollar? (E)a. $166 U c. $585 Fb. $422 F d. $1,400 U

91. What was the fixed overhead volume variance for the period to the nearest dollar? (E)a. $978 F c. $163 Fb. $993 F d. $815 F

Questions 92 & 93 are based on the following information. G & N 9eAn outdoor barbecue grill manufacturer has a standard costing system based on machine-hours (MHs) as the measure of activity. Data from the company's flexible budget for manufacturing overhead are given below:

Denominator level of activity 4,600 MHs

Fixed overhead cost $50,140The following data pertain to operations for the most recent period:

Actual hours 5,000 MHsStandard hours allowed for the actual output 4,743 MHsActual total fixed overhead cost $48,690

92. What was the fixed overhead budget variance for the period to the nearest dollar? (E)a. $5,810 U c. $1,450 Fb. $2,503 F d. $3,009 U

93. What was the fixed overhead volume variance for the period to the nearest dollar? (E)a. $2,801 U c. $1,468 Fb. $1,559 F d. $4,360 F

Questions xx thru xx are based on the following information. H & MSacto Co.’s standard fixed overhead cost is $3 per direct labor hour based on budgeted fixed costs of $300,000. The standard allows 2 direct labor hours per unit. During 1995,Sacto produced 55,000 units of product, incurred $315,000 of fixed overhead costs, and recorded 106,000 actual hours of direct labor.

156. What is Sacto’s fixed overhead spending variance for 1995? (E)a. $30,000 (F) d. $15,000 (U)b. $12,000 (F) e. $15,000 (F)c. $18,000 (U)

157. What is the standard activity level on which Sacto based its fixed overhead rate? (E)a. 110,000 direct labor hours d. 50,000 direct labor hoursb. 105,000 direct labor hours e. 106,000 direct labor hoursc. 100,000 direct labor hours

158. What is Sacto’s fixed overhead volume variance for 1995? (E)a. $30,000 (F) d. $30,000 (U)b. $12,000 (F) e. $15,000 (F)c. $18,000 (U)

Budgeted Fixed Overhead159. If actual fixed manufacturing overhead was $108,000 and there was a $2,600 unfavorable

spending variance and a $2,000 unfavorable volume variance, budgeted fixed manufacturing overhead must have been (E)a. $112,600 c. $106,000b. $110,600 d. $105,400

CMA EXAMINATION QUESTIONS Page 63 of 142

MANAGEMENT ADVISORY SERVICES STANDARD COSTS AND VARIANCE ANALYSIS

Applied Fixed Overhead160. Fixed manufacturing overhead was budgeted at $400,000, and 25,000 direct labor hours were

budgeted. If the fixed overhead volume variance was $16,000 favorable and the fixed overhead spending variance was $12,000 unfavorable, fixed manufacturing overhead applied must be (E)a. $416,000 d. $388,000b. $412,000 e. $384,000.c. $404,000

161. Fixed manufacturing overhead was budgeted at $210,000, and 25,000 direct labor hours were budgeted. If the fixed overhead volume variance was $8,000 unfavorable and the fixed overhead spending variance was $3,000 favorable, fixed manufacturing overhead applied must be (E)a. $218,000 d. $205,000b. $213,000 e. $202,000c. $207,000

ComprehensiveQuestions 80-81 are based on the following information: G & N 10eThe Cloward Company uses a standard cost system in which manufacturing overhead is applied to units of product on the basis of direct labor-hours (DLHs). During August, the company actually used 6,100 direct labor-hours and produced 2,500 units of product. The standard cost card for one unit of product includes the following data for manufacturing overhead:

Variable overhead: 2.5 DLHs @ $3.00 per DLH.Fixed overhead: 2.5 DLHs @ $2.50 per DLH.

For August, the company incurred $16,150 of fixed overhead costs and recorded a $625 favorable volume variance.

80. The denominator level of activity used by Cloward in setting the predetermined overhead rate for August is: (E)A. 6,750 DLHs. C. 6,250 DLHs.B. 6,000 DLHs. D. 6,500 DLHs.

81. The amount of fixed overhead cost contained in the company's budget for August is: (E)A. $16,775. C. $15,525. B. $16,250. D. $15,000.

Questions 82-83 are based on the following information: G & N 10eThe Hawkins Company uses a standard costing system in which manufacturing overhead is applied to units of product on the basis of direct labor-hours (DLHs). During February, the company

actually used 9,200 direct labor-hours and made 2,900 units of finished product. The standard cost card for one unit of product includes the following:

Variable overhead: 3 DLHs @ $4.75 per DLHFixed overhead: 3 DLHs @ $3.00 per DLH

For February, the company incurred $28,450 in fixed overhead costs and recorded a $900 favorable volume variance.

82. The amount of fixed overhead cost contained in the company's budget for February is: A. $27,000. C. $25,200. B. $27,550. D. $29,350.

83. The denominator activity level in direct labor-hours used by Hawkins in setting the predetermined overhead rate for February is: A. 9,300 hours. C. 9,000 hours.B. 8,400 hours. D. 8,700 hours.

Questions 71-73 are based on the following information: G & N 10eAble Control Company, which manufactures electrical switches, uses a standard cost system in which manufacturing overhead costs are applied to units of product on the basis of direct labor-hours (DLHs). The standard overhead costs are shown below:

CMA EXAMINATION QUESTIONS Page 64 of 142

MANAGEMENT ADVISORY SERVICES STANDARD COSTS AND VARIANCE ANALYSIS

Variable overhead (5 DLHs @ $8.00 per DLH) $40Fixed overhead (5 DLHs @ $12.00* per DLH) $60

*Based on 300,000 DLHs per month.

The following information is available for the month of October: Plans had called for the production of 60,000 switches. 56,000 switches were actually produced. 275,000 direct labor-hours were worked at a total cost of $2,550,000. Actual variable overhead costs were $2,340,000. Actual fixed overhead costs were $3,750,000.

71. The fixed overhead spending variance for October was: (E)A. $48,000 Unfavorable. C. $300,000 Favorable.B. $150,000 Unfavorable. D. $390,000 Unfavorable.

72. The variable overhead spending variance for October was: (E)A. $60,000 Favorable. C. $100,000 Unfavorable.B. $110,000 Unfavorable. D. $140,000 Unfavorable.

73. The variable overhead efficiency variance for October was: (E)A. $40,000 Favorable. C. $160,000 Unfavorable.B. $60,000 Favorable. D. $210,000 Unfavorable.

Questions 89-91 are based on the following information: G & N 10eThe Stephens Company uses a standard cost system in which manufacturing overhead is applied to units of product on the basis of direct labor-hours (DLHs). The company has the following flexible budget (in condensed form) for manufacturing overhead:

Direct labor-hours (DLH) 15,000 30,000Total variable overhead costs $17,250 $34,500Total fixed overhead costs $45,000 $45,000

The following data concerning production pertain to last year's operations: The company used a denominator activity of 22,500 direct labor hours to compute the

predetermined overhead rate. The company made 10,275 units of product and worked 21,300 actual hours during the

year. Actual variable overhead was $23,856 and actual fixed overhead was $46,275 for the

year.

The standard direct labor time is 2 hours per unit of product.

89. The fixed overhead cost applied during the year was: (E)A. $20,550. C. $45,000.B. $41,100. D. $46,275.

90. The fixed overhead budget variance was: (E)A. $2,550 U. C. $1,275 U. B. $5,175 U. D. $5,175 F.

91. The volume variance was: (E)A. $7,800 U. C. $1,200 F. B. $1,500 F. D. $3,900 U.

Questions 92-95 are based on the following information: G & N 10eA furniture manufacturer has a standard costing system based on machine-hours (MHs) as the measure of activity. Data from the company's flexible budget for manufacturing overhead are given below:

CMA EXAMINATION QUESTIONS Page 65 of 142

MANAGEMENT ADVISORY SERVICES STANDARD COSTS AND VARIANCE ANALYSIS

Denominator level of activity 1,400 MHsOverhead costs at the denominator activity level:Variable overhead cost $12,040Fixed overhead cost $17,360

The following data pertain to operations for the most recent period:Actual hours 1,300 MHsStandard hours allowed for the actual output 1,440 MHsActual total variable overhead cost $11,440Actual total fixed overhead cost $18,560

92. What is the predetermined overhead rate to the nearest cent? (E)A. $21.00 C. $23.08B. $21.43 D. $22.62

93. How much overhead was applied to products during the period to the nearest dollar? (E)A. $30,000 C. $27,300B. $30,240 D. $29,400

94. What was the fixed overhead budget variance for the period to the nearest dollar? (E)A. $2,440 F C. $1,999 UB. $1,200 U D. $704 F

95. What was the fixed overhead volume variance for the period to the nearest dollar? (E)A. $1,240 U C. $1,736 F B. $496 F D. $516 F

Questions 96-99 are based on the following information: G & N 10eA manufacturer of playground equipment has a standard costing system based on direct labor-hours (DLHs) as the measure of activity. Data from the company's flexible budget for manufacturing overhead are given below:

Denominator level of activity 5,800 DLHsFixed overhead cost $58,870

The following data pertain to operations for the most recent period:Actual hours 6,100 DLHsStandard hours allowed for the actual output 6,018 DLHsActual total fixed overhead cost $58,320

96. What is the predetermined fixed overhead rate to the nearest cent? (E)A. $10.15 C. $9.65 B. $9.56 D. $10.06

97. How much fixed overhead was applied to products during the period to the nearest dollar? (E)A. $58,870 C. $61,915 B. $61,083 D. $58,320

98. What was the fixed overhead budget variance for the period to the nearest dollar? (E)A. $3,595 U C. $2,763 U B. $550 F D. $784 F

99. What was the fixed overhead volume variance for the period to the nearest dollar? (E)A. $2,213 F C. $3,045 FB. $2,113 F D. $832 U

ComprehensiveVariable Overhead Spending Variance & Volume VarianceQuestions 69 & 70 are based on the following information. AICPA, AdaptedRaff Co. has a standard cost system in which manufacturing overhead is applied to units of product on the basis of direct labor hours (DLHs). The following standards are based on 100,000 direct labor hours:

Variable overhead 2 DLHs @ $3 per DLH = $6 per unitFixed overhead 2 DLHs @ $4 per DLH = $8 per unit

The following information pertains to operations during March:Units actually produced 38,000Actual direct labor hours worked 80,000Actual manufacturing overhead incurred: Variable overhead $250,000 Fixed overhead $384,000

CMA EXAMINATION QUESTIONS Page 66 of 142

MANAGEMENT ADVISORY SERVICES STANDARD COSTS AND VARIANCE ANALYSIS

69. For March, the variable overhead spending variance was: (E)a. $6,000 F. c. $12,000 U.b. $10,000 U. d. $22,000 F.

70. For March, the fixed overhead volume variance was: (E)a. $96,000 U. c. $80,000 U.b. $96,000 F. d. $80,000 F.

Questions 81 thru 85 are based on the following information. Pol BobadillaThe following data are the actual results for Roadtrek Co. for October:

Actual output 9,000 casesActual variable overhead P405,000Actual fixed overhead P122,000Actual machine time 40,500 machine hours

Standard cost and budget information for Roadtrek Company follows:Standard variable overhead rate P9.00 per MHStandard quantity of machine hours 4 hours per caseBudgeted fixed overhead P1,440,000 per yearBudgeted output 10,000 cases per month

81. The variable overhead spending variance for the month of October is (E)A. P40,500 U C. P45,000 UB. P81,000 U D. P81,000 F

82. The overhead efficiency variance is (E)A. P4,500 U C. P4,500 FB. P40,500 U D. P40,500 F

83. The amount of fixed overhead controllable variance is (E)A. P2,000 U C. P42,500 UB. P2,000 F D. P42,500 F

84. The amount of fixed overhead volume variance is (E)A. P12,000 F C. P21,000 FB. P12,000 U D. P21,000 U

85. The amount of variable overhead volume variance is (E)A. Zero. C. P12,000 FB. P9,000 U D. P2,250 U

Questions 59-62 are based on the following information: G & N 10eThe Phelps Company uses a flexible budget to plan and control manufacturing overhead costs. Overhead costs are applied to products on the basis of direct labor-hours. The standard cost card shows that 5 direct labor-hours are required per unit of product. Phelps Company had the following budgeted and actual data for March:

Actual BudgetedUnits produced 22,000 20,000Direct labor-hours 105,000 100,000*Variable overhead costs $91,000 $80,000Fixed overhead costs $52,000 $50,000

*Represents the denominator activity for the month.

59. The variable overhead spending variance for March is: (E)A. $ 7,000 unfavorable. C. $13,000 unfavorable.B. $ 9,000 unfavorable. D. $11,000 unfavorable.

60. The variable overhead efficiency variance for March is: (E)A. $8,000 unfavorable. C. $8,000 unfavorable.B. $4,000 favorable. D. $4,000 unfavorable.

61. The fixed overhead budget variance for March is: (E)A. $2,000 favorable. C. $2,000 unfavorable.B. $ 500 favorable. D. $2,500 favorable.

62. The fixed overhead volume variance for March is:(E) A. $1,000 favorable. C. $2,500 unfavorable.B. $5,000 favorable. D. $5,000 unfavorable.

Questions 63-66 are based on the following information: G & N 10eDerf Company uses a standard cost system in which it applies manufacturing overhead on the basis of direct labor-hours. Two direct labor-hours are required for each unit produced. The denominator activity was set at 9,000 units. Manufacturing overhead was budgeted at $135,000 for the period; 20 percent of this cost was fixed. The 17,200 hours worked during the period resulted in production of 8,500 units. Variable manufacturing overhead cost incurred was $108,500 and fixed manufacturing overhead cost was $28,000.

63. The variable overhead spending variance for the period was: (E)

CMA EXAMINATION QUESTIONS Page 67 of 142

MANAGEMENT ADVISORY SERVICES STANDARD COSTS AND VARIANCE ANALYSIS

A. $5,300 unfavorable. C. $6,300 unfavorable.B. $1,200 unfavorable. D. $6,500 unfavorable.

64. The variable overhead efficiency variance for the period was: (E)A. $5,300 unfavorable. C. $1,500 unfavorable.B. $1,200 unfavorable. D. $6,500 unfavorable.

65. The fixed overhead budget variance for the period was: (E)A. $6,300 unfavorable. C. $1,500 unfavorable.B. $2,500 unfavorable. D. $1,000 unfavorable.

66. The fixed overhead volume variance for the period was: (E)A. $750 unfavorable. C. $1,500 unfavorable.B. $2,500 unfavorable. D. $1,000 unfavorable.

Questions 67-70 are based on the following information: G & N 10eThe Dodge Company makes and sells a single product and uses a standard cost system in which manufacturing overhead costs are applied to units of product on the basis of direct labor-hours. The standard cost card shows that 5 direct labor-hours are required per unit of product. The Dodge Company had the following budgeted and actual data for the year:

Actual BudgetedUnits produced 33,000 30,000Direct labor-hours 157,500 150,000*Variable overhead costs $136,500 $120,000Fixed overhead costs $ 78,000 $ 75,000

*Represents the denominator activity.

67. The variable overhead spending variance was: (E)A. $4,500 U. C. $13,500 U.B. $10,500 U. D. $16,500 U.

68. The variable overhead efficiency variance was: (E)A. $6,000 U. C. $12,000 U. B. $6,000 F. D. $12,000 F.

69. The fixed overhead budget variance was: (E)A. $750 F. C. $7,500 F.

B. $3,750 F. D. $3,000 U.

70. The fixed overhead volume variance for was: (E)A. $7,500 F. C. $3,750 U.B. $1,500 F. D. $7,500 U.

Questions 61 thru 64 are based on the following information. HorngrenRoberts Corporation manufactured 100,000 buckets during February. The overhead cost-allocation base is $5.00 per machine-hour. The following variable overhead data pertain to February.

Actual BudgetedProduction 100,000 units 100,000 unitsMachine-hours 9,800 hours 10,000 hoursVariable overhead cost per machine-hour $5.25 $5.00

162. What is the actual variable overhead cost? (E)a. $49,000 c. $51,450b. $50,000 d. none of the above

163. What is the flexible-budget amount? (E)a. $49,000 c. $51,450b. $50,000 d. none of the above

164. What is the variable overhead spending variance?a. $1,000 favorable c. $2,450 unfavorableb. $1,450 unfavorable d. none of the above

165. What is the variable overhead efficiency variance? (E)a. $1,000 favorable c. $2,450 unfavorableb. $1,450 unfavorable d. none of the above

Questions 65 thru 68 are based on the following information. HorngrenRoberson Corporation manufactured 30,000 ice chests during September. The overhead cost-allocation base is $11.25 per machine-hour. The following variable overhead data pertain to September.

Actual BudgetedProduction 30,000 units 24,000 unitsMachine-hours 15,000 hours 10,800 hoursVariable overhead cost per machine-hour: $11.00 $11.25

CMA EXAMINATION QUESTIONS Page 68 of 142

MANAGEMENT ADVISORY SERVICES STANDARD COSTS AND VARIANCE ANALYSIS

166. What is the actual variable overhead cost? (E)a. $121,500 c. $165,000b. $151,875 d. $168,750

167. What is the flexible-budget amount? (E)a. $121,500 c. $165,000b. $151,875 d. $168,750

168. What is the variable overhead spending variance? (E)a. $3,750 favorable c. $13,125 unfavorableb. $16,875 unfavorable d. $30,375 unfavorable

169. What is the variable overhead efficiency variance? (E)a. $3,750 favorable c. $13,125 unfavorableb. $16,875 unfavorable d. $30,375 unfavorable

Questions 44 thru 47 are based on the following information. HorngrenShimon Corporation manufactures industrial-sized water coolers and uses budgeted machine-hours to allocate variable manufacturing overhead. The following information pertains to the company's manufacturing overhead data.

Budgeted output units 15,000 unitsBudgeted machine-hours 5,000 hoursBudgeted variable manufacturing overhead costs for 15,000 units $161,250Actual output units produced 22,000 unitsActual machine-hours used 7,200 hoursActual variable manufacturing overhead costs $242,000

170. What is the budgeted variable overhead cost rate per output unit? (E)a. $10.75 c. $32.25b. $11.00 d. $48.40

171. What is the flexible-budget amount for variable manufacturing overhead? (E)a. $165,000 c. $242,000b. $236,500 d. none of the above

172. What is the flexible-budget variance for variable manufacturing overhead? (E)a. $5,500 favorable c. $4,300 favorableb. $5,500 unfavorable d. none of the above

47. Variable manufacturing overhead costs were __________ for actual output. (E)

a. higher than expected c. lower than expectedb. the same as expected d. unable to be determined

Questions 48 thru 51 are based on the following information. HorngrenWhite Corporation manufactures football jerseys and uses budgeted machine-hours to allocate variable manufacturing overhead. The following information pertains to the company's manufacturing overhead data.

Budgeted ActualOutput units 20,000 18,000Machine-hours 30,000 28,000Variable manufacturing overhead costs $360,000 $342,000

173. What is the budgeted variable overhead cost rate per output unit? (E)a. $12.00 c. $18.00b. $12.21 d. $19.00

174. What is the flexible-budget amount for variable manufacturing overhead? (E)a. $324,000 c. $380,000b. $342, 000 d. none of the above

175. What is the flexible-budget variance for variable manufacturing overhead? (E)a. $18,000 favorable c. zerob. $18,000 unfavorable d. none of the above

51. Variable-manufacturing overhead costs were __________ for actual output. (E)a. higher than expected c. lower than expectedb. the same as expected d. unable to be determined

Questions 100 thru 102 are based on the following information. G & N 9eThe Upton Company employs a standard costing system in which variable overhead is assigned to production on the basis of direct labor hours. Data for the month of February include the following: Variable manufacturing overhead cost incurred: $48,700 Total variable overhead variance: $300 F Standard hours allowed for actual production: 7,000 Actual direct labor hours worked: 6,840

100.The standard variable overhead rate per direct labor hour is: (E)a. $6.91. c. $7.00.b. $6.95. d. $7.12.

101.The variable overhead spending variance is: (E)CMA EXAMINATION QUESTIONS Page 69 of 142

MANAGEMENT ADVISORY SERVICES STANDARD COSTS AND VARIANCE ANALYSIS

a. $820 F. c. $740 F.b. $820 U. d. $740 U.

102.The variable overhead efficiency variance is: (E)a. $430 U. c. $1,120 F.b. $740 F. d. $950 U.

Questions 81 thru 84 are based on the following information. HorngrenJenny’s Corporation manufactured 25,000 grooming kits for horses during March. The fixed-overhead cost-allocation rate is $20.00 per machine-hour. The following fixed overhead data pertain to March.

Actual Static BudgetProduction 25,000 units 24,000 unitsMachine-hours 6,100 hours 6,000 hoursFixed overhead costs for March $123,000 $120,000

176. What is the flexible-budget amount? (E)a. $120,000 c. $123,000b. $122,000 d. $125,000

177. What is the amount of fixed overhead allocated to production? (E)a. $120,000 c. $123,000b. $122,000 d. $125,000

178. What is the fixed overhead spending variance? (E)a. $1,000 unfavorable c. $3,000 unfavorableb. $2,000 favorable d. $5,000 favorable

179. What is the fixed overhead production-volume variance? (E)a. $1,000 unfavorable c. $3,000 unfavorableb. $2,000 favorable d. $5,000 favorable

Questions 85 thru 88 are based on the following information. HorngrenMatthew’s Corporation manufactured 10,000 golf bags during March. The fixed overhead cost-allocation rate is $20.00 per machine-hour. The following fixed overhead data pertain to March.

Actual Static BudgetProduction 10,000 units 12,000 unitsMachine-hours 5,100 hours 6,000 hours

Fixed overhead cost for March $122,000 $120,000

180. What is the flexible-budget amount? (E)a. $100,000 c. $120,000b. $102,000 d. $122,000

181. What is the amount of fixed overhead allocated to production? (E)a. $100,000 c. $120,000b. $102,000 d. $122,000

182. What is the fixed overhead production-volume variance? (E)a. $2,000 unfavorable c. $20,000 unfavorableb. $18,000 favorable d. $22,000 unfavorable

183. Fixed overhead is (E)a. overallocated by $2,000. c. overallocated by $22,000.b. underallocated by $2,000. d. underallocated by $22,000.

Questions 71 thru 74 are based on the following information. G & N 9eA furniture manufacturer has a standard costing system based on machine-hours (MHs) as the measure of activity. Data from the company's flexible budget for manufacturing overhead are given below:

Denominator level of activity 3,300 MHsOverhead costs at the denominator activity level: Variable overhead cost $31,845 Fixed overhead cost $40,425

The following data pertain to operations for the most recent period:Actual hours 3,400 MHsStandard hours allowed for the actual output 3,078 MHsActual total variable overhead cost $32,980Actual total fixed overhead cost $38,975

71. What is the predetermined overhead rate to the nearest cent? (E)a. $21.90 c. $21.16b. $21.80 d. $21.26

72. How much overhead was applied to products during the period to the nearest dollar? (E)a. $74,460 c. $67,408b. $72,270 d. $71,955

73. What was the fixed overhead budget variance for the period to the nearest dollar? (E)CMA EXAMINATION QUESTIONS Page 70 of 142

MANAGEMENT ADVISORY SERVICES STANDARD COSTS AND VARIANCE ANALYSIS

a. $2,675 U c. $3,691 Fb. $1,450 F d. $1,270 F

74. What was the fixed overhead volume variance for the period to the nearest dollar? (E)a. $2,720 U c. $2,811 Ub. $1,225 F d. $3,945 U

Questions 75 thru 78 are based on the following information. G & N 9eA manufacturer of playground equipment has a standard costing system based on machine-hours (MHs) as the measure of activity. Data from the company's flexible budget for manufacturing overhead are given below:

Denominator level of activity 3,000 MHsFixed overhead cost $40,650

The following data pertain to operations for the most recent period:Actual hours 3,400 MHsStandard hours allowed for the actual output 3,172 MHsActual total fixed overhead cost $41,600

75. What is the predetermined fixed overhead rate to the nearest cent? (E)a. $12.24 c. $13.87b. $13.55 d. $11.96

76. How much fixed overhead was applied to products during the period to the nearest dollar? (E)a. $40,650 c. $41,600b. $42,981 d. $46,070

77. What was the fixed overhead budget variance for the period to the nearest dollar? (E)a. $4,470 U c. $2,790 Fb. $950 U d. $1,381 U

78. What was the fixed overhead volume variance for the period to the nearest dollar? (E)a. $2,256 F c. $3,089 Ub. $2,331 F d. $5,420 F

Questions 84-88 are based on the following information: G & N 10eThe Steff Company has the following flexible budget (in condensed form) for manufacturing overhead:

Direct labor-hours (DLH) 10,000 20,000Total variable overhead costs $11,500 $23,000Total fixed overhead costs $30,000 $30,000

The following data concerning production pertain to last year's operations: The company used a denominator activity of 15,000 direct labor-hours to compute the

predetermined overhead rate. The company made 6,850 units of product and worked 14,200 actual hours during the

year. Actual variable overhead was $15,904 and actual fixed overhead was $30,850 for the

year. The standard direct labor time is two hours per unit of product.

84. The variable element of the predetermined overhead rate was (per DLH): (E)A. $4.15. C. $2.00.B. $3.00. D. $1.15.

85. The fixed element of the predetermined overhead rate was (per DLH): (E)A. $4.15. C. $2.00.B. $3.00. D. $1.15.

86. The fixed overhead cost applied to work in process was: (E)A. $27,400. C. $30,850. B. $30,000. D. $13,700.

87. The fixed overhead budget variance was: (E)A. $3,450 unfavorable. C. $850 unfavorable. B. $3,450 favorable. D. $1,200 favorable.

88. The volume variance was: (E)A. $2,000 favorable. C. $1,000 favorable.B. $2,600 unfavorable. D. $800 favorable.

Questions 42-47 are based on the following information: G & N 10eFranklin Glass Works uses a standard cost system in which manufacturing overhead is applied to units of product on the basis of direct labor-hours. Each unit requires two standard hours of labor for completion. The denominator activity for the year was based on budgeted production of 200,000 units. Total overhead was budgeted at $900,000 for the year, and the fixed overhead rate was $3.00 per unit. The actual data pertaining to the manufacturing overhead for the year are presented below:

CMA EXAMINATION QUESTIONS Page 71 of 142

MANAGEMENT ADVISORY SERVICES STANDARD COSTS AND VARIANCE ANALYSIS

Actual production 198,000 UnitsActual direct labor-hours 440,000 direct labor-hoursActual variable overhead $352,000Actual fixed overhead $575,000

42. The standard hours allowed for actual production for the year total: (E)A. 247,500. C. 400,000.B. 396,000. D. 495,000.

43. Franklin's variable overhead efficiency variance for the year is: (M)A. $33,000 unfavorable. C. $35,200 unfavorable.B. $35,200 favorable. D. $33,000 favorable.

44. Franklin's variable overhead spending variance for the year is: (M)A. $20,000 unfavorable. C. $22,000 unfavorable.B. $22,000 favorable. D. $20,000 favorable.

45. Franklin's fixed overhead budget variance for the year is: (M)A. $19,000 favorable. C. $25,000 unfavorable.B. $25,000 favorable. D. $19,000 unfavorable.

46. The fixed overhead applied to Franklin's production for the year is: (M)A. $484,200. C. $594,000.B. $575,000. D. $600,000.

47. Franklin's fixed overhead volume variance for the year is: (M)A. $6,000 unfavorable. C. $25,000 favorable.B. $19,000 favorable. D. $55,000 unfavorable.

Questions 48-53 are based on the following information: G & N 10eA manufacturing company has a standard costing system based on direct labor-hours (DLHs) as the measure of activity. Data from the company's flexible budget for manufacturing overhead are given below:

Denominator level of activity 3,700 DLHsOverhead costs at the denominator activity level:Variable overhead cost $28,490Fixed overhead cost $47,545

The following data pertain to operations for the most recent period:Actual hours 3,900 DLHsStandard hours allowed for the actual output 3,850 DLHsActual total variable overhead cost $29,445Actual total fixed overhead cost $47,995

48. What is the predetermined overhead rate to the nearest cent? (E)A. $20.93 C. $19.50B. $19.86 D. $20.55

49. How much overhead was applied to products during the period to the nearest dollar? (E)A. $79,118 C. $77,440B. $76,035 D. $80,145

50. What was the variable overhead spending variance for the period to the nearest dollar? (E)A. $585 U C. $955 UB. $585 F D. $955 F

51. What was the variable overhead efficiency variance for the period to the nearest dollar? (E)A. $578 U C. $378 UB. $385 U D. $955 U

52. What was the fixed overhead budget variance for the period to the nearest dollar? (E)A. $615 F C. $1,478 UB. $2,120 U D. $450 U

53. What was the fixed overhead volume variance for the period to the nearest dollar? (E)A. $1,870 F C. $643 UB. $1,928 F D. $2,570 F

Questions 54-58 are based on the following information: G & N 10eThe Chase Company uses a standard cost system in which manufacturing overhead costs are applied to products on a basis of machine-hours. For November, the company's flexible budget for

CMA EXAMINATION QUESTIONS Page 72 of 142

MANAGEMENT ADVISORY SERVICES STANDARD COSTS AND VARIANCE ANALYSIS

manufacturing overhead showed the following total budgeted costs at the denominator activity level of 40,000 machine-hours:

Variable overhead costs (total):Maintenance $36,000Utilities $20,000Fixed overhead costs (total):Supervision $24,000Depreciation $16,00

During November 42,000 machine-hours were used to complete 13,200 units of product with the following actual overhead costs:

Variable overhead costs (total):Maintenance $43,420Utilities $32,510Fixed overhead costs (total):Supervision $26,970Depreciation $16,000

The standard time allowed to complete one unit of product is 3.6 machine-hours.

54. The total predetermined overhead rate per machine-hour for November was: (E)A. $1.75. C. $2.97.B. $2.40. D. $1.40.

55. The total amount of overhead cost applied to Work in Process during November was: (E)A. $ 47,520. C. $106,528.B. $ 66,528. D. $114,048.

56. The variable overhead efficiency variance for utilities cost for November was: (?)A. $2,760 favorable. C. $3,760 favorable.B. $3,760 unfavorable. D. $1,000 favorable.

57. The variable overhead spending variance for maintenance cost for November was: (?)A. $7,420 unfavorable. C. $9,820 unfavorable.B. $2,400 favorable. D. $5,620 unfavorable.

58. The fixed overhead budget variance for November was: (E)A. $ 2,970 unfavorable. C. $ 7,520 favorable.B. $ 4,550 favorable. D. $22,900 unfavorable.

ComprehensiveQuestions xx thru xx are based on the following information. H & MSmith Corporation uses a standard cost system. Information for the month of April is as follows:

Actual manufacturing overhead costs (13,000 is fixed) $40,000Direct labor:Actual hours worked 12,000 hrs.Standard hours allowed 10,000 hrs.Average actual labor cost per hour $9Standard cost data at 12,000 direct labor hours was:Variable factory overhead $24,000Fixed factory overhead 12,000 Total factory overhead $36,000

The factory overhead rate is based on a normal volume of 12,000 direct labor hours.

184. What is the variable overhead efficiency variance for Smith? (E)a. $1,000 (U) d. $10,000 (U)b. $4,000 (U) e. $3,000 (U)c. $2,000 (U)

185. What is the fixed overhead spending variance for Smith? (E)a. $1,000 (U) d. $10,000 (U)b. $4,000 (U) e. $3,000 (U)c. $2,000 (U)

Questions xx thru xx are based on the following information. H & MRitz Production Company has the following information:Standard factory overhead rates are based on a normal monthly volume of 10,000 units (1 standard direct hour per unit).

Standard factory overhead rates per direct labor hour are: Fixed $3.00 Variable 5.00 $8.00Units actually produced in current month 9,000 unitsActual factory overhead costs incurred (includes $35,000 fixed) $78,000Actual direct labor hours 9,000 hours

186. What is the fixed overhead spending variance for Ritz? (E)a. $3,000 (U) d. $5,000 (F)b. $0 e. $5,000 (U)c. $2,000 (F)

187. What is the fixed overhead volume variance for Ritz? (E)a. $3,000(U) d. $5,000(F)b. $0 e. $5,000(U)c. $2,000(F)

CMA EXAMINATION QUESTIONS Page 73 of 142

MANAGEMENT ADVISORY SERVICES STANDARD COSTS AND VARIANCE ANALYSIS

188. What is the variable overhead spending variance for Ritz? (E)a. $43,000 (U) d. $5,000 (F)b. $0 e. $5,000 (U)c. $2,000 (F)

Questions 92 through 95 are based on the following information. BarfieldStandard Company has developed standard overhead costs based on a capacity of 180,000 machine hours as follows:

Standard costs per unit:Variable portion 2 hours @ $3 = $ 6Fixed portion 2 hours @ $5 = 10 $16

During April, 85,000 units were scheduled for production, but only 80,000 units were actually produced. The following data relate to April: Actual machine hours used were 165,000. Actual overhead incurred totaled $1,378,000 ($518,000 variable plus $860,000 fixed). All inventories are carried at standard cost.

92. The variable overhead spending variance for April was (E)a. $15,000 U. c. $38,000 F.b. $23,000 U. d. $38,000 U.

93. The variable overhead efficiency variance for April was (E)a. $15,000 U. c. $38,000 F.b. $23,000 U. d. $38,000 U.

94. The fixed overhead spending variance for April was (E)a. $40,000 U. c. $60,000 F.b. $40,000 F. d. $60,000 U.

95. The fixed overhead volume variance for April was (E)a. $60,000 U. c. $100,000 F.b. $60,000 F. d. $100,000 U.

Questions 47 thru 52 are based on the following information. G & N 9eA manufacturing company has a standard costing system based on machine-hours (MHs) as the measure of activity. Data from the company's flexible budget for manufacturing overhead are given below:

Denominator level of activity 6,100 MHsOverhead costs at the denominator activity level:

Variable overhead cost $35,075 Fixed overhead cost $77,775

The following data pertain to operations for the most recent period:Actual hours 6,300 MHsStandard hours allowed for the actual output 5,994 MHsActual total variable overhead cost $36,540Actual total fixed overhead cost $76,875

47. What is the predetermined overhead rate to the nearest cent? (E)a. $17.91 c. $18.00b. $18.59 d. $18.50

48. How much overhead was applied to products during the period to the nearest dollar? (E)a. $112,850 c. $116,550b. $113,415 d. $110,889

49. What was the variable overhead spending variance for the period to the nearest dollar? (E)a. $315 U c. $1,465 Ub. $1,465 F d. $315 F

50. What was the variable overhead efficiency variance for the period to the nearest dollar? (E)a. $300 F c. $1,760 Ub. $1,465 U d. $1,775 U

51. What was the fixed overhead budget variance for the period to the nearest dollar? (E)a. $900 F c. $3,734 Fb. $452 F d. $3,450 U

52. What was the fixed overhead volume variance for the period to the nearest dollar? (E)a. $1,359 U c. $3,902 Ub. $2,550 F d. $1,352 U

Questions 53 thru 56 are based on the following information. G & N 9eThe Dillon Company makes and sells a single product and uses a flexible budget for overhead to plan and control overhead costs. Overhead costs are applied on the basis of direct labor-hours. The standard cost card shows that 5 direct labor-hours are required per unit. The Dillon Company had the following budgeted and actual data for March:

Actual BudgetedUnits produced 33,900 30,800Direct labor-hours 161,800 154,000Variable overhead costs $140,500 $123,200

CMA EXAMINATION QUESTIONS Page 74 of 142

MANAGEMENT ADVISORY SERVICES STANDARD COSTS AND VARIANCE ANALYSIS

Fixed overhead costs $80,000 $77,000

53. The variable overhead spending variance for March is: (E)a. $4,900 U. c. $14,700 U.b. $11,060 U. d. $17,300 U.

54. The variable overhead efficiency variance for March is: (E)a. $12,400 F. c. $12,400 U.b. $6,160 U. d. $6,160 F.

55. The fixed overhead budget variance for March is: (E)a. $900 F. c. $3,000 U.b. $3,900 F. d. $7,750 F.

56. The fixed overhead volume variance for March is: (E)a. $7,750 F. c. $1,550 Fb. $7,750 U. d. $3,900 U.

Questions 57 thru 60 are based on the following information. G & N 9eThe Ferris Company applies manufacturing overhead costs to products on the basis of direct labor hours. The standard cost card shows that 3 direct labor hours are required per unit of product. For August, the company budgeted to work 90,000 direct labor hours and to incur the following total manufacturing overhead costs:

Total variable overhead costs $ 99,000Total fixed overhead costs $118,800

During August, the company completed 28,000 units of product, worked 86,000 direct labor hours, and incurred the following total manufacturing overhead costs:

Total variable overhead costs $ 98,900Total fixed overhead costs $115,300

The denominator activity in the predetermined overhead rate is 90,000 direct labor hours.

57. For August, the variable overhead spending variance is: (E)a. $4,300 F. c. $6,500 F.b. $4,300 U. d. $6,500 U.

58. For August, the variable overhead efficiency variance is: (E)a. $1,800 F. c. $2,200 U.b. $0. d. $2,200 F.

59. For August, the fixed overhead budget variance is: (E)a. $3,500 F. c. $3,200 F.

b. $3,500 U. d. $3,200 U.

60. For August, the fixed overhead volume variance is: (E)a. $4,300 U. c. $4,980 F.b. $7,920 U. d. $4,980 U.

Questions 61 thru 64 are based on the following information. AICPA, AdaptedKing Company estimated that it would operate its manufacturing facilities at 800,000 direct labor hours for the year and this served as the denominator activity in the predetermined overhead rate. The total budgeted manufacturing overhead for the year was $2,000,000, of which $1,600,000 was variable and $400,000 was fixed. The standard variable overhead rate was $2 per direct labor hour. The standard direct labor time was 3 direct labor hours per unit. The actual results for the year are presented below:

Actual finished units 250,000Actual direct labor hours 764,000Actual variable overhead $1,610,000Actual fixed overhead $ 392,000

61. The variable overhead spending variance for the year is: (E)a. $2,000 F. c. $82,000 U.b. $10,000 U. d. $110,000 U.

62. The variable overhead efficiency variance for the year is: (E)a. $28,000 U. c. $100,000 F.b. $100,000 U. d. $28,000 F.

63. The fixed overhead spending variance for the year is: (E)a. $8,000 F. c. $17,000 U.b. $10,000 U. d. $74,000 F.

64. The fixed overhead volume variance for the year is: (E)a. $7,000 U. c. $41,667 U.b. $25,000 U. d. $18,000 F.

Questions 111 through 115 are based on the following information. CMA 1292 3-15 to 19Nanjones Company manufactures a line of products distributed nationally through wholesalers. Presented below are planned manufacturing data for 1992 and actual data for November 1992. The company applies overhead based on planned machine hours using a predetermined annual rate.

1992 Planning DataAnnual November

CMA EXAMINATION QUESTIONS Page 75 of 142

MANAGEMENT ADVISORY SERVICES STANDARD COSTS AND VARIANCE ANALYSIS

Fixed manufacturing overhead $1,200,000 $100,000Variable manufacturing overhead 2,400,000 220,000Direct labor hours 48,000 4,000Machine hours 240,000 22,000

November 1992 DataActual Planned*

Direct labor hours 4,200 4,000Machine hours 21,600 21,000Fixed manufacturing overhead $101,200Variable manufacturing overhead $214,000

* plan based on output

189. The predetermined overhead application rate for Nanjones Company for 1992 is (D)a. $5.00 c. $10.00b. $25.00 d. $15.00

190. The total amount of overhead applied to production for November 1992 was (D)a. $316,200. c. $320,000.b. $315,000. d. $300,000.

191. The amount of over- or under-applied variable manufacturing overhead for November was (D)a. $6,000 over-applied. c. $20,000 over-applied.b. $4,000 under-applied. d. $6,000 under-applied.

192. The variable overhead spending variance for November 1992 was (D)a. $2,000 favorable. c. $14,000 unfavorable.b. $6,000 favorable. d. $6,000 unfavorable.

193. The fixed overhead volume variance for November 1992 was (D)a. $1,200 unfavorable. c. $10,000 favorable.b. $5,000 unfavorable. d. $5,000 favorable.

Questions 116 through 121 are based on the following information. CMA 1290 3-5 to 10Franklin Glass Works' production budget for the year ended November 30 was based on 200,000 units. Each unit requires two standard hours of labor for completion. Total overhead was budgeted at $900,000 for the year, and the fixed overhead rate was estimated to be $3.00 per unit. Both fixed and variable overhead are assigned to the product on the basis of direct labor hours. The actual data for the year ended November 30 are presented as follows.

Actual production in units 198,000Actual direct labor hours 440,000Actual variable overhead $352,000Actual fixed overhead $575,000

194. The standard hours allowed for actual production for the year ended November 30 total (M)A. 247,500. C. 400,000.B. 396,000. D. 495,000.

195. Franklin's variable overhead efficiency variance for the year is (M)A. $33,000 unfavorable. C. $66,000 unfavorable.B. $35,520 favorable. D. $33,000 favorable.

CMA EXAMINATION QUESTIONS Page 76 of 142

MANAGEMENT ADVISORY SERVICES STANDARD COSTS AND VARIANCE ANALYSIS

196. Franklin's variable overhead spending variance for the year is (M)A. $20,000 unfavorable. C. $22,000 unfavorable.B. $19,800 favorable. D. $20,000 favorable.

197. Franklin's fixed overhead spending variance for the year is (E)A. $19,000 favorable. C. $5,750 favorable.B. $25,000 favorable. D. $25,000 unfavorable.

198. The fixed overhead applied to Franklin's production for the year is (M)A. $484,200. C. $594,000.B. $575,000. D. $600,000.

199. Franklin's fixed overhead volume variance for the year is (M)A. $6,000 unfavorable. C. $25,000 favorable.B. $19,000 favorable. D. $55,000 unfavorable.

DISPOSITION OF VARIANCESAllocation MethodUnderapplied Charged to Cost of Goods Sold200. Kell Company has the following selected debit balance accounts at the end of the current year:

Work-in-Process, $100,000; Finished Goods Inventory, $50,000; Cost of Goods Sold, $150,000; and Factory Overhead, $24,000. If over- or underapplied factory overhead is disposed of by the allocation method, the amount charged to Cost of Goods Sold will be (E)a. $6,000 d. $24,000b. $12,000 e. $4,000c. $8,000 H & M

Adjusted Cost of Goods Sold201. Account balances are as follows:

Manufacturing overhead $120,000 underappliedWork in process inventory 50,000Finished goods inventory 150,000Cost of goods sold 400,000

If underapplied or overapplied overhead is material and is allocated to Work in Process Inventory, Finished Goods Inventory, and Cost of Goods Sold (based on ending account balances), Cost of Goods Sold after adjustment would have a balance of (E)a. $720,000 c. $520,000b. $680,000 d. $480,000 H & M

202. Worley Company has underapplied overhead of $45,000 for the year ended December 31, 2002. Before disposition of the underapplied overhead, selected December 31, 2002 balances from Worley’s accounting recorded are as follows:

Sales $1,200,000Cost of goods sold 720,000Inventories:

Direct materials 36,000Work in process 54,000Finished goods 90,000

Under Worley’s cost accounting system, over- or underapplied overhead is allocated to appropriate inventories and cost of goods sold based on year-end balances. In its 2002 income statement, Worley should report cost of goods sold of (M)A. $682,500 C. $756,000B. $684,000 D. $757,500

Charged to Finished Goods, DM & DL Component of FG InventoryQuestions 1 thru 4 are based on the following information. AICPA adaptedJosey Manufacturing Corporation uses a standard cost system that records direct materials at actual cost, records materials price variances at the time that direct materials are issued to work in process, and prorates all variances at year end. Variances associated with direct materials are prorated based on the direct materials balances in the appropriate accounts, and variances associated with direct labor and factory overhead are prorated based on the direct labor balances in the appropriate accounts.The following information is available for Josey for the year ended December 31:

Finished goods inventory at December 31: Direct materials $ 87,000 Direct labor 130,500 Applied factory overhead 104,400Direct materials inventory at December 31 65,000Cost of goods sold for the year ended December 31: Direct materials 348,000 Direct labor 739,500 Applied factory overhead 591,600Direct materials price variance (unfavorable) 12,500Direct materials usage variance (favorable) 15,000Direct labor rate variance (unfavorable) 20,000Direct labor efficiency variance (favorable) 5,000Factory overhead incurred 690,000

There were no beginning inventories and no ending work in process inventory. Factory overhead is applied at 80% of standard direct labor cost.

CMA EXAMINATION QUESTIONS Page 77 of 142

MANAGEMENT ADVISORY SERVICES STANDARD COSTS AND VARIANCE ANALYSIS

*. The amount of direct materials price variance to be prorated to finished goods inventory at December 31 is a:A. $1,740 debit C. $2,610 creditB. $2,500 debit ($2,175) D. $3,000 credit

*. The total amount of direct materials in finished goods inventory at December 31, after all materials variances have been prorated, is:A. $86,500 ($85,565) C. $88,000B. $87,500 D. $86,000

*. The total amount of direct labor in finished goods inventory at December 31, after all variances have been prorated, is: (D)A. $126,750 C. $132,750B. $134,250 D. $133,750

*. The total cost of goods sold for the year ended December 31, after all variances have been prorated, is:A. $1,693,850 C. $1,675,450B. $1,684,750 ($1,108,200) D. $1,683,270

GROSS PROFIT. Alma Company budgeted that factory overhead for 2003 and 2004 would be P60,000 for each

year. The predicted and actual activity for 2003 and 2004 were 30,000 and 20,000 direct labor hours, respectively.

2003 2004Sales in units 25,000 25,000Selling price per unit P10 P10Direct materials and direct labor cost per unit P5 P5

The company assumes that the long-run production level is 20,000 direct labor hours per year. The actual factory overhead cost for the end of 2003 and 2004 was P60,000. Assume that it takes one direct labor hour to make one finished unit.When the annual estimated factory overhead rate is used, the gross profits for 2003 and 2004, respectively, are (D)A. P75,000 and P75,000 C. P75,000 and P55,000B. P125,000 and P125,000 D. P75,000 and P50,000 Pol Bobadilla

JOURNAL ENTRIESMaterial & Labor Components of InventoriesQuestions 1 & 2 are based on the following information. CMA adapted

Sam Company adopted a standard cost system several years ago. The standard costs for the prime costs of its single product are as follows:

Material (8 kilograms x $5.00/kg.) $40.00Labor (6 hours x $8.20/hr.) $49.20

The operating data in the following column were taken from the records for November:In-process beginning inventory noneIn-process ending inventory (75% complete as to labor;

material is issued at the beginning of processing) 800 unitsUnits completed 5,600 unitsBudgeted output 6,000 unitsPurchases of materials 50,000 kilogramsTotal actual labor costs $300,760Actual hours of labor 36,500 hoursMaterial usage variance $1,500 unfavorableTotal material variance $750 unfavorable

203. The total amount of material and labor cost transferred to the finished goods account for November is:A. $499,520 C. $550,010B. $535,200 D. $561,040

204. The total amount of material and labor cost in the ending balance of work in process inventory at the end of November is:A. 0 C. $61,520B. $9,840 D. $71,360

Material & Labor VariancesQuestions 61-65 are based on the following information: G & N 10eThe Odle Company makes and sells a single product called a Kitt. Odle employs a standard costing system. Each Kitt has a standard cost of 5 pounds of material at $12 per pound and 0.9 direct labor hours at $15 per hour. There were no inventories of any kind of June 1. During June, the following events occurred:

Purchased 17,000 pounds of material at a total cost of $190,000. Used 15,000 pounds of material to produce 2,400 Kitts. Used 1,900 hours of direct labor time at a total cost of $38,000.

61. To record the incurrence of direct labor cost and its use in production, the general ledger would include what kind of entry to the Labor Rate Variance account?

CMA EXAMINATION QUESTIONS Page 78 of 142

MANAGEMENT ADVISORY SERVICES STANDARD COSTS AND VARIANCE ANALYSIS

A. $ 9,500 credit. C. $15,200 debit. B. $ 9,500 debit. D. $ 2,000 debit.

62. To record the incurrence of direct labor cost and its use in production, the general ledger would include what kind of entry to the Labor Efficiency Variance account? A. $7,500 credit. C. $5,600 credit. B. $5,600 debit. D. $3,900 credit.

63. Odle Company purchased material on account. The entry to record the purchase of materials will include a: A. credit to Work in Process. C. credit to Accounts Payable. B. debit to Accounts Receivable. D. credit to Raw Materials Inventory.

64. To record the purchase of direct materials, the general ledger would include what kind of entry to the Materials Price Variance Account? A. $14,000 credit. C. $10,000 credit.B. $14,000 debit. D. $10,000 debit.

65. To record the use of direct materials in production, the general ledger would include what kind of entry to the Materials Quantity Variance Account? A. $46,000 debit. C. $60,000 debit.B. $60,000 credit. D. $36,000 debit.

Questions 1 thru 4 are based on the following information. AICPA adaptedKaiser Manufacturing Company uses a standard cost system in accounting for the costs of production of its only product, Product A. The standards for the production of one unit of Product A are as follows:

Direct materials: 10 feet of Item 1 at $.78 per foot and 3 feet of Item 2 at $1 per footDirect labor: 4 hours at $3.60 per hourFactory overhead: applied at 150% of standard direct labor costs

There was no inventory on hand at the end of the year. Materials price variances are isolated at purchase. Following is a summary of costs and related data for the production of Product A during the year: 100,000 feet of Item 1 were purchased at $.75 per foot. 30,000 feet of Item 2 were purchased at $.90 per foot. 8,000 units of Product A were produced that required 78,000 feet of Item 1, 26,000

feet of Item 2, and 31,000 hours of direct labor at $3.50 per hour. 6,000 units of Product A were sold.

205. The total debits to the direct materials account for the purchase of Item 1 should be:A. $75,000 C. $58,500

B. $78,000 D. $60,000

206. The total debits to the work in process account for direct labor should be:A. $111,600 C. $112,000B. $108,500 D. $115,200

207. Before allocation of standard variances, the balance in the materials quantity variance account of Item 2 was:A. $2,000 debit D. $600 debitB. $1,000 credit E. $1,000 debitC. $2,600 debit

208. If all standard variances are prorated to inventories and cost of goods sold, the amount of materials quantity variance for Item 2 to be prorated to direct materials inventory would be:A. $500 debit D. $333 creditB. $500 credit E. $333 debitC. 0

Questions 77 thru 80 are based on the following information. G & N 9eThe Dexon Company makes and sells a single product called a Mip and employs a standard costing system. The following standards have been established for one unit of Mip:

Standard Quantity or Hours Standard Cost per MipDirect materials 6 board feet $9.00Direct labor 0.8 hours $9.60

There were no inventories of any kind on August 1. During August, the following events occurred: Purchased 15,000 board feet at the total cost of $24,000. Used 12,000 board feet to produce 2,100 Mips. Used 1,700 hours of direct labor time at a total cost of $20,060.

77. To record the purchase of direct materials, the general ledger would include what entry to the Materials Price Variance Account? (M)a. $1,500 credit c. $6,000 creditb. $1,500 debit d. $6,000 debit

78. To record the use of direct materials in production, the general ledger would include what entry to the Materials Quantity Variance account? (M)a. $3,600 debit c. $900 debitb. $3,600 credit. d. $900 credit

79. To record the incurrence of direct labor cost and its use in production, the general ledger would include what entry to the Labor Rate Variance account? (M)

CMA EXAMINATION QUESTIONS Page 79 of 142

MANAGEMENT ADVISORY SERVICES STANDARD COSTS AND VARIANCE ANALYSIS

a. $240 credit c. $340 debitb. $240 debit d. $340 credit

80. To record the incurrence of direct labor costs and its use in production, the general ledger would include what entry to the Labor Efficiency Variance account? (M)a. $480 credit c. $1,200 debitb. $240 debit d. $1,200 credit

Closing of Over (under) Applied Overhead36. The operations of the Kerry Company resulted in underapplied overhead of $5,000. The entry

to close out this balance to Cost of Goods Sold and the effect of the underapplied overhead on Cost of Goods Sold would be: G & N 10e

Entry Effect on Cost of Goods Sold

A. Manufacturing Overhead 5,000 Deduct $5,000 Cost of Goods Sold 5,000

B. Cost of Goods Sold 5,000 Deduct $5,000 Manufacturing Overhead 5,000

C. Cost of Goods Sold 5,000 Add $5,000 Manufacturing Overhead 5,000

D. Manufacturing Overhead 5,000 Add $5,000 Cost of Goods Sold 5,000

COMPREHENSIVETotal Variance29. KNOTTY, Inc. estimated the cost of a project it started in October 19x4 as follows: Direct

materials, P495,000; direct labor, 6,000 hours at P30 per hour; variable overhead, P24 per direct labor hour. By the end of the month, all the required materials have been used at P491,900; labor was 80% complete at 4,650 hours at P30 per hour; and, the variable overhead amounted to P113,700. The total variance for the project as at the end of the month was (M)a. P7,500 unfavorable c. P9,000 favorableb. P8,400 unfavorable d. P9,00 favorable RPCPA 1094

DM and DL VariancesQuestions 127 and 128 are based on the following information. CIA 0595 III-81 & 82A company manufactures a machine component called Omega. The following relates to manufacturing operations in May.

Planned production 2,000 units of Omega

Actual production 2,100 units of OmegaStandard costs per unit of OmegaDirect materials $20 (5 lbs. @ $4)Direct labor $10 (1 hr. @ $10)Actual costs incurredDirect materials purchased and used $44,772 (10,920 lbs. @ $4.10)Direct labor $20,500 (2,000 hr. @ $10.25)

209. The direct materials efficiency variance was (E)A. $1,092 unfavorable. C. $3,680 unfavorable.B. $1,680 unfavorable. D. $3,772 unfavorable.

210. The direct labor flexible budget variance was (E)A. $500 favorable. C. $1,000 favorable.B. $500 unfavorable. D. $0

Questions 129 and 130 are based on the following information. RPCPA 1082The Sta. Anita Company has a budgeted normal monthly capacity of 5,000 labor hours with a standard production of 4,000 units are this capacity. Standard costs are:

Materials - 2 kilos at P1.00Labor - P8.00 per hourFactory overhead at normal capacity

Fixed expenses - P5,000Variable expenses - P1.50 per labor hour

During September, actual factory overhead totaled P11,250 and 4,500 labor hours cost P33,750. Production during the month was 3,500 units using 7,200 kilos of materials at a cost of P1.02 per kilo.

*. The material price variance during September was (M)a. P1,440 unfavorable d. P3,440 unfavorableb. P204 favorable e. None of the abovec. P140 favorable

*. The labor efficiency variance was (E)a. P2,250 unfavorable c. P2,187.50 favorableb. P1,000 unfavorable d. P62.50 favorable

Questions 131 through 133 are based on the following information. CIA 0594 III-72 to 74A company manufactures one product and has a standard cost system. In April the company had the following experience:

Direct Materials Direct LaborCMA EXAMINATION QUESTIONS Page 80 of 142

MANAGEMENT ADVISORY SERVICES STANDARD COSTS AND VARIANCE ANALYSIS

Actual $/unit of input (lbs. & hrs.) $28 $18 Standard price/unit of input $24 $20 Standard inputs allowed per unit of output 10 4 Actual units of input 190,000 78,000 Actual units of output 20,000 20,000

211. The direct materials price variance for April is (M)A. $760,000 favorable. C. $240,000 unfavorable.B. $760,000 unfavorable. D. $156,000 favorable.

212. The direct materials efficiency variance for April is (M)A. $156,000 favorable. C. $240,000 unfavorable.B. $240,000 favorable. D. $760,000 unfavorable.

213. The direct labor rate variance for April is (E)A. $240,000 favorable. C. $156,000 favorable.B. $156,000 unfavorable. D. $40,000 unfavorable.

Questions 32 thru 34 are based on the following information. H & MRinker Company produced 100 units of Product X. The total standard and actual costs for materials and direct labor for the 100 units of Product X are as follows:

Raw materials: Standard Actual Standard: 100 pounds at $2.00 per pound $200 Actual: 110 pounds at $1.90 per pound $209Direct labor: Standard: 200 hours at $10 per hour 2,000 Actual: 184 hours at $11 per hour 2,024

214. What is the material usage variance for Rinker Company?a. $9 (U) d. $11 (U)b. $20 (F) e. $20 (U)c. $11 (F)

215. What is the material price variance for Rinker Company?a. $9 (U) d. $11 (U)b. $20 (F) e. $20 (U)c. $11 (F)

216. What is the labor efficiency variance for Rinker Company?a. $160 (U) d. $160 (F)b. $184 (F) e. $24 (U)c. $184 (U)

Questions 134 through 137 are based on the following information. CMA 1291 3-1 to 4Arrow Industries employs a standard cost system in which direct materials inventory is carried at standard cost. Arrow has established the following standards for the prime costs of one unit of product.

Standard Quantity Standard Price Standard CostDirect materials 8 pounds $1.80 per pound $14.40Direct labor . 25 hour 8.00 per hour 2.00 $16.40

CMA EXAMINATION QUESTIONS Page 81 of 142

MANAGEMENT ADVISORY SERVICES STANDARD COSTS AND VARIANCE ANALYSIS

During November, Arrow purchased 160,000 pounds of direct materials at a total cost of $304,000. The total factory wages for November were $42,000, 90% of which were for direct labor. Arrow manufactured 19,000 units of product during November using 142,500 pounds of direct materials and 5,000 direct labor hours.

217. The direct materials purchase price variance for November is (E)A. $16,000 favorable. C. $14,250 favorable.B. $16,000 unfavorable. D. $14,250 unfavorable.

218. The direct materials usage (quantity) variance for November is (E)A. $14,400 unfavorable. C. $17,100 unfavorable.B. $1,100 favorable. D. $17,100 favorable.

219. The direct labor price (rate) variance for November is (E)A. $2,200 favorable. C. $2,000 unfavorable.B. $1,900 unfavorable. D. $2,090 favorable.

220. The direct labor usage (efficiency) variance for November is (E)A. $2,200 favorable. C. $2,000 unfavorable.B. $2,000 favorable. D. $1,800 unfavorable.

Questions 138 through 141 are based on the following information. CMA 0692 3-18 to 21Jackson Industries, which employs a standard cost system in which direct materials inventory is carried at standard cost. Jackson has established the following standards for the prime costs of one unit of product. During May, Jackson purchased 125,000 pounds of direct materials at a total cost of $475,000. The total factory wages for May were $364,000, 90% of which were for direct labor.Jackson manufactured 22,000 units of product during May using 108,000 pounds of direct materials and 28,000 direct labor hours.

Standard Quantity Standard Price Standard CostDirect materials 5 pounds $3.60/pound $18.00Direct labor 1.25 hours $12.00/hr. 15.00

$33.00

221. The purchase price variance for the direct materials acquired by Jackson Industries during May is (E)a. $21,600 favorable. c. $28,000 favorable.b. $25,000 unfavorable. d. $21,600 unfavorable.

222. The direct materials usage (quantity) variance for May is (E)A. $7,200 unfavorable. C. $5,850 unfavorable.B. $7,600 favorable. D. $7,200 favorable.

223. The direct labor price (rate) variance for May is (E)A. $8,400 favorable. C. $8,400 unfavorable.B. $7,200 unfavorable. D. $6,000 favorable.

224. The direct labor usage (efficiency) variance for May is (E)A. $5,850 favorable. C. $5,850 unfavorable.B. $6,000 unfavorable. D. $6,000 favorable.

Questions 78-82 are based on the following information: G & N 10e

CMA EXAMINATION QUESTIONS Page 82 of 142

MANAGEMENT ADVISORY SERVICES STANDARD COSTS AND VARIANCE ANALYSIS

The Geurtz Company uses standard costing. The company makes and sells a single product called a Roff. The following data are for the month of August:

Actual cost of direct material purchased and used: $65,560 Material price variance: $5,960 unfavorable Total materials variance: $22,360 unfavorable Standard cost per pound of material: $4 Standard cost per direct labor hour: $5 Actual direct labor hours: 6,500 hours Labor efficiency variance: $3,500 favorable Standard number of direct labor hours per unit of Roff: 2 hours Total labor variance: $400 unfavorable

78. The total number of units of Roff produced during August was: A. 10,800. C. 3,600.B. 14,400. D. 6,500.

79. The standard material allowed to produce one unit of Roff was: A. 1 lb. C. 3 lbs.B. 4 lbs. D. 2 lbs.

80. The actual material cost per pound was: A. $4.00. C. $3.30. B. $3.67. D. $4.40.

81. The actual direct labor rate per hour was: A. $ 5.60. C. $10.00. B. $ 5.00. D. $ 4.40.

82. The labor rate variance was: A. $3,900 favorable. C. $3,100 unfavorable.B. $3,900 unfavorable. D. $3,100 favorable.

Questions 52 through 55 are based on the following information. BarfieldThe following July information is for Kingston Company:

Standards: Material 3.0 feet per unit @ $4.20 per foot Labor 2.5 hours per unit @ $7.50 per hourActual: Production 2,750 units produced during the month Material 8,700 feet used; 9,000 feet purchased @ $4.50 per foot Labor 7,000 direct labor hours @ $7.90 per hour

(Round all answers to the nearest dollar.)

52. What is the material price variance (calculated at point of purchase)?a. $2,700 U c. $2,610 Fb. $2,700 F d. $2,610 U

53. What is the material quantity variance?a. $3,105 F c. $3,105 Ub. $1,050 F d. $1,890 U

54. What is the labor rate variance?a. $3,480 U c. $2,800 Ub. $3,480 F d. $2,800 F

55. What is the labor efficiency variance?a. $1,875 U c. $1,875 Ub. $938 U d. $1,125 U

Questions 61 through 64 are based on the following information. BarfieldThe following March information is available for Batt Manufacturing Company when it produced 2,100 units:

Standard: Material 2 pounds per unit @ $5.80 per pound Labor 3 direct labor hours per unit @ $10.00 per hourActual: Material 4,250 pounds purchased and used @ $5.65 per pound Labor 6,300 direct labor hours at $9.75 per hour

61. What is the material price variance?a. $637.50 U c. $630.00 Ub. $637.50 F d. $630.00 F

62. What is the material quantity variance?a. $275 F c. $290 Ub. $290 F d. $275 U

63. What is the labor rate variance?a. $1,575 U c. $1,594 Ub. $1,575 F d. $0

64. What is the labor efficiency variance?

CMA EXAMINATION QUESTIONS Page 83 of 142

MANAGEMENT ADVISORY SERVICES STANDARD COSTS AND VARIANCE ANALYSIS

a. $731.25 F c. $750.00 Fb. $731.25 U d. none of the above

Questions 57 thru 60 are based on the following information. G & N 9eBryan Company employs a standard cost system in which direct materials inventory is carried at standard cost. Bryan has established the following standards for the prime costs of one unit of product:

Standard Quantity Standard Price Standard CostDirect materials .... 6 pounds $ 3.50/pound $21.00Direct labor ........ 1.3 hours $11.00/hour 14.30

$35.30During March, Bryan purchased 165,000 pounds of direct material at a total cost of $585,750. The total factory wages for March were $400,000, 90 percent of which were for direct labor. Bryan manufactured 25,000 units of product during March using 151,000 pounds of direct material and 32,000 direct labor hours.

57. The price variance for the direct material acquired by the company during March is: (M)a. $7,550 favorable. c. $7,550 unfavorable.b. $8,250 unfavorable. d. $8,250 favorable.

58. The direct material quantity variance for March is: (M)a. $3,500 unfavorable. c. $3,500 favorable.b. $3,550 favorable. d. $3,550 unfavorable.

59. The direct labor rate variance for March is: (E)a. $ 8,000 favorable. c. $ 8,000 unfavorable.b. $48,000 unfavorable. d. $48,000 favorable.

60. The direct labor efficiency variance for March is: (E)a. $5,625 unfavorable. c. $5,625 favorable.b. $5,500 favorable. d. $5,500 unfavorable.

Questions 81 thru 84 are based on the following information. G & N 9eThe Alpha Company produces toys for national distribution. Standards for a particular toy are:

Materials: 12 ounces per unit at 56¢ per ounce.Labor: 2 hours per unit at $2.75 per hour.

During the month of December, the company produced 1,000 units. Information for the month follows: Materials: 14,000 ounces were purchased and used at a total cost of $7,140. Labor: 2,500 hours worked at a total cost of $8,000.

81. The materials price variance is: (E)a. $700 U. c. $420 F.b. $420 U. d. $700 F.

82. The materials quantity variance is: (E)a. $1,120 U. c. $1,820 U.b. $1,820 F. d. $1,120 F.

83. The labor rate variance is: (E)a. $2,500 F. c. $1,125 U.b. $1,125 F. d. $2,500 U.

84. The labor efficiency variance is: (E)a. $1,600 U. c. $1,375 F.b. $1,375 U. d. $1,600 F.

Questions 56 through 60 are based on the following information. BarfieldTimothy Company has the following information available for October when 3,500 units were produced (round answers to the nearest dollar).

Standards: Material 3.5 pounds per unit @ $4.50 per pound Labor 5.0 hours per unit @ $10.25 per hourActual: Material purchased 12,300 pounds @ $4.25 Material used 11,750 pounds Labor 17,300 direct labor hours @ $10.20 per hour

56. What is the labor rate variance?a. $875 F c. $865 Ub. $865 F d. $875 U

57. What is the labor efficiency variance?a. $2,050 F c. $2,040 Ub. $2,050 U d. $2,040 F

58. What is the material price variance (based on quantity purchased)?a. $3,075 U c. $2,938 Fb. $2,938 U d. $3,075 F

59. What is the material quantity variance?a. $2,250 F c. $225 F

CMA EXAMINATION QUESTIONS Page 84 of 142

MANAGEMENT ADVISORY SERVICES STANDARD COSTS AND VARIANCE ANALYSIS

b. $2,250 U d. $2,475 U

60. Assume that the company computes the material price variance on the basis of material issued to production. What is the total material variance?a. $2,850 U c. $5,188 Fb. $5,188 U d. $2,850 F

Questions 70-73 are based on the following information: G & N 10eThe Thompson Company uses standard costing and has established the following direct material and direct labor standards for each unit of Lept.

Direct materials 2 gallons at $4 per gallonDirect labor 0.5 hours at $8 per hour

During September, the company made 6,000 Lepts and incurred the following costs:Direct materials purchased 13,400 gallons at $4.10 per gallonDirect materials used 12,600 gallonsDirect labor used 2,800 hours at $7.65 per hour

70. The material price variance for September was: A. $1,340 favorable. C. $1,260 unfavorable. B. $1,260 favorable. D. $1,340 unfavorable.

71. The material quantity variance for September was: A. $2,460 unfavorable. C. $2,400 unfavorable.B. $5,600 unfavorable. D. $5,740 unfavorable.

72. The labor rate variance for September was: A. $1,530 unfavorable. C. $ 280 favorable.B. $ 980 favorable. D. $ 980 unfavorable.

73. The labor efficiency variance for September was: A. $33,600 favorable. C. $22,400 favorable. B. $ 1,600 favorable. D. $ 3,200 favorable.

THE FOLLOWING INFORMATION APPLIES TO QUESTIONS 111 THROUGH 113. HorngrenRuben’s Camera Shop has prepared the following flexible budget for September and is in the process of interpreting the variances. F denotes a favorable variance and U denotes an unfavorable variance.

VariancesFlexible Budget Price Efficiency

Material A $20,000 $1,000F $3,000UMaterial B 30,000 500U 1,500F

Direct manufacturing labor 40,000 500U 2,500F

111.The MOST likely explanation of the above variances for Material A is that (M)a. a lower price than expected was paid for Material A.b. higher-quality raw materials were used than were planned.d. the company used a higher-priced supplier.d. Material A used during September was $2,000 less than expected.

225. The actual amount spent for Material B was (E)a. $28,000. c. $30,000.b. $29,000. d. $31,000.

113.The MOST likely explanation of the above direct manufacturing labor variances is that (M)a. the average wage rate paid to employees was less than expected.b. employees did not work as efficiently as expected to accomplish the job. c. the company may have assigned more experienced employees this month than originally

planned.d. management may have a problem with budget slack and be using lax standards for both

labor-wage rates and expected efficiency.

CMA EXAMINATION QUESTIONS Page 85 of 142

MANAGEMENT ADVISORY SERVICES STANDARD COSTS AND VARIANCE ANALYSIS

THE FOLLOWING INFORMATION APPLIES TO QUESTIONS 96 THROUGH 100. HorngrenRobb Industries Inc. (RII) developed standard costs for direct material and direct labor. In 2004, RII estimated the following standard costs for one of their major products, the 10-gallon plastic container.

Budgeted quantity Budgeted priceDirect materials 0.10 pounds $30 per poundDirect labor 0.05 hours $15 per hour

During June RII produced and sold 5,000 containers using 490 pounds of direct materials at an average cost per pound of $32 and 250 direct manufacturing labor-hours at an average wage of $15.25 per hour.

226. June’s direct material flexible-budget variance is (E)a. $980 unfavorable. c. $680 unfavorable.b. $300 favorable. d. none of the above.

227. June’s direct material price variance is (E)a. $980 unfavorable. c. $680 favorable.b. $300 favorable. d. none of the above.

228. June’s direct material efficiency variance is (E)a. $980 unfavorable. c. $680 favorable.b. $300 favorable. d. none of the above.

229. June’s direct manufacturing labor price variance is (E)a. $62.50 unfavorable. c. $3,811.75 unfavorable.b. $62.50 favorable. d. none of the above.

230. June’s direct manufacturing labor efficiency variance isa. $62.50 unfavorable. c. $3,811.75 unfavorable.b. $62.50 favorable. d. none of the above.

THE FOLLOWING INFORMATION APPLIES TO QUESTIONS 101 THROUGH 106. HorngrenSawyer Industries Inc. (SII) developed standard costs for direct material and direct labor. In 2004, SII estimated the following standard costs for one of their major products, the 30-gallon heavy-duty plastic container.

Budgeted quantity Budgeted priceDirect materials 0.20 pounds $25 per pound

Direct labor 0.10 hours $15 per hourDuring July SII produced and sold 10,000 containers using 2,200 pounds of direct materials at an average cost per pound of $24 and 1,050 direct manufacturing labor hours at an average wage of $14.75 per hour.

231. July’s direct material flexible-budget variance is (E)a. $2,800 unfavorable. c. $5,000 unfavorable.b. $2,200 favorable. d. none of the above.

232. July’s direct material price variance is (Ea. $2,800 favorable. c. $5,000 unfavorable.b. $2,200 favorable. d. none of the above.

233. July’s direct material efficiency variance is (E)a. $2,800 unfavorable. c. $5,000 unfavorable.b. $2,200 favorable. d. none of the above.

234. July’s direct manufacturing labor flexible-budget variance is (E)a. $750.00 unfavorable. c. $487.50 unfavorable.b. $262.50 favorable. d. none of the above.

235. July’s direct manufacturing labor price variance is (E)a. $750.00 unfavorable. c. $487.50 favorable.b. $262.50 favorable. d. none of the above.

236. July’s direct manufacturing labor efficiency variance isa. $750.00 unfavorable. c. $487.50 favorable.b. $262.50 favorable. d. none of the above.

Questions 66-69 are based on the following information: G & N 10eArrow Industries employs a standard cost system in which direct materials inventory is carried at standard cost. Arrow has established the following standards for the prime costs of one unit of product.

Standard Quantity Standard Price Standard CostDirect materials 8 pounds $1.80 per pound $14.40Direct labor 0.25 hour $8.00 per hour $ 2.00

During May, Arrow purchased 160,000 pounds of direct material at a total cost of $304,000. The total direct labor wages for May were $37,800. Arrow manufactured 19,000 units of product during May using 142,500 pounds of direct material and 5,000 direct labor hours.

CMA EXAMINATION QUESTIONS Page 86 of 142

MANAGEMENT ADVISORY SERVICES STANDARD COSTS AND VARIANCE ANALYSIS

66. The direct material price variance for May is: A. $16,000 favorable. C. $14,250 favorable.B. $16,000 unfavorable. D. $14,250 unfavorable.

67. The direct material quantity variance for May is: A. $14,400 unfavorable. C. $17,100 unfavorable. B. $1,100 favorable. D. $17,100 favorable.

68. The direct labor rate variance for May is: A. $2,200 favorable. C. $2,000 unfavorable.B. $1,900 unfavorable. D. $2,090 favorable.

69. The direct labor efficiency variance for May is: A. $2,200 favorable. C. $2,000 unfavorable. B. $2,000 favorable. D. $1,800 unfavorable.

DM & DL Mix & Yield VariancesQuestions 96 through 101 are based on the following information. BarfieldXtra Klean manufactures a cleaning solvent. The company employs both skilled and unskilled workers. Skilled workers class C are paid $12 per hour, while unskilled workers class D are paid $7 per hour. To produce one 55-gallon drum of solvent requires 4 hours of skilled labor and 2 hours of unskilled labor. The solvent requires 2 different materials: A and B. The standard and actual material information is given below:

Standard:Material A: 30.25 gallons @ $1.25 per gallonMaterial B: 24.75 gallons @ $2.00 per gallonActual:Material A: 10,716 gallons purchased and used @ $1.50 per gallonMaterial B: 17,484 gallons purchased and used @ $1.90 per gallonSkilled labor hours: 1,950 @ $11.90 per hourUnskilled labor hours: 1,300 @ $7.15 per hour

During the current month Xtra Klean manufactured 500 55-gallon drums. (Round all answers to the nearest whole dollar.)

96. What is the total material price variance?a. $877 F c. $931 Ub. $877 U d. $931 F

97. What is the total material mix variance?a. $3,596 F c. $4,864 Fb. $3,596 U d. $4,864 U

98. What is the total material yield variance?a. $1,111 U c. $2,670 Ub. $1,111 F d. $2,670 F

99. What is the labor rate variance?a. $0 c. $2,583 Ub. $1,083 U d. $1,083 F

100.What is the labor mix variance?a. $1,083 U c. $1,083 Fb. $2,588 U d. $2,588 F

101.What is the labor yield variance?a. $2,583 U c. $1,138 Fb. $2,583 F d. $1,138 U

Total Variance and DL VarianceQuestions 142 and 143 are based on the following information. RPCPA 0583Edsol Company uses flexible budget in its standard cost system to develop variances. The following selected data are given.

Data on standard costs:Raw materials per unit 5 lbs. at P1.00/lb., P5.00Direct labor per unit 8 hrs. at P3.00/hr., P24.00Variable factory overhead per unit P3.00 per direct labor hour, P24.00Fixed factory overhead per month P25,000Normal activity per month 8,000 direct labor hoursUnits produced in April 1,000 unitsCosts incurred for April

Raw materials 5,000 lbs. at P1.10/lb.Direct labor 7,000 lbs. at P3.10/hr.Variable factory overhead P27,000Fixed factory overhead P28,000

*. The total variance to be explained for April is (M)a. P4,200 favorable c. P5,200 favorableb. P4,200 unfavorable d. P5,200 unfavorable

CMA EXAMINATION QUESTIONS Page 87 of 142

MANAGEMENT ADVISORY SERVICES STANDARD COSTS AND VARIANCE ANALYSIS

*. The labor efficiency variance for April is (M)a. P3,000 favorable c. P3,100 favorableb. P3,000 unfavorable d. P3,100 unfavorable

DM and OH VariancesQuestions 144 through 148 are based on the following information. GleimFuntime, Inc, manufactures video game machines. Market saturation and technological innovations have caused pricing pressures which have resulted in declining profits. To stem the slide in profits until new products can be introduced, an incentive program has been developed to reward production managers who contribute to an increase in the number of units produced and effect cost reductions.The managers have responded to the pressure of improving manufacturing in several ways. The video game machines are put together by the Assembly Group which requires parts from both the Printed Circuit Boards (PCB) and the Reading Heads (RH) groups. To attain increased production levels, the PCB and RH groups commenced rejecting parts that previously would have been tested and modified to meet manufacturing standards. Preventive maintenance on machines used in the production of these parts has been postponed with only emergency repair work being performed to keep production lines moving.The more aggressive Assembly Group production supervisors have pressured maintenance personnel to attend to their machines at the expense of other groups. This has resulted in machine downtime in the PCB and RH groups which, when coupled with demands for accelerated parts delivery by the Assembly Group, has led to more frequent parts rejections and increased friction among departments.Funtime operates under a standard cost system. The standard costs for video game machines are as follows:

Standard Cost per UnitCost Item Quantity Cost TotalDirect Materials Housing unit 1 $20 $20 Printed circuit boards 2 15 30 Reading heads 4 10 40Direct labor Assembly group 2 hours 8 16 PCB group 1 hour 9 9 RH group 1.5 hours 10 15Variable overhead 4.5 hours 2 9 Total standard cost per unit $139

Funtime prepares monthly performance reports based on standard costs. Presented below is the contribution report for May, when production and sales both reached 2,200 units.

Funtime Inc.Contribution Report

For the Month of May

Budget Actual VarianceUnits 2,000 2,200 200FRevenue $400,000 $440,000 $40,000FVariable costs Direct material 180,000 220,400 40,400U Direct labor 80,000 93,460 13,460U Variable overhead 18,000 18,800 800U Total variable costs 278,000 332,660 54,660UContribution margin $122,000 $107,340 $14,660U

Funtime’s top management was surprised by the unfavorable contribution to overall corporate profits in spite of the increased sales in May. Jack Rath, cost accountant, was assigned to identify to reasons for the unfavorable contribution results as well as the individuals or groups responsible. After review, Rath prepared the Usage Report presented below.

Funtime Inc.Usage Report

For the Month of May

Cost Item Quantity Actual CostDirect material Housing units 2,200 units $ 44,000 Printed circuit boards 4,700 units 75,200 Reading heads 9,200 units 101,200Direct labor Assembly 3,900 hours 31,200 Printed circuit boards 2,400 hours 23,760 Reading heads 3,500 hours 38,500Variable overhead 9,900 hours 18,800Total variable cost $332,660

Rath reported that the PCB and RH groups supported the increased production levels but experienced abnormal machine downtime, causing idle manpower which required the use of overtime to keep u with the accelerated demand for parts. The idle time was charged to direct labor. Rath also reported that the production managers of these two groups resorted to parts rejections, as opposed to testing and modification procedures formerly applied. Rath determined that the Assembly Group met management’s objectives by increasing production while using lower than standard hours.

237. What is the total materials price variance? (M)a. $346,500 favorable. c. $13,900 favorable.b. $346,500 unfavorable d. $13,900 unfavorable.

CMA EXAMINATION QUESTIONS Page 88 of 142

MANAGEMENT ADVISORY SERVICES STANDARD COSTS AND VARIANCE ANALYSIS

238. What is the total materials quantity variance? (M)a. $8,500 unfavorable. c. $9,200 unfavorable.b. $8,500 favorable. d. $9,200 favorable.

239. What is the variable overhead efficiency variance? (M)a. $0 c. $9,900 unfavorable.b. $900 unfavorable. d. $9,900 favorable.

240. What is the variable overhead spending variance? (M)a. $1,000 unfavorable. c. $1,800 unfavorable.b. $1,000 favorable. d. $1,800 favorable.

241. What is the contribution margin volume variance? (M)a. $9,800 unfavorable. c. $12,200 favorable.b. $9,800 favorable. d. $14,660 unfavorable.

DL and OH Variances27. SUPER Co. at normal capacity, operates at 600,000 labor hours with standard labor rate of

P20 per hour. Variable factory overhead is applied at the rate of P12 per labor hour. Four units should be completed in an hour.Last year, 1,350,000 units were produced using 300,000 labor hours. All labor hours were paid at the standard rate, and actual overhead cost consisted of P3,738,000 for variable items and P3,000,000 fixed items.The total labor and overhead costs saved, by producing at more than standard, amounted to (M)a. P450,000 c. P750,000b. P500,000 d. P1,200,000 RPCPA 0594

Questions 149 through 151 are based on the following information. CMA 0692 3-15 to 17An organization that specializes in reviewing and editing technical magazine articles. It set the following standards for evaluating the performance of the professional staff:

Annual budgeted fixed costs for normal capacity level of 10,000 articles reviewed and edited

$600,000

Standard professional hours per 10 articles 200 hoursFlexible budget of standard labor costs to process 10,000 articles $10,000,000

The following data apply to the 9,500 articles that were actually reviewed and edited during the current year.

Total hours used by professional staff 192,000 hoursFlexible costs $9,120,000Total cost 9,738,000

242. Using a flexible budget, the total cost planned for the review and editing of 9,500 articles should be (D)A. $9,500,000. C. $10,100,000.B. $10,070,000. D. $10,570,000.

243. The fixed cost spending variance for the year is (D)A. $18,000 unfavorable. C. $48,000 unfavorable.B. $30,000 favorable. D. $18,000 favorable.

244. The labor efficiency variance for the year is (D)A. $100,000 unfavorable. C. $380,000 favorable.B. $238,000 unfavorable. D. $500,000 favorable.

CMA EXAMINATION QUESTIONS Page 89 of 142

MANAGEMENT ADVISORY SERVICES STANDARD COSTS AND VARIANCE ANALYSIS

Questions 152 and 153 are based on the following information. CMA 0693 3-19 & 20Tiny Tykes Corporation had the following activity relating to its fixed and variable factory overhead for the month of July.

Actual costs Fixed overhead $120,000 Variable overhead 80,000

Flexible budget(Standard input allowed for actual output achieved x the budgeted rate) Variable overhead 90,000

Applied(Standard input allowed for actual output achieved x the budgeted rate) Fixed overhead 125,000

Variable overhead spending variance 2,000FProduction volume variance 5,000U

245. If the budgeted rate for applying variable factory overhead was $20 per direct labor hour, how efficient or inefficient was Tiny Tykes Corporation in terms of using direct labor hours as an activity base? (M)a. 100 direct labor hours inefficient. c. 400 direct labor hours inefficient.b. 100 direct labor hours efficient. d. 400 direct labor hours efficient.

246. The fixed factory overhead efficiency variance is (M)a. $3,000 favorable. c. $5,000 favorable.b. $3,000 unfavorable. d. Never a meaningful variance.

Questions 154 through 156 are based on the following information. RPCPA 1087The summarized flexible budget of AAA, Inc. is shown below:

Percent of Normal Operating Capacity80% 90% *100% 110%

Variable overhead P25,000 P30,000 P35,000 P40,000Fixed overhead 50,000 50,000 50,000 50,000Total Factory overhead P75,000 P80,000 P85,000 P90,000

* Normal capacityAccording to standards established, 150,000 units of the product should be manufactured at its normal capacity. Standard labor time per unit of products is ten minutes. Actual production in 1986 was 132,000 units in 24,000 hours.

CMA EXAMINATION QUESTIONS Page 90 of 142

MANAGEMENT ADVISORY SERVICES STANDARD COSTS AND VARIANCE ANALYSIS

*. What is the standard direct labor time allowed to finish 132,000 units? (E)a. 22,000 hours c. 25,000b. 24,000 hours d. none of these

*. What is the standard variable factory overhead rate? (E)a. P0.96 c. P1.40b. P1.00 d. none of the above

*. What is the standard fixed factory overhead rate? (E)a. P1.50 c. P2.00b. P1.75 d. none of the above

Questions 157 through 161 are based on the following information. CMA 1294 3-26 to 30Water Control Inc. manufactures water pumps and uses a standard cost system. The standard factory overhead costs per water pump are based on direct labor hours and are as follows:

Variable overhead (4 hours of $8/hour) $32Fixed overhead (4 hours at $5*/per hour 20Total overhead cost per unit $52

* Based on a capacity of 100,000 direct labor hours per month.

The following additional information is available for the month of November. 22,000 pumps were produced although 25,000 had been scheduled for production. 94,000 direct labor hours were worked at a total cost of $940,000. The standard direct labor rate is $9 per hour. The standard direct labor time per unit is 4 hours. Variable overhead costs were $740,000. Fixed overhead costs were $540,000.

247. The fixed overhead spending variance for November was (E)a. $40,000 unfavorable. c. $460,000 unfavorable.b. $70,000 unfavorable. d. $240,000 unfavorable.

248. The variable overhead spending variance for November was (E)a. $60,000 favorable. c. $48,000 unfavorable.b. $12,000 favorable. d. $40,000 unfavorable.

249. The variable overhead efficiency variance for November was (E)a. $48,000 unfavorable. c. $96,000 unfavorable.b. $60,000 favorable. d. $200,000 unfavorable.

250. The direct labor price variance for November was (E)a. $54,000 unfavorable. c. $60,000 favorable.b. $94,000 unfavorable. d. $148,000 unfavorable.

251. The direct labor efficiency variance for November was (E)a. $108,000 favorable. c. $60,000 favorable.b. $120,000 favorable d. $54,000 unfavorable.

DL & Variable Overhead VariancesQuestions 93 thru 97 are based on the following information. G & N 9eThe Clark Company makes a single product and uses standard costing. Some data concerning this product for the month of May follow:

Labor rate variance:.................................. $ 7,000 FLabor efficiency variance:............................ $12,000 FVariable overhead efficiency variance:................ $ 4,000 FNumber of units produced:............................. 10,000Standard labor rate per direct labor hour:............ $12Standard variable overhead rate per direct labor hour: $ 4Actual labor hours used:.............................. 14,000Actual variable manufacturing overhead costs:......... $58,290

93. The variable overhead spending variance for May was: (M)a. $2,290 F. c. $1,710 F.b. $2,290 U. d. $1,710 U.

94. The actual direct labor rate for May in dollars per hour was: (D)a. $12.50. c. $11.75.b. $12.00. d. $11.50.

95. The total standard cost for direct labor for May was: (D)a. $168,000. c. $120,000.b. $180,000. d. $161,000.

CMA EXAMINATION QUESTIONS Page 91 of 142

MANAGEMENT ADVISORY SERVICES STANDARD COSTS AND VARIANCE ANALYSIS

96. The total standard cost for variable overhead for May was: (D)a. $56,000. c. $60,000.b. $40,000. d. $50,000.

97. The standard hours allowed to make one unit of finished product are: (D)a. 1.0. c. 1.5.b. 1.2. d. 2.0.

DM, DL and OH VariancesQuestions 3 through 6 are based on the following information. RPCPA 580The Willard Manufacturing Co., Inc. uses standard cost systems in accounting for manufacturing costs. On June 1, 19x9, it started the manufacture of a new product known as “Whippy.” The standard costs of a unit of “Whippy” are:

Raw materials 3 kilos @ P1.00 per kilo P3.00Direct labor 1 hour @ P4.00 per hour 4.00Overhead 75% of direct labor cost 3.00

P10.00The following data were obtained from Willard’s records for the month of June:

Actual production of “Whippy” 2,000 unitsUnits sold of “Whippy” 1,250 units

Debit CreditSales P25,000Purchases P13,650Materials price variance 650Materials quantity variance 500Direct labor rate variance 380Direct labor efficiency variance 400Manufacturing overhead total variance 250

The amount shown above for the materials price variance is applicable to raw materials purchased during June.

3. The actual quantity of raw materials used (in kilos) for the month of June isa. 3,750 kilos c. 6,500 kilosb. 6,000 kilos d. 6,650 kilos

4. The actual hours worked for the month of June isa. 1,900 hours c. 2,000 hours.b. 1,905 hours d. 2,100 hours.

5. The actual total overhead for the month of June is

a. P3,750 c. P6,000b. P5,750 d. P6,250

6. The actual direct labor rate for the month of June isa. P3.60 c. P4.00b. P3.80 d. P4.20

Questions 162 through 164 are based on the following information. RPCPA 1088MAXIM MFG CO., which uses a standard cost system, manufactures one product with the following standard costs:

Direct materials 2 Kilos at P10 P20.00Direct labor 1 hour at P8 8.00Factory overhead 80% of direct labor 6.40TOTAL STANDARD UNIT COST P34.40

Total production in units 10,000 unitsDirect materials purchased 22,000 kilos at P11Actual quantity of materials used 21,000 kilosActual labor cost 9,500 at P7.50Factory overhead total variance P1,000 unfavotable

*. The direct material usage variance is (E)a. P11,000 unfavorable c. P10,000 unfavorableb. P20,000 unfavorable d. P40,000 favorable

CMA EXAMINATION QUESTIONS Page 92 of 142

MANAGEMENT ADVISORY SERVICES STANDARD COSTS AND VARIANCE ANALYSIS

*. The direct labor efficiency variance is (E)a. P4,000 favorable c. P3,750 favorableb. P3,750 unfavorable d. P4,750 favorable

*. The actual total factory overhead for the month of January is (D)a. P65,000 c. P57,000b. P63,000 d. P60,000

Questions 165 through 167 are based on the following information. RPCPA 0582Superior Company, which started business on April 1, uses a standard cost system in accounting for manufacturing costs. The standard costs for a unit of its product are:

Materials: 2 units at P3 per kilo P6.00Labor: 1 hour at P4 per hour 4.00Factory overhead: 75% of direct labor cost 3.00

P13.00

Following data were gathered from Superior’s records for April:Units produced 5,000Units sold 4,000Sales P100,000Purchases (11,000 kilos) P 38,500Materials price variance (applicable to April purchases) P550 unfavorableActual quantity of materials used 10,500 kilosActual labor hours worked 4,800 hoursDirect labor rate variance P800 favorableFactory overhead total variance P500 unfavorable

*. The material quantity variance for April was (E)a. P500 c. P1,500b. P1,000 d. P2,000

*. The direct labor efficiency variance for April was (E)a. P800 c. P200b. P400 d. P4

*. The actual factory overhead for April was (E)a. P500 c. P15,000b. P5,000 d. P15,500

Questions 168 through 171 are based on the following information. CMA 0696 3-22 to 25Ardmore Enterprises uses a standard cost system in its small appliance division. The standard cost of manufacturing one unit of Zeb is as follows:

Materials = 60 pounds at $1.50 per pound $ 90Labor = 3 hours at $12 per hour 36Factory overhead – 3 hours at $8 per hour 24 Total standard cost per unit $150

The budgeted variable factory overhead rate is $3 per labor hour, and the budgeted fixed factory overhead is $27,000 per month. During May, Ardmore produced 1,650 units of Zeb compared with a normal capacity of 1,800 units. The actual cost per unit was as follows:

Materials (purchased and used) 58 pounds at $1.65 per pound) $95.70Labor = 3.1 hours at $12 per hour 37.20Factory overhead – $39,930 per 1,650 units 24.20 Total actual cost per unit $157.10

252. The total materials quantity variance for May is (E)a. $14,355 favorable. c. $4,950 favorable.b. $14,355 unfavorable. d. $4,950 unfavorable.

253. The materials price variance for May is (E)a. $14,355 unfavorable. c. $14,355 favorable.b. $14,850 unfavorable. d. $14,850 favorable.

254. The labor rate variance for May is (E)a. $1,920 favorable. c. $4,950 unfavorable.b. $0. d. $4,950 favorable.

CMA EXAMINATION QUESTIONS Page 93 of 142

MANAGEMENT ADVISORY SERVICES STANDARD COSTS AND VARIANCE ANALYSIS

255. The flexible overhead variance for May is (E)a. $3,270 unfavorable. c. $1,920 unfavorable.b. $3,270 favorable. d. $1,920 favorable.

Questions 172 through 175 are based on the following information. GleimLondon Enterprises uses a standard cost system in its appliance division. The standard cost of manufacturing one unit of Gimmicks is as follows:

Materials – 120 pounds at $1.50 per pound $180Labor – 3 hours @ $15 per hour 45Factory overhead - $16 per labor hour 48 Total standard cost per unit $273

The budgeted variable factory overhead rate is $6 per labor hour, and the budgeted fixed factory overhead is $54,000 per month. During June, London Enterprises produced 1,650 units of Gimmicks compared with a normal capacity of 1,800 units. The actual cost per unit was

Materials (purchased and used) – 116 pounds at $1.65 per pound $191.40Labor – 3.1 hours @ $15 per hour 46.50Factory overhead ($79,860 for 1,650 units) 48.40 Total standard cost per unit $286,30

256. The total materials quantity variance for June is (E)a. $9,900 favorable. c. $28,710 favorable.b. $9,900 unfavorable. d. $28,710 unfavorable.

257. The materials price variance for June is (E)a. $28,710 unfavorable. c. $28,710 favorable.b. $29,700 unfavorable d. $29,700 favorable.

258. The labor rate variance for June is (E)a. $2,700 unfavorable. c. $2,475 unfavorable.b. $2,700 favorable. d. $0

259. The flexible budget overhead variance for June is (E)a. $0 c. $3,840 unfavorable.b. $6,540 favorable. d. $3,840 favorable.

Questions xx thru xx are based on the following information. H & MThe Wade Company has developed the following standards for one of its products.

Direct materials: 20 pounds x $6 per poundDirect labor: 5 hours x $16 per hourVariable manufacturing overhead: 5 hours x $4 per hour

The following activity occurred during the month of November:Materials purchased: 250,000 pounds $5.20 per pound

CMA EXAMINATION QUESTIONS Page 94 of 142

MANAGEMENT ADVISORY SERVICES STANDARD COSTS AND VARIANCE ANALYSIS

Materials used: 220,000 poundsUnits produced: 10,000 unitsDirect labor: 48,000 hours at $15.00 per hourActual variable manufacturing overhead: $204,000

The company records materials price variances at the time of purchase.

260. The direct materials price variance isa. $32,000 unfavorable c. $48,000 unfavorableb. $32,000 favorable d. $48,000 favorable

261. The direct labor efficiency variance isa. $32,000 favorable c. $80,000 unfavorableb. $32,000 unfavorable d. $80,000 favorable

262. The variable manufacturing overhead spending variance isa. $4,000 favorable c. $8,000 favorableb. $4,000 unfavorable d. $12,000 unfavorable

263. The variable manufacturing overhead efficiency variance isa. $4,000 favorable c. $8,000 favorableb. $4,000 unfavorable d. $12,000 unfavorable

Questions 1 thru 3 are based on the following information. H & MSheridan Manufacturing Company uses a standard cost system. The following information pertains to 1995.

Actual factory overhead costs ($22,000 is fixed) $53,500Actual direct labor costs (22,500 hours) $175,500Standard direct labor for 11,000 units:Standard hours allowed 22,000 hrs.Labor rate $8.00

The factory overhead rate is based on an activity level of 10,000 units. Standard cost data or 10,000 units is:

Variable factory overhead $30,000Fixed factory overhead 18,000Total factory overhead $48,000

264. What is the total labor budget variance for Sheridan Manufacturing Company?a. $15,500(U) d. $20,000(U)b. $4,000(U) e. $500(F)c. $4,500(F)

265. What is the variable overhead efficiency variance for Sheridan Manufacturing Company?a. $750 (F) d. $2,250 (F)b. $4,000 (U) e. $2,250 (U)c. $750 (U)

266. What is the fixed overhead volume variance for Sheridan Manufacturing Company?a. $1,800 (F) d. $1,800 (U)b. $4,800 (F) e. $3,600 (U)c. $5,500 (U)

Questions 56-60 are based on the following information: G & N 10eCox Engineering performs cement core tests in its laboratory. The following standards have been set for each core test performed:

Standard Hours orQuantity Standard Price or Rate

Direct materials 3 pounds $0.75 per poundDirect labor 0.4 hours $12 per hourVariable manufacturing overhead 0.4 hours $9 per hour

During March, the laboratory performed 2,000 core tests. On March 1 no direct materials (sand) were on hand. Variable manufacturing overhead is assigned to core tests on the basis of direct labor hours. The following events occurred during March:

8,600 pounds of sand were purchased at a cost of $7,310. 7,200 pounds of sand were used for core tests. 840 actual direct labor hours were worked at a cost of $8,610. Actual variable manufacturing overhead incurred was $3,200.

CMA EXAMINATION QUESTIONS Page 95 of 142

MANAGEMENT ADVISORY SERVICES STANDARD COSTS AND VARIANCE ANALYSIS

56. The materials price variance for March is: A. $860 unfavorable. C. $281 unfavorable.B. $860 favorable. D. $281 favorable.

57. The materials quantity variance for March is: A. $ 900 favorable. C. $1,950 unfavorable.B. $1,950 favorable. D. $ 900 unfavorable.

58. The labor rate variance for March is: A. $4,578 unfavorable. C. $4,578 favorable. B. $1,470 unfavorable. D. $1,470 favorable.

59. The labor efficiency variance for March is: A. $480 favorable. C. $192 favorable.B. $480 unfavorable. D. $192 unfavorable.

60. The variable overhead efficiency variance for March is: A. $320 unfavorable. C. $360 unfavorable.B. $320 favorable. D. $360 favorable.

Materials, Labor & Variable Overhead VarianceQuestions 61 thru 66 are based on the following information. (Set A) G & N 9eThe Litton Company has established standards as follows:

Direct material 3 lbs. @ $4/lb. = $12 per unitDirect labor 2 hrs. @ $8/hr. = $16 per unitVariable manuf. overhead 2 hrs. @ $5/hr. = $10 per unit

Actual production figures for the past year are given below. The company records the materials price variance when materials are purchased.

Units produced 600Direct material used 2,000 lbs.Direct material purchased (3,000 lbs.) $11,400Direct labor cost (1,100 hrs.) $ 9,240Variable manuf. overhead cost incurred $ 5,720

The company applies variable manufacturing overhead to products on the basis of direct labor hours.

61. The materials price variance is: (E)a. $400 U. c. $600 F.b. $400 F. d. $600 U.

62. The materials quantity variance is: (E)a. $800 U. c. $760 U.b. $4,000 U. d. $760 F.

63. The labor rate variance is: (E)a. $480 F. c. $440 F.b. $480 U. d. $440 U.

64. The labor efficiency variance is: (E)a. $800 F. c. $840 F.b. $800 U. d. $840 U.

65. The variable overhead spending variance is: (E)a. $240 U. c. $220 F.b. $220 U. d. $240 F.

66. The variable overhead efficiency variance is: (E)a. $520 F. c. $500 U.b. $520 U. d. $500 F.

Questions 67 thru 71 are based on the following information. (Set B) G & N 9eThe Albright Company uses standard costing and has established the following standards for its single product:

Direct materials .......... 2 gallons at $3 per gallonDirect labor .............. 0.5 hours at $8 per hourVariable manuf. overhead .. 0.5 hours at $2 per hour

During November, the company made 4,000 units and incurred the following costs:Direct materials purchased ........ 8,100 gallons at $3.10 per gallonDirect materials used ............. 7,600 gallonsDirect labor used ................. 2,200 hours at $8.25 per hourActual variable manuf. overhead ... $4,175

The company applies variable manufacturing overhead to products on the basis of direct labor hours.

67. The material price variance for November was: (E)a. $2,310 U. c. $810 U.b. $2,310 F. d. $810 F.

68. The material quantity variance for November was: (E)a. $1,200 U. c. $300 U.

CMA EXAMINATION QUESTIONS Page 96 of 142

MANAGEMENT ADVISORY SERVICES STANDARD COSTS AND VARIANCE ANALYSIS

b. $1,200 F. d. $1,500 F.

69. The labor rate variance for November was: (E)a. $1,050 U. c. $2,150 U.b. $550 U. d. $2,150 F.

70. The labor efficiency variance for November was: (E)a. $1,050 U. c. $1,600 F.b. $550 U. d. $1,600 U.

71 The total variable overhead variance for November was: (M)a. $175 U. c. $225 U.b. $225 F. d. $400 U.

Questions 72 thru 76 are based on the following information. G & N 9eCole laboratories makes and sells a lawn fertilizer called Fastgro. The company has developed standard costs for one bag of Fastgro as follows:

Standard Quantity Standard Cost per BagDirect material 20 pounds $8.00Direct labor 0.1 hours 1.10Variable manuf. overhead .. 0.1 hours .40

The company had no beginning inventories of any kind on Jan. 1. Variable manufacturing overhead is applied to production on the basis of direct labor hours. During January, the following activity was recorded by the company:

Production of Fastgro: 4,000 bags Direct materials purchased: 85,000 pounds at a cost of $32,300 Direct labor worked: 390 hours at a cost of $4,875 Variable manufacturing overhead incurred: $1,475 Inventory of direct materials on Jan. 31: 3,000 pounds

72. The materials price variance for January is: (M)a. $1,640 F. c. $1,700 F.b. $1,640 U. d. $1,300 U.

73. The materials quantity variance for January is: (M)a. $800 U. c. $300 F.b. $300 U. d. $750 F.

74. The labor rate variance for January is: (M)a. $475 F. c. $585 F.b. $475 U. d. $585 U.

75. The labor efficiency variance for January is: (M)a. $475 F. c. $130 U.b. $350 U. d. $110 F.

76. The total variance for variable overhead for January is: (M)a. $85 F. c. $100 U.b. $40 F. d. $125 F.

Overhead VariancesQuestions 65 through 74 are based on the following information. BarfieldRedd Co. uses a standard cost system for its production process and applies overhead based on direct labor hours. The following information is available for August when Redd made 4,500 units:

Standard: DLH per unit 2.50 Variable overhead per DLH $1.75 Fixed overhead per DLH $3.10 Budgeted variable overhead $21,875 Budgeted fixed overhead $38,750Actual: Direct labor hours 10,000 Variable overhead $26,250 Fixed overhead $38,000

65. Using the one-variance approach, what is the total overhead variance?a. $6,062.50 U c. $9,687.50 Ub. $3,625.00 U d. $6,562.50 U

66. Using the two-variance approach, what is the controllable variance?a. $5,812.50 U c. $4,375.00 Ub. $5,812.50 F d. $4,375.00 F

67. Using the two-variance approach, what is the noncontrollable variance?a. $3,125.00 F c. $3,875.00 Fb. $3,875.00 U d. $6,062.50 U

68. Using the three-variance approach, what is the spending variance?a. $4,375 U c. $8,000 Ub. $3,625 F d. $15,750 U

69. Using the three-variance approach, what is the efficiency variance?

CMA EXAMINATION QUESTIONS Page 97 of 142

MANAGEMENT ADVISORY SERVICES STANDARD COSTS AND VARIANCE ANALYSIS

a. $9,937.50 F c. $2,187.50 Ub. $2,187.50 F d. $2,937.50 F

70. Using the three-variance approach, what is the volume variance?a. $3,125.00 F c. $3,875.00 Ub. $3,875.00 F d. $6,062.50 U

71. Using the four-variance approach, what is the variable overhead spending variance?a. $4,375.00 U c. $8,750.00 Ub. $4,375.00 F d. $6,562.50 U

72. Using the four-variance approach, what is the variable overhead efficiency variance?a. $2,187.50 U c. $2,187.50 Fb. $9,937.50 F d. $2,937.50 F

73. Using the four-variance approach, what is the fixed overhead spending variance?a. $7,000 U c. $750 Ub. $3,125 F d. $750 F

74. Using the four-variance approach, what is the volume variance?a. $3,125 F c. $6,063 Ub. $3,875 F d. $3,875 U

Questions 75 through 84 are based on the following information. BarfieldSpots Inc. uses a standard cost system for its production process. Spots applies overhead based on direct labor hours. The following information is available for July:

Standard:Direct labor hours per unit 2.20Variable overhead per hour $2.50Fixed overhead per hour (based on 11,990 DLHs) $3.00Actual:Units produced 4,400Direct labor hours 8,800Variable overhead $29,950Fixed overhead $42,300

75. Using the four-variance approach, what is the variable overhead spending variance?a. $7,950 U c. $7,975 Ub. $25 F d. $10,590 U

76. Using the four-variance approach, what is the variable overhead efficiency variance?a. $9,570 F c. $2,200 Fb. $9,570 U d. $2,200 U

77. Using the four-variance approach, what is the fixed overhead spending variance?a. $15,900 U c. $6,930 Ub. $6,330 U d. $935 F

78. Using the four-variance approach, what is the volume variance?a. $6,930 U c. $0b. $13,260 U d. $2,640 F

79. Using the three-variance approach, what is the spending variance?a. $23,850 U c. $14,280 Fb. $23,850 F d. $14,280 U

80. Using the three-variance approach, what is the efficiency variance?a. $11,770 F c. $7,975 Ub. $2,200 F d. $5,775 U

81. Using the three-variance approach, what is the volume variance?a. $13,260 U c. $6,930 Ub. $2,640 F d. $0

82. Using the two-variance approach, what is the controllable variance?a. $21,650 U c. $5,775 Ub. $16,480 U d. $12,080 U

83. Using the two-variance approach, what is the noncontrollable variance?a. $26,040 F c. $6,930 Ub. $0 d. $13,260 U

84. Using the one-variance approach, what is the total variance?a. $19,010 U c. $12,705 Ub. $6,305 U d. $4,730 U

THE FOLLOWING INFORMATION APPLIES TO QUESTIONS 111 THROUGH 115. HorngrenVariances Spending Efficiency Production-VolumeVariable manufacturing overhead $ 4,500 F $15,000 U (B)

CMA EXAMINATION QUESTIONS Page 98 of 142

MANAGEMENT ADVISORY SERVICES STANDARD COSTS AND VARIANCE ANALYSIS

Fixed manufacturing overhead $10,000 U (A) $40,000 U

111.Above is a (E)a. 4-variance analysis. c. 2-variance analysis.b. 3-variance analysis. d. 1-variance analysis.

112.In the above chart, the amounts for (A) and (B), respectively, area. $10,500 U; $55,000 U c. Zero; $55,000 Ub. $10,500 U; Zero d. Zero; Zero

267. In a 3-variance analysis the spending variance should be (E)a. $ 4,500 F. c. $ 5,500 U.b. $10,000 U. d. $10,500 U.

268. In a 2-variance analysis the flexible-budget variance and the production-volume variance should be __________, respectively.a. $5,500 U; $55,000 U c. $10,500 U; $50,000 Ub. $20,500 U; $40,000 U d. $60,500 U; Zero

269. In a 1-variance analysis the total overhead variance should be (E)a. $20,500 U. c. $121,000 U.b. $60,500 U. d. none of the above.

Relevant CostingQuestions 176 through 182 are based on the following information. GleimPortCo Products is a divisionalized furniture manufacturer. The divisions are autonomous segments, with each division being responsible for its own sales, costs of operations, working capital management, and equipment acquisition. Each division serves a different market in the furniture industry. Because the markets and products of the divisions are so different, there have never been any transfers between divisions.The Commercial Division manufactures equipment and furniture that is purchased by the restaurant industry. The division plans to introduce a new line of counter and chair units that feature a cushioned seat with Russ Fiegel of the Office Division. They both believe a cushioned seat currently made by the Office Division for use on its deluxe office stool could be modified for use on the new counter chair. Consequently, Kline has asked Russ Fiegel for a price for 100-unit lots of the cushioned seat. The following conversation took place about the price to be charged for the cushioned seats:Fiegel: “John, we can make the necessary modifications to the cushioned seat easily. The raw materials used in your seat are slightly different and should cost about 10% more than those used in our deluxe office stool. However, the labor time should be the same because the seat

fabrication operation basically is the same. I would price the seat at our regular rate – full cost plus 30% markup.”Kline: “That’s higher than I expected, Russ. I was thinking that a good price would be your variable manufacturing costs. After all, your capacity costs will be incurred regardless of this job.”Fiegel: “John, I’m at capacity. By making the cushion seats for you, I’ll have to cut my production of deluxe office stools. Of course, I can increase my production of economy office stools. The labor time freed by not having to fabricate the frame or assemble the deluxe stool can be shifted to the frame fabrication and assembly of the economy office stool. Fortunately, I can switch my labor force between these two models of stools without any loss of efficiency. As you know, overtime is not a feasible alternative in our community. I’d like to sell it to you at variable cost, but I have excess demand for both products. I don’t mind changing my product mix to the economy model if I get a good return on the seats I make for you. Here are my standard costs for the two stools and a schedule of my manufacturing overhead.:Kline: “I guess I see your point, Russ, but I don’t want to price myself out of the market. Maybe we should talk to Corporate to see if they can give us any guidance.”

CMA EXAMINATION QUESTIONS Page 99 of 142

MANAGEMENT ADVISORY SERVICES STANDARD COSTS AND VARIANCE ANALYSIS

Office DivisionStandard Costs and Prices

DeluxeOffice Stool

EconomyOffice Stool

Raw materials Framing $ 8.15 $ 9.76 Cushioned seats Padding 2.40 - Vinyl 4.00 - Molded seat (purchased) - 6.00

Direct labor Frame fabrication(0.5x$7.50/DLH) 3.75 (0.5x$7.50/DLH) 3.75 Cushion fabrication(0.5x$7.50/DLH) 3.75 - Assembly fabrication(0.5x$7.50/DLH) 3.75 (0.3x$7.50/DLH) 2.25Mfg. Overhead (1.5DLH x $12.80/DLH) 19.20 (0.8x$12.80/DLH) 10.24 Total standard cost $45.00 $32.00 Selling price (30% markup) $58.50 $41.60

*Attaching seats to frames and attaching rubber feet

CMA EXAMINATION QUESTIONS Page 100 of 142

MANAGEMENT ADVISORY SERVICES STANDARD COSTS AND VARIANCE ANALYSIS

Office DivisionManufacturing Overhead Budget

Overhead Item Nature AmountSupplies Variable – at current market prices $420,000Indirect labor Variable 375,000Supervision Nonvariable 250,000Power Use varies with activity; rates are fixed 180,000Heat and light Nonvariable – light is fixed regardless of production 140,000

while heat/air-conditioning varies with fuel charges

Property taxes and insurance taxes

Nonvariable – any change in amounts/rates is independent of production

200,000

Depreciation Fixed dollar total 1,700,000Employment benefits 20% of supervision, direct and indirect labor 575,000

Total overhead $3,840,000Capacity in DLH 300,000 Overhead rate/DLH $12.80

CMA EXAMINATION QUESTIONS Page 101 of 142

MANAGEMENT ADVISORY SERVICES STANDARD COSTS AND VARIANCE ANALYSIS

270. What amount of employee benefits is associated with direct labor costs? (M)a. $675,000 c. $450,000b. $75,000 d. $500,000

271. What is the variable manufacturing overhead rate? (M)a. $7.80/hr. c. $5.17/hr.b. $11.25/hr. d. $5.00/hr.

272. What is the transfer price per 100-unit lot based on variable manufacturing costs to produce the modified cushioned seat? (M)a. $1,329 c. $789b. $1,869 d. $1,986

273. What is the fixed manufacturing overhead rate? (M)a. $7.80/hr. c. $5.17/hr.b. $11.25/hr. d. $5.00/hr.

274. How many economy office stools can be produced with the labor hours currently used to make 100 deluxe stools? (M)a. 80. c. 100.b. 125. d. 150.

275. When computing the opportunity cost for the deluxe office stool, what is the contribution margin per unit produced? (M)a. $25.20 c. $13.56b. $15.84 d. $33.30

276. What is the opportunity cost of the Office Division if 125 economy stools can be made in the time required for 100 deluxe stools?a. $789 c. $1,329b. $1,869 d. $540

Questions 48 through 51 are based on the following information BarfieldsThe following information is provided for the IHM Co. for June 2001.

Actual Standard1,800 units 5 DLHs per unit @ $10.00 per DLH8,900 DLHs @ $10.50 per DLH VOH rate per DLH $ .75Variable OH $6,400 FOH rate per DLH $1.90Fixed OH $17,500 Budgeted FOH $16,910

48. What is the price variance?

a. $4,450 F c. $1,000 Fb. $4,450 U d. $1,000 U

49. What is the efficiency variance?a. $4,450 F c. $1,000 Fb. $4,450 U d. $1,000 U

50. What is the spending variance?a. $590 U c. $190 Fb. $590 F d. $190 U

51. What is the volume variance?a. $590 U c. $190 Fb. $590 F d. $190 U

277. Toimi Inc. had the following variances for the most recent month:Materials Price Variance $3,500 UMaterials Usage Variance $ 720 FDirect Labor Rate Variance $5,770 FDirect Labor Efficiency Variance $6,980 U

Other information included: actual wages paid $72,310; materials purchased $130,760; standards per unit were 2 labor hours at $5 per hour, 3 pounds at $6 per pound. There were no changes in materials inventories.

Required: D, L & H 9ea. Find the units produced.b. Find the standard labor hours.c. Find the actual labor hours.d. Find the standard quantity of materials allowed.e. Find the actual quantity of materials used.

278. Ralph Inc. had the following variances for the most recent month:Direct Labor Rate Variance $14,560 UDirect Labor Efficiency Variance $ 3,660 UVariable Overhead Spending Variance $12,320 F

Other information included: actual wages paid $105,560; materials purchased $124,860; standards per unit were 2 labor hours at $5 per hour and variable overhead at $6 per hour.

Required: D, L & H 9ea. Find the units produced.b. Find the standard labor hours.

CMA EXAMINATION QUESTIONS Page 102 of 142

MANAGEMENT ADVISORY SERVICES STANDARD COSTS AND VARIANCE ANALYSIS

c. Find the actual labor hours.d. Find the variable overhead efficiency variance.e. Find the actual variable overhead.

279. Gagne Company uses the following equation to budget manufacturing overhead.Manufacturing overhead = $600,000 + $2 per direct labor hour

Gagne has budgeted 300,000 direct labor hours for the year. Actual results were 320,000 direct labor hours and $1,249,000 total manufacturing overhead.

Required: D, L & H 9ea. Find the predetermined overhead rate. b. Find total overhead applied for the year. c. Compute (overapplied underapplied) overhead and circle the correct direction.d. Find the overhead budget variance and state whether favorable or unfavorable.e. Compute the volume variance and state whether favorable or unfavorable

280. Bruno Company uses the following equation to budget manufacturing overhead.Manufacturing overhead = $200,000 + $3 per direct labor hour

Bruno has budgeted 100,000 direct labor hours for the year. Actual results were 90,000 direct labor hours and $457,000 total manufacturing overhead.

Required: D, L & H 9ea. Find the predetermined overhead rate.b. Find total overhead applied for the year.c. Compute (overapplied underapplied) overhead and circle the correct direction.d. Find the overhead budget variance and state whether favorable or unfavorable.e. Compute the volume variance and state whether favorable or unfavorable

281. Tracton Corporation uses a standard costing system in which manufacturing overhead costs are applied to products on the basis of machine time.

REQUIRED: (D) N & Ga. Several numbers and labels have been omitted from the analysis of fixed overhead below.

Supply the missing numbers and labels.

?$ __?____

Flexible Budget Fixed Overhead Cost

$ __?____

Fixed Overhead Cost Applied to Work in Process

302,100 MH x $1.08$ __?____

Budget Variance,$1,880 U

?$ __?____

Total variance, $388 F

b. Suppose that 6 minutes of machine time is standard per unit of production. How many units were actually produced in the situation above?

c. Again suppose that 6 minutes of machine time is standard per unit of production. How many units of production were assumed when the predetermined application rate for fixed overhead was established?

282.. You have just been hired as the controller of the Eastern Division of Global Manufacturing. Performance records for last year are incomplete, with only the following data available:Variable overhead rate $3.00 per direct labor-hourBudgeted fixed overhead $84,800Total actual overhead cost $262,500Fixed overhead budget variance $7,200 unfavorableVariable overhead efficiency variance $15,000 unfavorableActual direct labor-hours worked 55,000 direct labor-hoursDenominator activity level 53,000 direct labor-hoursStandard hours per unit 2 direct labor-hours

REQUIRED: (D) N & GPrepare a complete analysis of manufacturing overhead for the past year. Indicate actual, standard, and denominator activity levels; variable overhead spending and efficiency variances; and fixed overhead budget and volume variances.

FLEXIBLE BUDGET283. Bay City estimates production overhead costs equal to $200,000 + $4X + $7Y, where X is the

number of direct labor hours used and Y is the number of machine hours used. Bay City budgeted 20,000 direct labor hours and 50,000 machine hours for 20X2. Bay City produced 30,000 units in 20X2, each requiring 1 direct labor hour and 2.5 machine hours. Actual production costs were $890,000.

Required: D, L & H 9ea. Calculate the flexible budget allowance for production overhead costs for 20X2. b. Find the amount and direction of the budget variance for 20X2 for production overhead.

(favorable unfavorable) Circle one answer.

284. Westrum estimates production overhead costs equal to $300,000 + $2X, where X is the number of machine hours used. Westrum budgeted 40,000 machine hours for 20X4. Westrum produced 23,000 units in 20X4, each requiring 3 machine hours. Actual production costs were $420,000.

Required: D, L & H 9ea. Calculate the flexible budget allowance for production overhead costs for 20X4.

CMA EXAMINATION QUESTIONS Page 103 of 142

MANAGEMENT ADVISORY SERVICES STANDARD COSTS AND VARIANCE ANALYSIS

b. Find the amount and direction of the budget variance for 20X4 for production overhead. (favorable unfavorable) Circle one answer.

CMA EXAMINATION QUESTIONS Page 104 of 142

MANAGEMENT ADVISORY SERVICES STANDARD COSTS AND VARIANCE ANALYSIS

9 . Normal setup hours = (15,000 / 250) x 5 = 300 hoursOH rate = $14,400 / 300 = $48 per setup hour$14,400 – [(11,250 / 250) x 5 x $48] = $3,600 (U)

18 . Answer (D) is correct. If the design phase is 60% complete, the costs that should have been incurred equal $1,800,000 (60% x $3,000,000). Consequently, the overrun is $700,000 (2,500,000 - $1,800,000). Answer (A) is incorrect because the design phase has incurred an overrun. Answer (B) is incorrect because $500,000 is the excess of budgeted cost over actual cost incurred to date. Answer (C) is incorrect because the design phase has incurred an overrun.

19 . Answer (C) is correct. The company planned to produce 100,000 units at $6 each ($4 variable + $2 fixed cost), or a total of $600,000, consisting of $400,000 of variable costs and $200,000 of fixed costs. Total production was only 80,000 units at a total cost of $515,000. The flexible budget for a production level of 80,000 units includes variable costs of $320,000 ($4 x 80,000 units). Fixed costs would remain at $200,000. Thus, the total flexible budget costs are $520,000. Given that actual costs were only $515,000, the variance is $5,000 favorable. Answer (A) is incorrect because $85,000 favorable is based on a production level of 100,000 units. Answer (B) is incorrect because the variance is favorable. Answer (D) is incorrect because the variance is favorable.

20 . REQUIRED: The flexible-budget operating income.DISCUSSION: (D) A flexible budget is formulated for several different activity levels. Assuming that unit sales price ($100,000 ÷ 10,000 units = $10), variable costs of sales ($60,000 ÷ 10,000 units = $6), and total fixed costs ($30,000) do not change, a flexible budget may be prepared for the actual sales level (12,000 units). Hence, the budgeted contribution margin (sales – variable cost of sales) equals $48,000 [(12,000 units x $10) – (12,000 units x $6)]. The operating income is therefore $18,000 ($48,000 CM - $30,000 FC).Answer (A) is incorrect because $12,000 assumes that all costs are variable. Answer (B) is incorrect because $19,200 is based on actual variable costs. Answer (C) is incorrect because $30,000 is based on actual sales revenue.

22 . 20,000 units ($600,000/25,000) = $480,000

23 . $450,000, given in the static budget

24 . $512,000 - (20,000 x $600,000/25,000) = $32,000 U

25 . Answer (D) is correct. Each unit of finished product contains 2 yards of direct material. However, the problem states that the 20% direct material spoilage is calculated on the quantity of direct material input. Although, not mentioned, the facts on this question infer that the spoilage is normal and should be part of the product’s standard cost. The solutions approach would be to setup the

CMA EXAMINATION QUESTIONS Page 105 of 142

MANAGEMENT ADVISORY SERVICES STANDARD COSTS AND VARIANCE ANALYSIS

following formula:26 . Answer (D) is correct. If 1.5 yards remain in each unit after spoilage of 25% of the direct materials input, the total per unit input must have been 2 yards (1.5 ÷ 75%). The standard unit direct materials

cost is therefore $4.00 (2 yards x $2). Answer (A) is incorrect because the 1.5 yards of good output should be divided (not multiplied) by 75% to determine the standard yards of material per unit. Answer (B) is incorrect because $3.00 is the cost per unit excluding spoilage. Answer (C) is incorrect because $3.75 is found by adding 25% of the materials of the finished product as spoilage and then multiplying by the $2.00 cost per yard [(1.5 x 1.25) x $2].

27 . Answer (C) is correct. The materials purchase price variance equals the quantity purchased multiplied by the difference between the actual price and the standard price, or $135 unfavorable [($.75 - $.72) x 4,500 lbs.]. The variance is unfavorable when the actual price exceeds the standard price. Answer (A) is incorrect because $117 unfavorable is based on the standard input for 1,300 units. Answer (B) is incorrect because $123 unfavorable is based on the actual quantity used. Answer (D) is incorrect because $150 unfavorable is based on the assumption that 5,000 lbs. were purchased.

28 . Variable standard cost per unitDirect materials (15 pounds x $16)$240Direct labor (4 hours x $24) 96Variable overhead (4 hours x $14) 56$392

29 . $170,000 – (10,000 x $16) = $10,000 unfavorable

30 . REQUIREMENT: To determine Lem’s material price variance for May.Answer (B) is correct. The direct materials price variance is the difference between actual unit prices and standard unit prices multiplied by the actual quantity, as shown below.

AQ x AP – AQ x SP = Materials price variance$10,080 – (4,200m x $2.50.) = $420F

The $420 price variance is favorable because the actual purchase price of the material was lower than the standard price. Since the material was purchased for only $2,40 per meter ($10,080 cost 4,200m), Lem saved $0.10 per meter compared to the standard price, for a total price savings of $420 (4,200m x $0.10/m). Note that the standard quantity of materials is ignored in order to isolate these price differences; differences in quantity are addressed by the materials usage variance.

31 . Answer (C) is correct. The direct materials price variance is found by multiplying the difference between the actual price (AP) of direct materials and the standard price (SP) per unit by the actual quantity (AQ).

AQ(AP - SP) = MPV28,000($2.00 - $2.20) = $5,600 favorable

Answer (A) is incorrect because the direct materials price variance is found by multiplying the actual quantity (28,000) times the difference between the AP ($2.00) and the SP ($2.20). Answer (B) is incorrect because the direct materials price variance is found by multiplying the actual quantity (28,000) times the difference between the AP ($2.00) and the SP ($2.20). Answer (D) is incorrect because $2,200 unfavorable is the usage variance.

CMA EXAMINATION QUESTIONS Page 106 of 142

MANAGEMENT ADVISORY SERVICES STANDARD COSTS AND VARIANCE ANALYSIS

32

$240= 1,600 (x - $3.60)1,600 x= $240 + $5,760x= $3.7533 . (6,000 x $16) - $6,000 = $90,000

$90,000/6,000 pounds = $15

34 . 80,000 x 2 x $5 = $800,000$800,000 - $40,000 = $760,000$760,000/$5 = 152,000 pounds

35 . 20,000 x $4 x $6 = $480,000$480,000 + $30,000 = $510,000$510,000/$6 = 85,000

36 . Answer (D) is correct. The company produced 12,000 units of output, each of which required two units of raw materials. Thus, the standard input allowed for raw materials was 24,000 units at a standard cost of $2.50 each. An unfavorable quantity variance signifies that the actual quantity used was greater than the standard input allowed. The materials quantity variance equals the difference between actual and standard quantities, times the standard price per unit. Consequently, since 1,000 ($2,500U ÷ $2.50) additional units were used, the actual total quantity must have been 25,000 units (24,000 standard + 1,000). Answer (A) is incorrect because 12,000 units is the number of units of finished product. Answer (B) is incorrect because 12,500 units assumes that each unit of finished product includes only one unit of raw materials. Answer (C) is incorrect because 23,000 units assumes a favorable quantity variance.

37 . Answer (B) is correct. Given that the company produced 12,000 units with a total standard cost for materials of $60,000, the standard cost must be $5.00 ($60,000 ÷ 12,000 units) per unit of finished product. Because each unit of finished product requires two units of raw materials, the standard unit cost for raw materials must be $2.50. Answer (A) is incorrect because the unit standard cost is $2.50. Answer (C) is incorrect because $3 is the actual cost per unit of raw materials. Answer (D) is incorrect because $5 is the total standard cost of raw materials for each unit of finished product.

38 . x = $5.10 [7,800 - (2,000 x 4)]x = ($1,020) favorable

39 . Answer (C) is correct. The price variance equals actual quantity times the difference between the actual and standard prices. Actual usage and the standard price were 25,000 units and $2.50, respectively. Actual price was $3.00 ($105,000 total cost ÷ 35,000 units purchased). Consequently, the materials price variance is $12,500 unfavorable [($3.00 - $2.50) x 25,000 units].

CMA EXAMINATION QUESTIONS Page 107 of 142

MANAGEMENT ADVISORY SERVICES STANDARD COSTS AND VARIANCE ANALYSIS

40

$3,000= x (30,000 - 29,000)1,000 x= $3,000X= $3Y= $2.80 - $3.00(30,000)Y= ($6,000) favorable41 . $20,000 + 1,000 U - 1,200 F = $19,800

42 . $30,000 – 500 F + 800 U = $30,300

43 . Answer (C) is correct. The direct materials purchase price variance is the difference between the standard price per pound ($4.00) and actual price per pound ($297,000 ÷ 75,000 = $3.96) times the pounds of direct materials purchased for the month. Hence, the price variance is $3,000 F [($4.00 - $3.96) x 75,000 pounds]. The quantity variance is the difference between actual usage of direct materials (70,000 pounds) and standard usage of direct materials for actual output (23,000 actual units produced x 3 pounds = 69,000 pounds) valued at the standard price per pound ($4.00). The quantity variance is therefore $4,000 U [(70,000 - 69,000) x $4]. Answer (A) is incorrect because the price variance is $3,000 F and the quantity variance is $4,000 U. Answer (B) is incorrect because the quantity variance is $4,000 U. Answer (D) is incorrect because the price variance is $3,000 F.

44 . Answer (D) is correct. The standard cost per part is $1.45. The actual cost was $18,000 for 12,000 parts, or $1.50 each. Thus, the price variance is $600 unfavorable [12,000 parts x ($1.50 - $1.45)]. The variance is unfavorable because the actual cost was higher than the standard cost. Answer (A) is incorrect because $450 unfavorable equals the standard quantity needed for the actual output times the $.05 unfavorable price variance per part. Answer (B) is incorrect because the variance is unfavorable, and $450 is the amount of the variance that relates only to the standard input for the actual output. Answer (C) is incorrect because the variance is unfavorable. Furthermore, the variance is based on the quantity purchased, not the quantity consumed. [Note: The materials price variance is sometimes isolated at the time of transfer to production.]

45 . Answer (A) is correct. Standard usage was three parts per radio at $1.45 each. For a production level of 3,000 units, the total materials needed equaled 9,000 parts, but materials actually used totaled 10,000 parts. Thus, the variance is $1,450 unfavorable [$1.45 standard cost per part x (10,000 used - 9,000 standard usage)]. Answer (B) is incorrect because the variance is unfavorable. The actual quantity used exceeded the standard input allowed. Answer (C) is incorrect because $4,350 unfavorable assumes that 12,000 parts were consumed. Answer (D) is incorrect because $4,350 favorable assumes that 12,000 parts were consumed and that the variance is favorable.

46 . Answer (D) is correct. The materials price variance measures the difference between what was actually paid for the goods purchased and the standard amount allowed for the goods purchased. Thus, it equals the difference between actual price and standard price, multiplied by the actual quantity purchased. This question assumes that price variances are isolated at the time of purchase. If they are isolated when the materials are used, the variance is the difference between standard and actual price, times the amount used (not amount purchased). Accordingly, the direct materials purchase price variance is $1,250 F {[($38,750 TC ÷ 25,000 lbs.) - $1.60 per lb.] x 25,000 lbs.}. The variance is favorable because actual price was less than standard price.

CMA EXAMINATION QUESTIONS Page 108 of 142

MANAGEMENT ADVISORY SERVICES STANDARD COSTS AND VARIANCE ANALYSIS

Answer (A) is incorrect because $1,125 favorable results from multiplying the standard direct 48 . Answer (C) is correct. The direct materials price variance measures the difference between what was actually paid for the goods purchased and the standard price allowed for the goods purchased.

Thus, it equals the difference between actual price and standard price, multiplied by the actual quantity purchased. The direct materials price variance is $5,000 favorable {[$2.50 - ($120,000 ÷ 50,000 units)] x 50,000 units}. Answer (A) is incorrect because $7,500 unfavorable equals the difference between actual and budgeted direct materials, multiplied by the standard price per pound of direct materials. Answer (B) is incorrect because zero equals the difference between the budgeted direct materials purchases at standard cost and the actual cost of the direct materials. Answer (D) is incorrect because $5,100 unfavorable equals the difference between the standard and actual costs of direct materials per pound, multiplied by the actual pounds used instead of the actual pounds purchased.

49 . Answer (A) is correct. The direct materials efficiency variance measures the difference between the actual use of inputs and the budgeted quantity of inputs allowed for the activity level achieved. The direct materials efficiency variance equals the standard unit price times the difference between inputs actually used and standard inputs. The direct materials efficiency variance is $500 favorable {[(32,000 x 1.60) - 51,000] x $2.50}. Answer (B) is incorrect because $3,000 favorable uses the amount of direct materials purchased instead of the actual amount used. Answer (C) is incorrect because $7,500 unfavorable equals the difference between the standard amount allowed for the budgeted units to be produced and the amount of direct materials used, multiplied by the standard price per pound. Answer (D) is incorrect because $8,000 unfavorable equals the difference between the budgeted finished units and the actual finished units, multiplied by the standard cost of a finished unit.

50 . $50,400 – (17,000 x $3) = $600 (F)

51 . [10,400 – (2,000 x 5)] x $3 = $1,200 (U)

52 . 6,000/1,000 = 6.00 lbs.53

?. (5,900 – 6,000) x $12 = $1,200 (F)

54 . Answer (B) is correct. The materials mix variance equals the actual total quantity used times the difference between the budgeted weighted-average standard unit cost for the budgeted mix and the budgeted weighted-average standard unit cost for the actual mix. This variance is favorable if the standard weighted-average cost for the actual mix is less than the standard weighted-average cost for the budgeted mix. The standard mix weighted-average standard unit cost is $.225 per liter ($135 standard total cost ÷ 600 liters). The standard cost of the actual quantity used was $18,606 (see below). Thus, the actual mix weighted-average standard unit cost was $.220398 ($18,606 ÷ 84,420 liters used), and the mix variance was $388.50 favorable [($.220398 - $.225) x 84,420 liters].

$.200 x 26,600 = $ 5,320.00 .425 x 12,880 = 5,474.00 .150 x 37,800 = 5,670.00 .300 x 7,140 = 2,142.00$18,606.00Answer (A) is incorrect because $294 favorable is the materials quantity variance. Answer (C) is incorrect because $94.50 unfavorable is the materials yield variance. Answer (D) is incorrect because $219.50 favorable is based on the actual mix of purchases.

55 . Answer (C) is correct. The materials yield variance equals the difference between the actual input and the standard input allowed for the actual output, times the budgeted weighted-average standard cost per input unit at the standard mix. The standard input for the actual output was 84,000 liters (140 batches x 600 liters per batch). The standard mix budgeted weighted-average standard unit cost is

CMA EXAMINATION QUESTIONS Page 109 of 142

MANAGEMENT ADVISORY SERVICES STANDARD COSTS AND VARIANCE ANALYSIS

56 . Materials Mix VarianceMaterialAQSMAQ - SMSP(AQ – SM)SPA30,00035,000(5,000)$1.00$(5,000)B20,00015,0005,000 3.0015,000$10,000 (U)

57 . (40,000 - 36,000) x ($8,000/4,000) = $8,000 (U)

58 . MUV = mix variance + yield variance = $10,000 (U) + $8,000 (U) = $18,000 (U)

59 . Answer (D) is correct. Dividing the actual cost of $60,025 by the 4,900 units used results in an average cost of $12.25 per unit. Answer (A) is incorrect because $12 is the standard cost. Answer (B) is incorrect because $12.01 is based on the budgeted usage. Answer (C) is incorrect because $12.24 equals the $60,000 standard cost for 5,000 units divided by 4,900 units.

60 . Answer (C) is correct. The price variance equals the actual quantity times the difference between the actual price and the standard price. The actual price is $12.25, and the standard price is $12 (given). Thus, the price variance is $1,225 unfavorable [4,900 units x ($12.25 actual - $12.00 standard)]. Answer (A) is incorrect because a price variance exists. The actual price paid was greater than the standard allowed. Answer (B) is incorrect because the variance is unfavorable. Answer (D) is incorrect because the variance is unfavorable.

61 . 1,000 units x $82 = $82,000

62 . ($80 - $82) x 1,000 = $2,000 F

63 . (1,000 – 1,200) x $82 = $16,400 U 64 . 1,200 units x $24 = $28,800

65 . ($25 - $24) x 1,200 = $1,200 U

66 . (1,200 – 1,000) x $24 = $4,800 F

67 . Answer (C) is correct. The standard unit cost is $2.90. The actual cost was $3.00 per unit ($36,000 ÷ 12,000 units). Thus, the unfavorable price variance is $1,200 [($3.00 - $2.90) x 12,000 units]. The variance is unfavorable because the actual cost was higher than the standard cost.

CMA EXAMINATION QUESTIONS Page 110 of 142

MANAGEMENT ADVISORY SERVICES STANDARD COSTS AND VARIANCE ANALYSIS

68 . Answer (A) is correct. At the given production level, 9,000 components (3,000 x 3) are needed. However, 10,000 were used. Consequently, the materials efficiency (quantity or usage) variance was $2,900 unfavorable [(10,000 - 9,000) x $2.90 standard cost per component]. Answer (B) is incorrect because the variance was unfavorable. The actual quantity used was greater than the quantity budgeted. Answer (C) is incorrect because the quantity variance is based on the quantity used during the period (10,000), not the quantity purchased (12,000). Answer (D) is incorrect because the quantity variance is based on the quantity used during the period (10,000), not the quantity purchased (12,000).

69 . Answer (A) is correct. The 3,000 radios require three units each of Part X, a total of 9,000 units. At a standard unit cost of $2.90, the 9,000 units will total $26,100. Answer (B) is incorrect because $27,000 is based on actual unit costs incurred rather than the $2.90 standard cost. Answer (C) is incorrect because $29,000 is based on the actual quantity used rather than the standard quantity. Answer (D) is incorrect because $36,000 is the amount of actual purchases for the month.

70 . REQUIRED: The standard direct labor cost per unit DISCUSSION: (A) The hourly wage worker is $12.50 ($500 40 hrs). The direct labor cost per hour is $15 [($12.50 x 1.0) + benefits equal to 20% of wages]. Consequently, the standard direct labor cost per unit is $30 ($15 x 2 hrs.).Answer (B) is incorrect because the weekly wages and benefits per worker ($500 x 1.2) should be divided by 40 hours per week, not be 50 workers. Answer (C) is incorrect because $15.00 is the DL cost per hour. Two DL hours are required per unit. Answer (D) is incorrect because the weekly wages ad benefits per worker ($500 x 1.2) should be divided by 40 hours per week, not by 50 workers. Furthermore, 2 DL hours are required per unit.

71 . Answer (A) is correct. The hourly wage per worker is $15.00 ($600 ÷ 40 hours). The direct labor cost per hour is $18.00 [$15.00 x (1.0 + benefits equal to 20% of wages)]. Consequently, the standard direct labor cost per unit is $54 ($18 x 3 hours). Answer (B) is incorrect because the weekly wages and benefits per worker ($600 x 1.2) should be divided by 40 hours per week, not by 60 workers to determine the direct labor cost per hour. Answer (C) is incorrect because $30 results from omitting the employee benefits (20% of wages) from the calculation. Answer (D) is incorrect because $18.00 is the DL cost per hour.

72 . Answer (B) is correct. The standard direct labor unit cost equals 3 hours times the cost per DLH. This amount is determined by adding employee benefits to weekly wages and dividing by hours per week.

Weekly wages $245.00Plus benefits (.25 x $245) 61.25$306.25Divided by hours/week÷ 35Cost/DLH $ 8.75DLH/unit x3Unit DL cost $ 26.25Answer (A) is incorrect because $21.00 excludes employee benefits. Answer (C) is incorrect because the standard direct labor unit cost equals 3 hours times the cost per DLH. The cost per DLH equals weekly wages plus employee benefits, divided by productive

hours per week, or $8.75 [($245.00 + 61.25) ÷ 35]. The standard direct labor cost per unit is $26.25 ($8.75 x 3 hours). Answer (D) is incorrect because $36.75 results from dividing weekly wages plus benefits by 25 employees instead of 35 weekly productive hours per employee.

73 . Answer (B) is correct. The total flexible budget direct labor variance equals the difference between cost at actual hours and actual wages and the cost at standard hours and standard wages, or $1,900 U ($48,500 - $46,600).

CMA EXAMINATION QUESTIONS Page 111 of 142

MANAGEMENT ADVISORY SERVICES STANDARD COSTS AND VARIANCE ANALYSIS

materials that should have been used in production [(3 x 7,500) = 22,500 lbs.] times the difference between standard price and actual price [($1.60 - $1.55) = $.05]. Answer (B) is

incorrect because $1,150 favorable results from multiplying the direct materials issued to production (23,000 lbs.) times the difference between standard price and actual price [($1.60 -

74 . Answer (D) is correct. The standard cost of materials for 8,500 units is $127,500 (8,500 x $15). Thus, no variance arose with respect to materials. Because labor for 9,000 units was budgeted at $81,000, the unit labor cost is $9. Thus, the labor budget for 8,500 units is $76,500, and the total labor variance is $1,275 ($77,775 - $76,500). Because the actual cost is greater than the budgeted amounts, the $1,275 variance is unfavorable. Given that the actual time per unit (45 minutes) was the same as that budgeted, no labor efficiency variance was incurred. Hence, the entire $1,275 unfavorable variance must be attributable to the labor rate (or price) variance. Answer (A) is incorrect because there is no materials variance; the actual cost was equal to the budgeted cost for materials. Answer (B) is incorrect because no labor efficiency variance occurred. Budgeted hours were identical to actual hours for 8,500 units. Answer (C) is incorrect because no labor efficiency variance occurred. Budgeted hours were identical to actual hours for 8,500 units.

75 . 14,000 x $40 = $560,000$560,000 - $70,000 = $490,000$490,000/14,000 = $35

76 . 20,000 x $5 = $100,000$100,000 + $4,000 = $104,000$104,000/20,000 = $5.20

77 . REQUIRED: The direct labor payroll for March.DISCUSSION: (A) When the actual direct labor rate is unknown, the total direct labor payroll is found by multiplying the actual hours by the standard rate, then subtracting the favorable labor variance.

(40,000 x $6.30) – ($8,400) = $243,600Answer (B) is incorrect because $252,000 equals actual hours times the standard rate. Answer (C) is incorrect because $264,600 equals standard hours times the standard rate. Answer (D) is incorrect because the favorable rate variance should be subtracted from the payroll calculated at the standard rate for the actual hours.

78 . Answer (A) is correct. When the actual direct labor rate is unknown, the total direct labor payroll can be found by multiplying the actual hours by the standard rate, then subtracting the favorable labor variance. (32,000 x $5.04) - $6,720 = $154,560Answer (B) is incorrect because the total $6,720 DL rate variance, not the DL rate variance per standard hour times the actual DL hours, should be subtracted in the calculation. Answer (C) is incorrect because the total $6,720 DL rate variance, not the DL rate variance per standard hour times the actual DL hours, should be used in calculating the payroll. Furthermore, a favorable DL rate variance should be subtracted from, not added to, the standard DL costs allowed for hours worked. Answer (D) is incorrect because the $6,720 DL rate variance is favorable, and should therefore be subtracted from, not added to, the standard payroll for the hours worked.

CMA EXAMINATION QUESTIONS Page 112 of 142

MANAGEMENT ADVISORY SERVICES STANDARD COSTS AND VARIANCE ANALYSIS

79 . REQUIRED: The standard direct labor rate for the month.DISCUSSION: (D) The labor rate variance equals actual hours times the difference between the actual and standard rates. The variance is favorable, so the standard rate must exceed the actual rate.

10,000 (SR - $7.50) = $5,500 SR = $7.50 + ($5,500 10,000) SR = $8.05Answer (A) is incorrect because $6.95 treats the $0.55 variance per unit as unfavorable. Answer (B) is incorrect because actual hours, not standard hours, are used to determined the SR. Furthermore, the favorable variance should be added, not subtracted, in calculating the standard rate. Answer (C) is incorrect because actual hours, not standard hours, should be used in determining the standard rate.

80 . Answer (D) is correct. The direct labor rate variance equals actual hours times the difference between the actual and standard rates. When the variance is favorable, the standard rate exceeds the actual rate and the following equation is used:

AH(SR - AR) = favorable rate variance10,000(SR - $8.25) = $5,600 F SR - $8.25 = $.56 SR = $8.81

Answer (A) is incorrect because $7.69 results from treating the $.56 variance per unit as unfavorable and subtracting it from the AR of $8.25. Answer (B) is incorrect because actual hours, not standard hours, are used to determine the SR. Furthermore, the favorable variance should be added, not subtracted, in calculating the standard rate. Answer (C) is incorrect because $8.25 is the actual rate.

81 . REQUIRED: The standard direct labor rate for the month.DISCUSSION: (D) The labor rate variance equals actual hours times the difference between the actual and standard rates. The variance is unfavorable, so the actual rate must exceed the standard rate.

10,000 (SR - P7.50) = – P5,000 SR = P7.50 – (P5,000 10,000) SR = P7.00Answer (A) is incorrect because $6.95 treats the $0.55 variance per unit as unfavorable. Answer (B) is incorrect because actual hours, not standard hours, are used to determined the SR. Furthermore, the favorable variance should be added, not subtracted, in calculating the standard rate. Answer (C) is incorrect because actual hours, not standard hours, should be used in determining the standard rate.

82 . 53. REQUIRED: The amount of an unfavorable direct labor efficiency variance.DISCUSSION: (B) The direct labor efficiency (quantity) variance equals standard amounts of labor hours. Thus, the direct labor efficiency variance is $1,200 {$12 standard rate x [4,100 actual hours – (4 standard hours x 1,000 units)]}. The variance is unfavorable because more labor hours were used than the standard.

CMA EXAMINATION QUESTIONS Page 113 of 142

MANAGEMENT ADVISORY SERVICES STANDARD COSTS AND VARIANCE ANALYSIS

85 . Answer (D) is correct. The direct labor efficiency (quantity) variance equals standard price times the difference between actual and standard amounts of labor hours. The standard amount for the actual output is 6,500 direct labor hours (1.25 DLH x 5,200 units). The variance is $1,200 U [(6,500 standard hours - 6,600 actual hours) x $12]. Answer (A) is incorrect because $4,200 equals $12 times the difference between actual hours (6,600) and the hours budgeted for planned output (5,000 x 1.25 = 6,250). Answer (B) is incorrect because $3,000 equals $12 times 6,500 standard hours for the actual output minus 6,250 standard hours for the budgeted output. Answer (C) is incorrect because $2,220 equals actual cost minus budgeted cost.

86 . Answer (A) is correct. The labor efficiency variance is $980 ($9,800 - $8,820). It is the difference between actual and standard hours multiplied by the standard labor rate. Answer (B) is incorrect because the labor rate variance is $200. It is the difference between the actual and standard rates time the actual hours. Answer (C) is incorrect because the volume variance is the difference between budgeted fixed overhead and the amount applied based on the standard input allowed for the actual output. Answer (D) is incorrect because the term "spending variance" is usually applied to overhead variances.

87 . REQUIRED: The actual hours worked.DISCUSSION: (D) The standard hours allowed equaled 2,000, and the labor efficiency variance was $1,600 unfavorable; i.e., actual hours exceeded standard hours. The labor efficiency variance equals the standard rate ($8 per hour) times the excess hours. Given that the variance is $1,600, 200 excess hours ($1,600 $8) must have been worked. Thus, 2,200 actual hours (2,000 standard + 200 excess) were worked.Answer (A) is incorrect because the 200-hour difference between AH and SH should be added to, not subtracted from, the standard hours allowed. Answer (B) is incorrect because the difference between AH and SH must be determined using the standard rate per hour. The efficiency variance was also incorrectly treated as favorable and subtracted from the SH. Answer (C) is incorrect because the difference between AH and SH must be determined using the standard rate per hour.

88 . Answer (D) is correct. The standard hours allowed equaled 3,000, and the labor efficiency variance was $1,870 unfavorable; that is, actual hours exceeded standard hours. The labor efficiency variance equals the standard rate ($8.50) times the excess hours. Given that the variance is $1,870, 220 excess hours ($1,870 ÷ $8.50) must have been worked. Thus, 3,220 actual hours (3,000 standard + 220 excess) were worked. Answer (A) is incorrect because the 220 hour difference between AH and SH should not be subtracted from the standard hours allowed. Answer (B) is incorrect because the excess hours should be determined using the standard, not the actual, rate per hour, and the result should be added to, not incorrectly subtracted from, standard hours allowed. Answer (C) is incorrect because excess hours should be determined using the standard, not the actual, rate per hour.

89

$4,200= $3.75 (x - 10,000)$3.75 x= $4,200 + $37,500X= 11,12090 . Answer (B) is correct. The direct labor rate variance is determined by multiplying the difference between the actual and standard rates by the actual hours. The standard rate equals the direct labor

efficiency variance divided by the difference between the actual and standard hours. The actual rate equals the total direct labor payroll divided by the actual hours. $3,840 ÷ (41,200 - 42,000) = $4.80 SR $247,200 ÷ 41,200 = 6.00 AR $1.20 diff. x41,200 AH DL rate variance = $49,440 UAnswer (A) is incorrect because $44,496 was determined using an

actual rate of $5.88. Answer (C) is incorrect because the variance is unfavorable. Answer (D) is incorrect because the $50,400 results from multiplying $1.20 by standard hours (42,000). CMA EXAMINATION QUESTIONS Page 114 of 142

MANAGEMENT ADVISORY SERVICES STANDARD COSTS AND VARIANCE ANALYSIS

Answer (A) is incorrect because $1,220 uses the actual labor rate. Answer (C) is incorrect 92 . Answer (C) is correct. The direct labor efficiency variance equals actual quantity minus standard quantity, times the standard price. The standard quantity is the amount of standard labor hours

required for the actual good output achieved. Actual labor hours equaled 3,200, standard hours were 2,500 (1,000 units of output x 2.5 hours), and the standard direct labor price was $8. Hence, the labor efficiency variance was $5,600 U [(3,200 actual hours - 2,500 standard hours) x $8]. Answer (A) is incorrect because $2,240 unfavorable is the direct labor rate variance. Answer (B) is incorrect because the variance is unfavorable. Answer (D) is incorrect because $6,090 equals the difference between actual hours and standard hours for the actual output (3,200 - 2,500 = 700), times the actual direct labor cost per hour ($8.70).

93 . Answer (D) is correct. The direct labor rate variance equals actual DLH times the difference between the actual and standard direct labor rates, or $250 unfavorable {6,250 actual DLH x [($75,250 actual costs ÷ 6,250 actual DLH) - $12.00 standard rate]}. Answer (A) is incorrect because $10 unfavorable equals the difference between the planned and actual DLH times the difference between the standard and actual rates. Answer (B) is incorrect because $240 unfavorable equals the planned DLH times the difference between the standard and actual rates. Answer (C) is incorrect because $248 unfavorable equals the DLH allowed for the actual output times the difference between the standard and actual rates.

94 . Answer (A) is correct. The direct labor efficiency variance equals the standard direct labor rate times the difference between the actual DLH and the standard DLH allowed for the actual output, or $600 unfavorable {$12 x [6,250 actual DLH - (.4 DLH x 15,500 pounds actually produced)]}. Answer (B) is incorrect because $602 unfavorable is based on the actual direct labor rate. Answer (C) is incorrect because $2,400 unfavorable equals the standard direct labor rate times the difference between the DLH allowed for the planned output and the DLH allowed for the actual output. Answer (D) is incorrect because $3,000 unfavorable equals the standard direct labor rate times the difference between the DLH allowed for the planned output and the actual DLH.

95 . $12,000/($10 - $9) = 12,000 actual hours(12,000 x $9) – (10,000 x $10) = $8,000 (U)

96 . $12,000/($10 – $9) = 12,000 hours97

?. 107.Answer (A) is correct. The labor yield variance is the portion of the labor efficiency variance attributable to the difference between actual hours and the standard hours allowed. Total hours worked were 1,575 (550 + 650 + 375), and standard hours allowed equaled 1,500 (500 + 500 + 500). The 75-hour difference is multiplied by the weighted-average standard labor rate, which equals $6.67 {[(500 x $8) + (500 x $7) + (500 x $5)] ÷ 1,500 standard DLH}. Thus, the variance is $500 unfavorable ($6.67 x 75). Answer (B) is incorrect because the labor yield variance is $500. Answer (C) is incorrect because the difference between the 1,575 AH (550 + 650 + 375) and the 1,500 SH (500 + 500 + 500) must be multiplied by the $6.67 weighted-average SR {[(500 x $8) + (500 x $7) + (500 x $5)] ÷ 1,500 standard DLH}. Answer (D) is incorrect because $1,500 is three times the actual labor yield variance of $500.

CMA EXAMINATION QUESTIONS Page 115 of 142

MANAGEMENT ADVISORY SERVICES STANDARD COSTS AND VARIANCE ANALYSIS

98 . Answer (A) is correct. The labor mix variance is the sum of the products of the difference between actual and standard hours for each class of labor times the difference between the budgeted standard rate for that class of labor and the weighted-average standard labor rate.

ClassStandard HoursActual HoursVarianceBud. Std.-W.A. Std.

ProductIII500550- 50+$1.33$ 66.67 UII 500650- 150+ .33 50.00 UI 500375+ 125- 1.67 208.33 U$325.00 UAnswer (B) is incorrect because $66.67 is the labor variance for labor Class III only. Answer (C) is incorrect because the labor mix variance is the sum of the products of the difference between actual and standard hours for each class of labor times the difference between the average

standard labor rate for that class of labor. Answer (D) is incorrect because $50.00 is the labor variance for labor Class II only.

99 . Answer (A) is correct. The labor yield variance is the difference between actual and budgeted inputs, times the budgeted weighted-average rate for the planned mix. Total hours worked were 1,575 (550 + 650 + 375), standard hours allowed equaled 1,500 (500 + 500 + 500), and the budgeted weighted-average rate for the planned mix was $6.67 {[(500 x $8) + (500 x $7) + (500 x $5)] ÷ 1,500 standard DLH}. Thus, the variance is $500 unfavorable ($6.67 x 75). Answer (B) is incorrect because $320 is the labor mix variance. Answer (C) is incorrect because $820 is the labor efficiency variance. Answer (D) is incorrect because $515 is based on the budgeted weighted-average rate for the actual mix.

100 . Answer (B) is correct. The labor mix variance is the difference between the budgeted weighted-average rates for the actual and planned mixes, times the actual labor inputs. The budgeted weighted-average rate for the planned mix is $6.67 (see preceding question). The budgeted weighted-average rate for the actual mix is $6.873 [(550 x $8) + (650 x $7) + (375 x $5) ÷ 1,575 actual DLH]. Thus, the mix variance is $320 [($6.873 - 6.67) x 1,575]. Answer (A) is incorrect because $50.00 is the variance for labor class II only [($7 - $6.67) x (650 DLH - 500 DLH)]. Answer (C) is incorrect because $66.67 is the variance for labor class III only [($8 - $6.67) x (550 DLH - 500 DLH)]. Answer (D) is incorrect because $500 is the labor yield variance.

101 . Answer (C) is correct. Labor mix and labor yield variances are the components of the total labor efficiency variance. For example, if the labor yield variance was $500 U and the labor mix variance was $320 U, the total labor efficiency variance would be $820 U. Answer (A) is incorrect because the total labor variance equals the labor efficiency and the labor rate variances. Answer (B) is incorrect because the labor rate variance is the variance of price of the labor. Answer (D) is incorrect because the efficiency variance is not labor variances.

102 . Labor Mix VarianceLabor typeAQSMAQ - SMSP(AQ – SM)SPMixing4,5005,000(500)$10.00$(5,000)Finishing3,0002,500500 5.00 2,500 $2,500 (F)

103 . (40,000 - 36,000) x ($6,250/4,000) = $6,250 (U)

104 . LEV = mix variance + yield varianceCMA EXAMINATION QUESTIONS Page 116 of 142

MANAGEMENT ADVISORY SERVICES STANDARD COSTS AND VARIANCE ANALYSIS

= $2,500 (F) + $6,250 (U) = $3,750 (U)106 . Answer (C) is correct.Actual hours x standard rates (8,150 x 12) + (4,300 x 8) + (4,400 x 6)158,600Actual hours x weighted average standard rates (16,850 x 9.329)

157,194Mix Variance1,406Answer (A) is incorrect because $2,205 results from calculating the budgeted weighted-average standard rate for the actual mix using actual rates. Answer (B) is incorrect because $2,205 results from calculating the budgeted weighted-average standard rate for the actual mix using actual rates. Moreover, the variance is unfavorable. Answer (D) is incorrect because the

variance was unfavorable.

107 . Answer (B) is correct. The labor yield variance is the difference between actual and budgeted inputs, multiplied by the budgeted weighted-average standard rate for the standard mix. Total actual hours worked were 16,850 (8,150 + 4,300 + 4,400), and standard hours allowed equaled 17,050 (7,920 + 4,620 + 4,510). The budgeted weighted-average standard rate for the standard mix was $9.33 {[(7,920 x $12) + (4,620 x $8) + (4,510 x $6)] ÷ 17,050}. Hence, the yield variance is $1,866 favorable [$9.33 x (16,850 - 17,050)]. Answer (A) is incorrect because $1,908 favorable results from multiplying by the weighted-average actual hourly rate for the actual mix instead of by the budgeted weighted-average standard rate for the standard mix. Answer (C) is incorrect because $1,733 favorable results from multiplying by the simple average of the standard hourly rates. Answer (D) is incorrect because $460 favorable results from multiplying each labor class's hour variance by its respective standard hourly rate.

108 . 1,000 units x 24 hours x 30 days = 720,000 units

109 . 800 units x 24 hours x 27 days = 518,400 units

110 . Answer (B) is correct. Standard costs are predetermined target costs which should be attainable under efficient conditions. Currently attainable standards should be achieved under efficient operating conditions. Therefore, engineering estimates based on attainable performance would provide the best basis for Flint in establishing standard hours allowed.

111 . Answer (C) is correct. The maintenance cost is a mixed cost containing both fixed and variable elements. To calculate the monthly total fixed costs, divide the annual amount by 12.Monthly fixed maintenance costs: $6,000 ÷ 12 = $500Variable maintenance costs: 30,000 x $0.70/hour =21,000Total maintenance costs = $21,500Answer (A) is incorrect because $20,500 maintenance cost is arrived at by subtracting the $500 fixed costs per month from the $21,000 variable costs. The $500 should be added to the $21,000. Answer (B) is incorrect because $21,000 only includes the variable maintenance costs. Answer (D) is incorrect because $27,000 is determined by adding annual fixed maintenance costs of $6,000 to variable maintenance costs for the month of $21,000. Fixed maintenance costs for the month of $500 should be added instead.

112 . Answer (C) is correct. The formula is for an annual period. Thus, the first step is to divide the $7,200 of fixed costs by 12 months to arrive at monthly fixed costs of $600. Variable costs will be $.60 per unit, or $12,000 for 20,000 units. The total flexible budget amount is therefore $12,600 ($600 + $12,000).

CMA EXAMINATION QUESTIONS Page 117 of 142

MANAGEMENT ADVISORY SERVICES STANDARD COSTS AND VARIANCE ANALYSIS

113 . Answer (C) is correct. A flexible budget consists of a fixed cost component and a variable cost component. The fixed cost component can be expected to remain constant throughout the budget's relevant range. The variable cost component, however, will change at a constant rate within the budget's range. The increase in budgeted cost of $1,200 ($21,000 - $19,800) per 1,000 units of production can therefore be attributed to the variable cost component. The flexible budget for 12,000 units of production will be $23,400 [the cost for 10,000 ($21,000) + $2,400 (2 x $1,200)]. Answer (A) is incorrect because the flexible budget for 12,000 units should be computed by determining the variable cost per unit of $1.20 [($21,000 - $19,800) ÷ 1,000] and the total fixed costs of $9,000 [$21,000 - ($1.20 x 10,000)]. These costs can then be used to determine the total cost of using 12,000 units of electricity [$9,000 FC + (12,000 x $1.20)]. Answer (B) is incorrect because the flexible budget for 12,000 units should be computed by determining the variable cost per unit of $1.20 [($21,000 - $19,800) ÷ 1,000] and the total fixed costs of $9,000 [$21,000 - ($1.20 x 10,000)]. These costs can then be used to determine the total cost of using 12,000 units of electricity [$9,000 FC + (12,000 x $1.20)]. Answer (D) is incorrect because $22,200 is arrived at by subtracting the increase in budgeted cost of $1,200. The flexible budget for 12,000 units of production will be $23,400 [the cost for 10,000 ($21,000) + $2,400 (2 x $1,200).

114 . Answer (D) is correct. The flexible budget formula is Shipping costs = $16,000 + ($.50 x lbs. shipped)Therefore, to determine the flexible budget amount, multiply the actual pounds shipped (12,300) times the standard cost ($.50) to arrive at a total expected variable cost of $6,150. Adding the variable cost to $16,000 of fixed cost produces a budget total of $22,150. Answer (A) is incorrect because $20,680 is based on the variation in the actual number of sales orders from those planned, rather than on pounds shipped. Answer (B) is incorrect because $20,920 is based on the number of shipments, not the number of pounds shipped. Answer (C) is incorrect because $20,800 is based on planned pounds shipped of 9,600, not actual pounds shipped of 12,300.

115 . Answer (D) is correct. According to the flexible budget formula, total fixed costs and unit variable cost are expected to be $18,000 and $.60, respectively. Thus, budgeted variable costs for the actual output should be $7,500 (12,500 lbs. x $.60). Adding the variable costs to the $18,000 of fixed costs produces a budget total of $25,500. Answer (A) is incorrect because $18,000 equals the fixed costs. Answer (B) is incorrect because $18,492 is based on the number of shipments. Answer (C) is incorrect because $23,760 is based on planned pounds shipped of 9,600.

116 . Answer (D) is correct. Given actual overhead of $39,500 and underapplied overhead of $1,500, overhead applied was $38,000 ($39,500 - $1,500). Overhead is applied at the rate of $4 per unit ($40,000 budgeted overhead ÷ 10,000 budgeted units). Accordingly, 9,500 units were produced ($38,000 applied overhead ÷ $4 per unit application rate). Answer (A) is incorrect because 10,250 would have been produced if overhead had been overapplied by $1,500 [($39,500 + $1,500) ÷ $4]. Answer (B) is incorrect because 10,000 is the result of dividing budgeted, not applied, overhead by the application rate. Answer (C) is incorrect because 9,875 units would have been produced if $39,500 had been the amount of applied overhead.

117 . REQUIRED: The applied factory O/H for the month.DISCUSSION: (A) The applied factory O/H equals the standard direct hours allowed for actual production multiplied by the total standard O/H rate per hour.

39,000($4 VOH + $1 FOH) = $195,000Answer (B) is incorrect because $197,000 includes the $2,000 favorable O/H variance. This variance should not be added to the $195,000 applied factory O/H. Answer (C) is incorrect because the actual DLH worked were used to determine the applied FO when the standard DLH allowed for actual production should have been used. Answer (D) is incorrect because the actual DLH worked were used instead of the standard DLH allowed. furthermore, the $2,000 favorable O/H variance should not be included.

CMA EXAMINATION QUESTIONS Page 118 of 142

MANAGEMENT ADVISORY SERVICES STANDARD COSTS AND VARIANCE ANALYSIS

120 . Answer (C) is correct. The total overhead variance is the difference between the actual overhead and applied (absorbed) overhead. Given that neither fixed nor variable overhead differed from budgeted amounts, the only variance was caused by under- or overabsorption of fixed overhead. The variable overhead rate does not vary with the capacity. The fixed overhead rate at 90% capacity is

Given that the actual capacity achieved was 75%, and that 30,000 standard hours were allowed, $120,000 (30,000 x $4.00) of fixed overhead was applied. Thus, $24,000 ($144,000 FOH - $120,000) was underabsorbed. Answer (A) is incorrect because $28,500 assumes a fixed overhead application rate of $5.75. Answer (B) is incorrect because $28,500 assumes a fixed overhead application rate of $5.75. Answer (D) is incorrect because the overhead variance for the year is $24,000 underabsorbed, not overabsorbed.

121 . REQUIRED: The factory O/H variance for the year.DISCUSSION: (D) The total O/H variance is computed by determining the difference between the actual O/H and applied O/h. Given that actual factory O/H was equal to the budgeted amount for the attained capacity, the only variance was caused by under- or overabsorption of fixed O/H. The fixed O/H rate at the 90% activity level is $4 ($108,000 fixed O/H 27,000 DLH). Given that the actual activity level achieved was 80% and that 24,000 standard hours were allowed, $96,000 (24,000 x $4.00) of fixed O/H was absorbed. Hence, underabsorbed (underapplied) fixed O/H was $12,000 ($108,000 - $96,000).Answers (A) and (B) are incorrect because $96,000 of O/H was absorbed. Answer (C) is incorrect because the O/H variance for the year is $12,000 underabsorbed, not overabsorbed.

122 . Answer (A) is correct. To determine Gage’s actual manufacturing overhead from the information given, total actual manufacturing costs must first be computed.Standard manufacturing cost$100,000Excess of actual manufacturing cost over standard costs20,000Total actual costs$120,000Since prime costs consist of direct materials and direct manufacturing labor, these costs are deducted from total actual costs to derive the portion that are overhead costs. Ordinarily, normal spoilage is not added to actual manufacturing overhead. The cost of normal spoilage should be added to the cost of good units produced. Freight out is also excluded because it is a selling expense.

Total actual costs$120,000Prime costs(80,000)Actual manufacturing overhead$40,000123 . Answer (C) is correct. The total O/H equation is y = $132,000 + $13(DMH). This equation is derived by summing individual O/H items. The fixed portion needs to be converted to a rate by dividing it

by normal capacity. Thus, the fixed O/H rate is $1.32 ($132,000 ÷ 100,000). To calculate the total O/H rate, the fixed rate is added to the variable rate. Hence, the total O/H rate per DMH is $14.32 ($1.32 + $13.00). Answer (A) is incorrect because $13.00 per DMH is the variable O/H rate per machine hour. The standard O/H rate also includes the fixed O/H rate. Answer (B) is incorrect because $1.32 per DMH is the fixed O/H rate per machine hour. The standard O/H rate per machine hour includes the variable rate per DMH. Answer (D) is incorrect because $13.76 per DMH is the actual total O/H rate (total actual cost ÷ actual hours).

124 . Answer (D) is correct. In a standard-cost system, O/H is applied using the standard activity allowed for actual production. The standard activity allowed is the standard activity per equivalent unit times the actual production, or 94,000 hours (4 DMH x 23,500). The O/H applied is $1,346,080 (94,000 x $14.32).

CMA EXAMINATION QUESTIONS Page 119 of 142

MANAGEMENT ADVISORY SERVICES STANDARD COSTS AND VARIANCE ANALYSIS

127 . REQUIRED: The controllable (budget) variance.DISCUSSION: (A) In two-way analysis, the budget (controllable) variance is the total factory O/H variance not attributable to the volume variance. The total factory O/H variance equals the difference between actual total factory O/H and the factory O/H applied based on the standard input allowed for the actual output, or $10,500 unfavorable ([$147,000 actual – ($6.50 x 21,000 DLH) applied]. The volume variance (budgeted fixed factory O/H – amount applied) is $13,500 unfavorable {($108,000 budgeted) – [($108,000 24,000 DLH) x 21,000 DLH]}. Thus, the controllable variance must be $3,000 favorable ($13,500 unfavorable – $10,500 unfavorable).Answer (B) is incorrect because the $13,500 unfavorable is the volume variance. Answer (C) is incorrect because $9,000 favorable equals the difference between total actual factory O/H and factory O/H budgeted at normal capacity. Answer (D) is incorrect because $10,500 unfavorable equals the total factory O/H variance.

128 . Answer (A) is correct. Two-way analysis computes only two overhead variances: the budget (controllable) variance and the volume variance. The product of the variable overhead rate and the standard direct labor hours allowed for capacity attained is the budgeted variable overhead. The budgeted fixed overhead is then added to the budgeted variable overhead, giving the total budgeted overhead for the standard input allowed for actual output. The difference between the actual overhead and budgeted total overhead is the budget (controllable) variance. Actual overhead equals $220,500. Budgeted variable overhead equals $2 per hour ($72,000 ÷ 36,000 DLH). Thus, budgeted variable overhead based on standard hours allowed equals $63,000 ($2 x 31,500 DLH). The total budgeted overhead is $225,000 ($63,000 + $162,000 FOH). The variance is $4,500 favorable ($225,000 - $220,500) because budgeted overhead exceeds actual overhead. Answer (B) is incorrect because $7,500 is based on the DLH worked (33,000). Answer (C) is incorrect because $7,500 is based on the DLH worked (33,000). Answer (D) is incorrect because $13,500 is based on normal capacity of 36,000 DLH.

129 . $40,000 1,600 = $25

130

Actual factory overhead$47,800Budget allowance:Variable ($5 x 2,000)10,000Fixed (80% x $40,000)32,00042,000Spending variance$5,800131

Actual factory overhead$575,000Budget allowance:Variable factory overhead (52,000 x $6)$312,000Budgeted fixed overhead 250,000 562,000Controllable variance$ 13,000unfavorable132

Budget allowance based on standard hours allowed [(52,000 x $6) + $250,000]CMA EXAMINATION QUESTIONS Page 120 of 142

MANAGEMENT ADVISORY SERVICES STANDARD COSTS AND VARIANCE ANALYSIS

133 . Answer (C) is correct. The total O/H equation is y = $132,000 + $13(DMH). This equation is derived by summing individual O/H items. The fixed portion needs to be converted to a rate by dividing it by normal capacity. Thus, the fixed O/H rate is $1.32 ($132,000 ÷ 100,000). To calculate the total O/H rate, the fixed rate is added to the variable rate. Hence, the total O/H rate per DMH is $14.32 ($1.32 + $13.00). Answer (A) is incorrect because $13.00 per DMH is the variable O/H rate per machine hour. The standard O/H rate also includes the fixed O/H rate. Answer (B) is incorrect because $1.32 per DMH is the fixed O/H rate per machine hour. The standard O/H rate per machine hour includes the variable rate per DMH. Answer (D) is incorrect because $13.76 per DMH is the actual total O/H rate (total actual cost ÷ actual hours).

134 . Answer (D) is correct. In a standard-cost system, O/H is applied using the standard activity allowed for actual production. The standard activity allowed is the standard activity per equivalent unit times the actual production, or 94,000 hours (4 DMH x 23,500). The O/H applied is $1,346,080 (94,000 x $14.32). Answer (A) is incorrect because the standard O/H rate is multiplied by the 94,000 DMH allowed (not 98,700 actual DMH) for the 23,500 equivalent units of production. Answer (B) is incorrect because $1,432,000 is for the original 100,000 DMH budgeted, not the 94,000 DMH budgeted for the production of 23,500 units. Answer (C) is incorrect because $1,358,250 is the total actual O/H.

135 . Answer (A) is correct. The total O/H variance is the over- or underapplied O/H, that is, the difference between applied O/H and the actual O/H. The applied O/H was determined to be $1,346,080. The actual O/H is $1,358,250 ($133,250 + $1,225,000). Consequently, the amount of underapplied O/H is $12,170 U ($1,358,250 - $1,346,080). Answer (B) is incorrect because the applied O/H is $1,346,080, which is based on the budgeted DMH for the equivalent units of production, not on the actual DMH. Answer (C) is incorrect because the applied O/H is $1,346,080, which is based on the budgeted DMH for the equivalent units of production, not on the actual DMH. Furthermore, because the actual O/H is greater than the O/H applied, the underapplied O/H results in an unfavorable variance. Answer (D) is incorrect because $73,750 favorable assumes that standard input for the actual output was 100,000 DMH and that overhead applied was therefore $1,432,000.

136 . Answer (A) is correct. The spending variance is the difference between the actual total overhead and the sum of budgeted fixed overhead and the variable overhead budgeted for the actual input. The total actual overhead is $140,000 ($106,250 + $33,750). The sum of budgeted fixed overhead and variable overhead budgeted for the actual input is $131,250 ($100,000 + $31,250). Thus, the total spending variance is $8,750 ($140,000 - $131,250). The variance is unfavorable because the actual overhead exceeds the budgeted overhead. Answer (B) is incorrect because $6,250 is the difference between the actual and budgeted variable overhead. Answer (C) is incorrect because $3,750 is the difference between the fixed and variable components of the variance. Answer (D) is incorrect because $2,500 is the difference between the actual and budgeted fixed overhead.

137 . REQUIRED: The spending variance assuming a three-way variance analysis.DISCUSSION: (A) The spending variance is the difference between the actual total factory O/H and the budgeted amount for the actual input.

Budgeted $7,000 + (3,500 x $2.50)$15,750 .Actual (15,000) .$ 750 FAnswer (B) is incorrect because the spending variance is favorable. Answer (C) is incorrect because $950 favorable is based on actual fixed factory O/H. Answer (D) is incorrect because $200 unfavorable is the fixed factory O/H spending (budget) variance.

CMA EXAMINATION QUESTIONS Page 121 of 142

MANAGEMENT ADVISORY SERVICES STANDARD COSTS AND VARIANCE ANALYSIS

140 . Answer (B) is correct. The variable overhead efficiency variance equals the standard variable overhead rate times the difference between the actual input and the standard input allowed for the actual output. The standard rate for variable overhead is $2 per direct labor hour. Actual direct labor hours are 24,500. Standard labor hours are 24,000 (8,000 units x 3 hours per unit). Thus, the variable overhead efficiency variance is $1,000 [2 x (24,500 - 24,000)]. The variance is unfavorable because actual hours exceeded standard hours. Answer (A) is incorrect because an unfavorable variable overhead efficiency variance exists. Answer (C) is incorrect because $2,000 is the total variable overhead variance (actual overhead minus the overhead rate applied to the standard hours). Answer (D) is incorrect because $3,000 is the difference between actual variable overhead and the product of the standard rate and the actual input (the variable overhead spending variance). This variance is favorable.

141

Actual factory overhead$178,500Budget allowance:Variable for actual hours(121,000 x $.50)$ 60,500Fixed 110,000170,500Spending variance$ 8,000unfavorable142

Budget allowance for actual hours [(121,000 x $.50) + $110,000]

$170,500Budget allowance for standard hours:Variable (130,000 x $.50)$ 65,000Fixed 110,000 175,000 Variable efficiency variance$(4,500)favorable143 . Answer (B) is correct. The cost of indirect materials for 144,000 units was expected to be $180,000. Consequently, the unit cost of indirect materials is $1.25 ($180,000 ÷ 144,000). Multiplying the

$1.25 unit cost times the 10,800 units produced results in an expected total indirect materials cost of $13,500. Answer (A) is incorrect because the flexible budget amount for indirect materials is $13,500. Answer (C) is incorrect because the flexible budget amount for indirect materials is $13,500. Answer (D) is incorrect because the flexible budget amount for indirect materials is $13,500.

144

Variable budget allowance for actual hours (2,100 x $3)$6,300Variable budget allowance for standard hours ($3 x 1,000 x 2) 6,000$300unfavorable145 . Answer (D) is correct. The variable delivery expense should total $166,400 given sales of 52,000 units ($160,000 ÷ 50,000 units = $3.20 per unit). Thus, the variance is $3,400 ($166,400 - $163,000).

It is favorable since the actual cost is less than that budgeted. Answer (A) is incorrect because the total variable delivery expense will increase as production increases. Answer (B) is incorrect because the variance is favorable. Answer (C) is incorrect because the total variable delivery expense will increase as production increases.

146 . Answer (B) is correct. The $144,000 annual amount equals $12,000 per month. Since volume is expected to be 5,000 units per month, and the $12,000 is considered a variable cost, budgeted cost per unit is $2.40 ($12,000 ÷ 5,000 units). If 4,500 units are produced, the total variable costs should be $10,800 (4,500 units x $2.40). Subtracting the $10,100 of actual costs from the budgeted figure results in a favorable variance of $700. Answer (A) is incorrect because $1,900 is calculated using 5,000 units produced instead of the actual 4,500. Answer (C) is incorrect because $1,900 is calculated using 5,000 units produced instead of the actual 4,500. Answer (D) is incorrect because the $700 variance is favorable.

CMA EXAMINATION QUESTIONS Page 122 of 142

MANAGEMENT ADVISORY SERVICES STANDARD COSTS AND VARIANCE ANALYSIS

151 . $8,400 - $9,000 = $600 (F)

152 . $600 (F) - 180 (U) = $780 (F)

153 . Answer (D) is correct. Since depreciation is a fixed cost, that cost will be the same each month regardless of production. Therefore, the budget for September would show depreciation of $21,500 ($258,000 annual depreciation x 1/12). Answer (A) is incorrect because depreciation is a fixed cost that will be the same each month regardless of production. The budget for September would show depreciation of $21,500 ($258,000 x 1/12). Answer (B) is incorrect because $20,425 is based on the units-of-production method. Answer (C) is incorrect because $20,500 is the amount shown in the accounts.

154 . Answer (D) is correct. The budgeted depreciation expense should be $100,000 at all levels of production. Thus, the variance will be $3,000 unfavorable ($100,000 budgeted - $103,000 actual). Answer (A) is incorrect because the variance is unfavorable. Answer (B) is incorrect because the variance is unfavorable. Answer (C) is incorrect because a flexible budget fixed cost should not increase when production increases.

155 . Answer (C) is correct. The $324,000 for supervisory salaries is a fixed cost, at a rate of $27,000 per month. Since these costs are fixed, volume is irrelevant. Thus, the variance is the difference between actual costs of $28,000 and the budgeted costs of $27,000, which equals $1,000 unfavorable. Answer (A) is incorrect because $350 results from calculating supervisory salaries on the basis of volume rather than as fixed costs {[($324,000 ÷ 180,000 units) x 15,750 units] - $28,000}. Answer (B) is incorrect because $350 results from calculating supervisory salaries on the basis of volume rather than as fixed costs {[($324,000 ÷ 180,000 units) x 15,750 units] - $28,000}. Answer (D) is incorrect because the variance is $1,000 unfavorable. Actual costs are greater than budgeted costs.

156 . $315,000 –$300,000 = $15,000 (U)

157 . $300,000/$3 = 100,000 DLH

158 . $300,000 – (55,000 x 2 x $3) = $30,000 (F)

159 . $108,000 – $2,600 = $105,400

160 . $400,000 + $16,000 = $416,000

161 . $210,000 - $8,000 = $202,000

CMA EXAMINATION QUESTIONS Page 123 of 142

MANAGEMENT ADVISORY SERVICES STANDARD COSTS AND VARIANCE ANALYSIS

163 . 10,000 mh x $5.00 = $50,000

164 . ($5.25-$5.00) x 9,800 mh = $2,450 unfavorable

165 . [9,800 – 10,000] x $5.00 = $1,000 favorable

166 . 15,000 mh x $11.00 = $165,000

167 . [30,000 x (10,800/24,000)] x $11.25 = $151,875

168 . ($11.00-$11.25) x 15,000 mh = $3,750 favorable

169 . [15,000 - (30,000 x .45) mh] x $11.25 = $16,875 unfavorable

170 . $161,250/15,000 = $10.75

171 . 22,000 x ($161,250/15,000)] = $236,500

172 . $242,000 – [22,000 x ($161,250/15,000)] = $5,500 unfavorable

173 . $360,000/20,000 = $18.00

174 . 18,000 x ($360,000/20,000)] = $324,000

175 . $342,000 – [18,000 x ($360,000/20,000)] = $18,000 unfavorable

176 . $120,000, the same lump sum as the static budget

177 . [25,000 x (6,000/24,000)] x $20.00 = $125,000CMA EXAMINATION QUESTIONS Page 124 of 142

MANAGEMENT ADVISORY SERVICES STANDARD COSTS AND VARIANCE ANALYSIS

178 . $123,000 actual costs - $120,000 budgeted cost = $3,000 unfavorable179 . $120,000 - [25,000 x (6,000/24,000) x $20.00] = $5,000 favorable

180 . $120,000, the same lump sum as the static budget

181 . [10,000 x (6,000/12,000)] x $20.00 = $100,000

182 . $120,000 - [10,000 x (6,000/12,000) x $20.00] = $20,000 unfavorable

183 . $122,000 - [10,000 x (6,000/12,000) x $20.00] = $22,000 underallocated

184 . (12,000 – 10,000) x ($24,000/12,000) = $4,000 (U)

185 . $13,000 - $12,000 = $1,000 (U)

186 . $35,000 – (10,000 x $3) = $5,000 (U)

187 . ($3 x 10,000) – ($3 x 9,000) = $3,000 (U)

188 . $78,000 - $35,000 – ($5 x 9,000) = $2,000 (F)

189 . Answer (D) is correct. The predetermined overhead application rate is $15 [($1,200,000 FOH + $2,400,000 VOH) ÷ 240,000 machine hours]. Answer (A) is incorrect because $5 is the fixed overhead application rate. Answer (B) is incorrect because $25 is the fixed overhead per labor hour. Answer (C) is incorrect because $10 is the variable portion of the overhead application rate.

190 . Answer (B) is correct. Overhead is applied on the basis of planned machine hours. The predetermined overhead application rate is $15 [($1,200,000 FOH + $2,400,000 VOH) ÷ 240,000 machine hours]. Thus, total overhead applied was $315,000 ($15 x 21,000 planned machine hours based on output). Answer (A) is incorrect because the total overhead applied was $315,000 based on 21,000 hours at $15 per hour. Answer (C) is incorrect because the total overhead applied was $315,000 based on 21,000 hours at $15 per hour. Answer (D) is incorrect because $300,000 is based on planned direct labor hours at $75 per hour.

CMA EXAMINATION QUESTIONS Page 125 of 142

MANAGEMENT ADVISORY SERVICES STANDARD COSTS AND VARIANCE ANALYSIS

191 . Answer (B) is correct. Variable overhead applied in November was $210,000 [21,000 planned machine hours based on output x ($2,400,000 planned annual VOH ÷ 240,000 planned machine hours)]. Because the applied overhead was less than actual ($214,000), underapplied variable overhead equaled $4,000. Answer (A) is incorrect because the overhead was underapplied. Answer (C) is incorrect because the overhead was underapplied. Answer (D) is incorrect because $6,000 is based on the 22,000 machine hours planned for November rather than the planned hours for actual output.

192 . Answer (A) is correct. The variable overhead spending variance equals the difference between actual variable overhead and the product of the actual input and the budgeted application rate. At a variable overhead application rate (standard cost) of $10 per machine hour ($2,400,000 ÷ 240,000 hours), the total standard cost for the 21,600 actual hours was $216,000. Given actual costs of $214,000, the favorable variance is $2,000. Answer (B) is incorrect because $6,000 is based on planned machine hours of 22,000. Answer (C) is incorrect because the variance is favorable. Answer (D) is incorrect because the variance is favorable.

193 . Answer (D) is correct. The fixed overhead volume (idle capacity) variance is the difference between budgeted fixed costs and the product of the standard fixed overhead cost per unit of input and the standard units of input allowed for the actual output. Budgeted fixed costs for the month were $100,000. The standard cost of actual output was $105,000 [21,000 machine hours planned for actual output x ($1,200,000 planned annual FOH ÷ 240,000 planned annual machine hours) FOH application rate]. Hence, the fixed overhead volume variance was $5,000 favorable. It was favorable because the budget for fixed overhead was less than the amount applied to jobs. An overapplication of fixed overhead suggests that output exceeded expectations. Answer (A) is incorrect because the variance was favorable. Answer (B) is incorrect because the variance was favorable. Answer (C) is incorrect because $10,000 is based on 22,000 planned machine hours.

194 . Answer (B) is correct. Two standard hours are allowed for each unit of production. Given actual production of 198,000 units, total standard hours allowed equal 396,000 (2 x 198,000). Answer (A) is incorrect because total standard hours allowed equal 396,000. Answer (C) is incorrect because total standard hours allowed equal 396,000. Answer (D) is inc orrect because total standard hours allowed equal 396,000.

195 . Answer (A) is correct. The variable overhead efficiency variance equals the difference between actual and standard direct labor hours times the standard cost per hour. Fixed overhead was budgeted at $600,000 ($3 x 200,000 expected units). Thus, total variable overhead was estimated to be $300,000 ($900,000 total OH - $600,000), and the variable overhead application rate was $.75 per hour [$300,000 ÷ (2 hours x 200,000 units)]. Standard hours for actual production are 396,000 (198,000 units x 2). Actual hours worked were 440,000. Hence, the variable overhead efficiency variance is $33,000 [$.75 x (440,000 actual hours - 396,000 standard hours for actual output)]. The variance is unfavorable because actual hours exceeded budgeted hours. Answer (B) is incorrect because the variable overhead efficiency variance is $33,000U. Answer (C) is incorrect because the variable overhead efficiency variance is $33,000U. Answer (D) is incorrect because the variable overhead efficiency variance is $33,000U.

196

?. Answer (C) is correct. Based on the 440,000 hours actually worked and the $.75 per hour variable overhead rate, the total standard cost for variable overhead is $330,000. The actual variable overhead totaled $352,000. The $22,000 variable overhead spending variance is unfavorable because the actual cost was higher than the standard.

CMA EXAMINATION QUESTIONS Page 126 of 142

MANAGEMENT ADVISORY SERVICES STANDARD COSTS AND VARIANCE ANALYSIS

197 . Answer (B) is correct. Actual fixed overhead was $575,000. Budgeted fixed overhead was $3 per unit at an estimated production of 200,000 units; a total of $600,000. The difference of $25,000 is a favorable variance because the actual amount was less than that budgeted. Answer (A) is incorrect because the fixed overhead spending variance is $25,000F. Answer (C) is incorrect because the fixed overhead spending variance is $25,000F. Answer (D) is incorrect because the fixed overhead spending variance is $25,000F.

198 . Answer (C) is correct. Fixed overhead is applied at the rate of $3 per unit. The amount applied given actual production is $594,000 ($3 x 198,000 units). Answer (A) is incorrect because the fixed overhead applied is $594,000. Answer (B) is incorrect because the fixed overhead applied is $594,000. Answer (D) is incorrect because the fixed overhead applied is $594,000.

199 . Answer (A) is correct. The fixed overhead volume variance results when production varies from the denominator amount. The denominator amount is the level of production used to determine the standard cost per unit. Because production was expected to be 200,000 units (the denominator level), but actual production was only 198,000 units, an unfavorable volume variance of 2,000 units occurred. Thus, 2,000 units were not charged with $3 per unit of overhead, and the volume variance in dollars was $6,000U (2,000 units x $3). This underapplication of fixed overhead is unfavorable because it indicates an underuse of facilities; that is, activity was less than budgeted. Unlike other variances, this variance does not measure deviations from expected costs but rather the departure from the expected use of productive capacity. Answer (B) is incorrect because the fixed overhead volume variance is $6,000U. Answer (C) is incorrect because the fixed overhead volume variance is $6,000U. Answer (D) is incorrect because the fixed overhead volume variance is $6,000U.

200 . $150,000/($100,000 + $50,000 + $150,000) x $24,000 = $12,000

201 . Adjustment to cost of goods sold= $200,000/($25,000 + $75,000 + $200,000) x $120,000= $200,000/$300,000 x $120,000 = $80,000

Cost of goods sold$400,000Add: Adjustment for underapplied overhead 80,000Adjusted cost of goods sold$480,000202 . (D) the requirement is to determine the amount of costs of goods sold to be reported on the 2002 income statement. The balance in the cost of goods sold account is $720,000. This amount must

be increased by the portion of underapplied overhead allocated to cost of goods sold. The underapplied overhead is appropriately allocated to work in process, finished goods, and cost of goods sold. No overhead is allocated to direct materials inventory, since this account contains only the cost of unused materials. The other three accounts contain the cost of materials, labor, and overhead. The amounts to be allocated to work in process, finished goods,a nd cost of goods sold are determined by each account’s relative balance as compared to the total balance in the accounts. The total balance of the three accounts is $864,000 ($720,000 + $54,000 + $90,000). Therefore, the amount allocable to cost of goods sold is [($720,000/$864,000) x $45,000) or $37,500. Since overhead is underapplied, not enough costs were applied to production during the year. Thus, cost of goods sold is increase to $757,500 ($720,000 + $37,500).

CMA EXAMINATION QUESTIONS Page 127 of 142

MANAGEMENT ADVISORY SERVICES STANDARD COSTS AND VARIANCE ANALYSIS

206 . 8,000 x 4 x $3.60 = $115,200

207 . 26,000 - (8,000 x 3 x $1) = $2,000

208 . The variance would be allocated only to finished goods and cost of goods sold.

209 . Answer (B) is correct. The direct materials efficiency variance equals the standard unit price times the difference between inputs actually used and inputs that should have been used, or $1,680 unfavorable {[10,920 lbs. - (2,100 units x 5 lbs.)] x $4}. Answer (A) is incorrect because $1,092 unfavorable equals the price variance. Answer (C) is incorrect because $3,680 unfavorable assumes 10,000 pounds should have been used. Answer (D) is incorrect because $3,772 unfavorable assumes 10,000 pounds should have been used and that the standard price was $4.10.

210 . Answer (A) is correct. The direct labor flexible budget variance (the total direct labor variance) equals actual direct labor cost minus standard direct labor cost for actual output produced, or $500 favorable [$20,500 - ($10 x 2,100 units)]. Answer (B) is incorrect because the variance was favorable. Answer (C) is incorrect because $1,000 favorable is the labor efficiency variance. Answer (D) is incorrect because both hours and wage rate varied and did not offset each other.

211 . Answer (B) is correct. The direct materials price variance equals the actual price minus the standard price, times the actual quantity used in production. The variance is $760,000 unfavorable [($28 - $24) x 190,000]. The variance is unfavorable because the actual price exceeded the standard price. Answer (A) is incorrect because the variance is unfavorable. Answer (C) is incorrect because $240,000 is the direct materials efficiency variance. Answer (D) is incorrect because $156,000 is the direct labor rate variance.

212 . Answer (B) is correct. The direct materials efficiency variance equals the actual quantity minus the standard quantity, times standard price. The variance is $240,000 favorable {[190,000 - (10 x 20,000)] x $24}. The variance is favorable because the actual quantity was less than the standard quantity allowed for the actual output. Answer (A) is incorrect because $156,000 favorable is the direct labor rate variance. Answer (C) is incorrect because the variance is favorable. Answer (D) is incorrect because $760,000 is the direct materials price variance.

213 . Answer (C) is correct. The direct labor rate variance equals the actual rate minus the standard rate, times the actual amount of labor used. The variance is $156,000 favorable [($18 - $20) x 78,000]. The variance is favorable because the actual rate was less than the standard rate. Answer (A) is incorrect because $240,000 favorable is the direct materials efficiency variance. Answer (B) is incorrect because the variance was favorable. Answer (D) is incorrect because $40,000 results from multiplying the actual units of output by the difference between the actual rate and standard rate.

CMA EXAMINATION QUESTIONS Page 128 of 142

MANAGEMENT ADVISORY SERVICES STANDARD COSTS AND VARIANCE ANALYSIS

215 . 110 x ($2.00 - $1.90) = $11 (F)

216 . (184 - 200) x $10 = $160 (F)

217 . Answer (B) is correct. The direct materials price variance equals the actual quantity purchased times the difference between the actual and standard unit costs. The standard unit cost for direct materials is $1.80 per pound. The actual cost for 160,000 pounds was $304,000, or $1.90 per pound. The variance is unfavorable because the actual price exceeded the standard price. Thus, the variance is $16,000U ($.10 x 160,000 lbs.). Answer (A) is incorrect because the price variance is $16,000U. Answer (C) is incorrect because the price variance is $16,000U. Answer (D) is incorrect because the price variance is $16,000U.

218 . Answer (D) is correct. The direct materials quantity variance equals the standard price ($1.80 per pound) times the difference between the actual and standard quantities. The actual quantity used was 142,500 pounds. The standard quantity is 8 pounds per unit of product. Given that 19,000 units were produced, the standard quantity for the actual output was 152,000 pounds (8 lbs. x 19,000 units). Hence, the direct materials quantity variance is $17,100 [$1.80 x (152,000 - 142,500)]. Since the actual quantity used was less than the standard quantity, the variance is favorable. Answer (A) is incorrect because the direct materials quantity variance is $17,100F. Answer (B) is incorrect because the direct materials quantity variance is $17,100F. Answer (C) is incorrect because the direct materials quantity variance is $17,100F.

219 . Answer (A) is correct. The direct labor rate variance equals the actual quantity of labor used times the difference between the actual and standard prices for labor. The actual total price of labor was $42,000, 90% of which was for direct labor. Thus, the price of direct labor was $37,800. A total of 5,000 hours of direct labor was worked. Thus, the actual hourly rate was $7.56 ($37,800 ÷ 5,000 hrs.), and the variance is $2,200 [5,000 hrs. x ($8.00 - $7.56)]. The actual rate was less than standard, so the variance is favorable. Answer (B) is incorrect because the labor rate variance is $2,200F. Answer (C) is incorrect because the labor rate variance is $2,200F. Answer (D) is incorrect because the labor rate variance is $2,200F.

220 . Answer (C) is correct. The direct labor efficiency variance equals the standard labor rate times the difference between actual and standard hours. Because each unit requires .25 hours of labor, the standard hours allowed for November would have been 4,750 (.25 x 19,000 units of output). Accordingly, the variance is $2,000 [$8.00 standard rate x (5,000 actual hrs. - 4,750 standard hrs.)]. This variance is unfavorable because the actual hours exceeded the standard hours. Answer (A) is incorrect because the direct labor efficiency variance is $2,000U. Answer (B) is incorrect because the direct labor efficiency variance is $2,000U. Answer (D) is incorrect because the direct labor efficiency variance is $2,000U.

221 . Answer (B) is correct. The materials price variance equals the actual quantity purchased times the difference between actual and standard unit prices. Actual cost was $475,000, or $3.80 per pound. Hence, the variance is $25,000 unfavorable [125,000 pounds x ($3.80 - $3.60)]. Answer (A) is incorrect because the variance is unfavorable. The purchase price exceeded standard cost. Answer (C) is incorrect because the variance is unfavorable. The purchase price exceeded standard cost. Answer (D) is incorrect because the price variance is computed on the actual quantity purchased, not the quantity used.

CMA EXAMINATION QUESTIONS Page 129 of 142

MANAGEMENT ADVISORY SERVICES STANDARD COSTS AND VARIANCE ANALYSIS

222 . Answer (D) is correct. This variance equals the standard unit cost times the difference between the actual quantity used and the standard quantity for good production. Consequently, the variance is $7,200 favorable {$3.60 x [(5 pounds x 22,000 units) - 108,000 pounds used]}. Answer (A) is incorrect because the variance is favorable. Actual usage was less than the standard. Answer (B) is incorrect because the variance is calculated by multiplying the quantity difference times the standard unit cost of $3.60, not the actual unit cost. Answer (C) is incorrect because the variance is favorable. Actual usage was less than the standard.

223 . Answer (A) is correct. The direct labor rate variance equals the actual quantity of hours worked times the difference between the standard and actual labor rates. Total direct labor cost was $327,600 (90% x $364,000), and the actual unit direct labor cost was $11.70 ($327,600 ÷ 28,000 hours). Thus, the variance is $8,400 favorable [28,000 hours x ($12 - $11.70)]. Answer (B) is incorrect because the variance is favorable. The actual labor rate was less than the standard rate. Answer (C) is incorrect because the variance is favorable. The actual labor rate was less than the standard rate. Answer (D) is incorrect because $6,000 is the labor efficiency variance, not the labor rate variance.

224 . Answer (B) is correct. The direct labor efficiency variance equals the standard unit cost times the difference between actual hours and standard hours. Accordingly, the variance is $6,000 unfavorable {$12 x [28,000 hours - (1.25 hours x 22,000 units)]}. Answer (A) is incorrect because the variance is unfavorable. More hours were worked than allowed by the standards. Answer (C) is incorrect because the labor efficiency variance is $6,000 unfavorable. Answer (D) is incorrect because the variance is unfavorable. More hours were worked than allowed by the standards.

225 . $30,000 + 500 U – 1,500 F = $29,000

226 . (490 x $32) – (5,000 x 0.10 x $30) = 680 U

227 . 490 ($32 - $30) = 980 U

228 . $30 (490 – 500) = 300 F

229 . 250 dlh ($15.25 - $15.00) = $62.50 U

230 . [250 dlh - (5,000 x 0.05)] x $15 = Zero

231 . (2200 x $24) – (10,000 x 0.20 x $25) = 2,800 U

232 . 2200 ($24 - $25) = 2,200 FCMA EXAMINATION QUESTIONS Page 130 of 142

MANAGEMENT ADVISORY SERVICES STANDARD COSTS AND VARIANCE ANALYSIS

235 . 1050 dlh ($14.75 - $15.00) = $262.50 F

236 . [1050 dlh - (10,000 x 0.10)] x $15 = 750 U

237 . Answer (D) is correct. The total materials price variance is found by multiplying the difference between the standard price and the actual price by the actual quantity. The actual price is calculated by dividing actual cost by actual quantity. Therefore, the actual price for housing units is $20/unit ($44,000 ÷ 2,200), for printed circuit boards, $16/unit ($75,200 ÷ 4,700), and for reading heads, $11/unit ($101,200 ÷ 9,200). Thus, total materials price variance is

Housing units: 2,200 x ($20 - $20) = $ 0Printed circuit boards: 4,700 x ($15 - $16) =4,700 UReading heads: 9,200 x ($10 - $11) =9,200 U $13,900 UAnswer (A) is incorrect because $346,500 favorable results from using the standard cost per unit for each direct material, and also by reversing the order of subtraction. Answer (B) is incorrect because $346,500 unfavorable results from using the

standard cost per unit for each direct material. Answer (C) is incorrect because the price variance is unfavorable when the actual price is greater than the standard price.

238 . Answer (A) is correct. The total materials quantity variance is found by multiplying the difference between the standard quantity and actual quantity by the standard price. Standard quantities are calculated by multiplying the actual units by the standard quantity per unit. In this example, the standard quantity for housing units is 2,200 parts (2,200 x 1); for printed circuit boards, 4,400 parts (2,200 x 2); and for reading heads, 8,800 parts (2,200 x 4). Therefore, the total materials quantity variance is

Housing units: $20 x (2,200 - 2,200) = $ 0Printed circuit boards: $15 x (4,400 - 4,700) = 4,500 UReading heads: $10 x (8,800 - 9,200) = 4,000 U $8,500 UAnswer (B) is incorrect because $8,500 favorable results from reversing the order of subtraction. Answer (C) is incorrect because $9,200 unfavorable results from multiplying by actual price. Answer (D) is incorrect because $9,200 favorable

results from multiplying by the actual price and reversing the order of subtraction.

239 . Answer (A) is correct. The variable overhead efficiency variance is found by multiplying the difference between standard hours and actual hours by the standard rate. The number of standard hours is calculated by multiplying the actual units by the standard hours per unit. Therefore, the number of standard hours is 9,900 ($2,200 x 4.5 hours per unit), and the variable overhead efficiency is $0 [$2 x (9,900 - 9,900)]. Answer (B) is incorrect because $900 unfavorable results from multiplying by the budgeted number of units, 2,000, instead of actual, 2,200. Answer (C) is incorrect because $9,900 unfavorable results from using a standard hours per unit rate of 9 hours and reversing the order of subtraction. Answer (D) is incorrect because $9,900 favorable results from using a standard hours per unit rate of 9 hours.

240 . Answer (B) is correct. The variable overhead spending variance is found by subtracting actual variable overhead from the product of actual hours and the standard rate. Therefore, the variable overhead spending variance is $1,000 favorable [(9,900 x $2) - $18,800]. Answer (A) is incorrect because $1,000 unfavorable results from reversing the order of subtraction. Answer (C) is incorrect because $1,800 unfavorable results from using the budgeted variable overhead and by reversing the order of subtraction. Answer (D) is incorrect because $1,800 favorable results from using the budgeted variable overhead.

CMA EXAMINATION QUESTIONS Page 131 of 142

MANAGEMENT ADVISORY SERVICES STANDARD COSTS AND VARIANCE ANALYSIS

245 Answer (D) is correct. The variable overhead spending and efficiency variances are the components of the total variable overhead variance. Given that actual variable overhead was $80,000 and the flexible budget amount was $90,000, the total variance is $10,000 favorable. If the overhead spending variance is $2,000 favorable the efficiency variance must be $8,000 favorable ($10,000 total - $2,000 spending). At a rate of $20 per hour, this variance is equivalent to 400 direct labor hours ($8,000 ÷ $20). Answer (A) is incorrect because the variances are favorable. Answer (B) is incorrect because 100 direct labor hours are equivalent to the spending variance ($20 x 100 hours = $2,000). Answer (C) is incorrect because the variances are favorable.

246 . Answer (D) is correct. Variable overhead variances can be subdivided into spending and efficiency components. However, fixed overhead variances do not have an efficiency component because fixed costs, by definition, are not related to changing levels of output. Consequently, there is no concept of efficiency with respect to the incurrence of fixed costs. Fixed overhead variances are typically subdivided into a budget (or fixed overhead spending) variance and a volume variance. Answer (A) is incorrect because efficiency variances are applicable to variable costs. Answer (B) is incorrect because efficiency variances are applicable to variable costs. Answer (C) is incorrect because efficiency variances are applicable to variable costs.

247

CMA EXAMINATION QUESTIONS Page 132 of 142

MANAGEMENT ADVISORY SERVICES STANDARD COSTS AND VARIANCE ANALYSIS

?. Answer (A) is correct. The fixed overhead spending (budget) variance is the difference 248 . Answer (B) is correct. The variable overhead spending variance is the difference between actual variable overhead and the variable overhead based on the standard rate and the actual activity level.

Thus, the variable overhead spending variance was $12,000 favorable [$740,000 actual cost - ($8 standard rate x 94,000 actual hours)]. Because actual is less than standard, the variance was favorable. Answer (A) is incorrect because $60,000 favorable is based on 100,000 hours, not the actual hours of 94,000. Answer (C) is incorrect because $48,000 unfavorable is the variable overhead efficiency variance. Answer (D) is incorrect because $40,000 unfavorable is the fixed overhead spending variance.

249 . Answer (A) is correct. The variable overhead efficiency variance equals the standard price ($8 an hour) times the difference between the actual hours and the standard hours allowed for the actual output. Thus, the variance is $48,000 {$8 x [94,000 actual hours - (4 standard hours per unit x 22,000 units produced)]}. The variance is unfavorable because actual hours exceeded standard hours. Answer (B) is incorrect because $60,000 favorable is the variable overhead spending variance calculated based on capacity, not actual hours. Answer (C) is incorrect because $96,000 unfavorable is based on the difference between standard hours allowed for the actual output and capacity hours. Answer (D) is incorrect because $200,000 unfavorable is the excess of actual direct labor costs over actual variable overhead costs.

250 . Answer (B) is correct. The direct labor price variance equals actual labor hours times the difference between standard and actual labor rates. The actual labor cost was $940,000 for 94,000 hours, or $10 per hour. The standard rate was $9 per hour. Thus, the variance is $94,000 [94,000 hours x ($10 - $9)]. The variance is unfavorable because the actual rate paid was higher than the standard rate. Answer (A) is incorrect because $54,000 unfavorable is the direct labor efficiency variance. Answer (C) is incorrect because $60,000 favorable equals the actual rate times the difference between capacity and actual hours. Answer (D) is incorrect because $148,000 unfavorable is the total direct labor variance.

251 . Answer (D) is correct. The direct labor efficiency variance equals the standard rate times the difference between actual and standard hours. Hence, the variance is $54,000 {$9 x [94,000 hours - (4 standard hours per unit x 22,000 units)]}. The variance is unfavorable because the actual hours exceeded the standard hours. Answer (A) is incorrect because $108,000 favorable is based on the difference between standard and capacity hours. Answer (B) is incorrect because $120,000 favorable is based on the actual rate and the difference between standard hours and capacity. Answer (C) is incorrect because $60,000 favorable is based on the actual rate and the difference between actual hours and capacity.

252 . Answer (C) is correct. The materials quantity variance equals the standard price times the difference between the actual and standard quantities. Hence, the favorable materials quantity variance is $4,950 [$1.50 standard x 1,650 units x (60 standard pounds - 58 actual pounds)]. Answer (A) is incorrect because $14,355 is the amount of the materials price variance. Answer (B) is incorrect because $14,355 is the amount of the materials price variance. Answer (D) is incorrect because a favorable variance exists. The standard amount for the actual output exceeded the actual amount.

253 . Answer (A) is correct. The materials price variance equals the actual quantity used times the difference between the actual and standard price per unit. Thus, the unfavorable materials price variance is $14,355 [58 actual pounds x 1,650 units x ($1.65 actual price - $1.50 standard price)]. Answer (B) is incorrect because $14,850 is based on the standard unit quantity, not the actual quantity. Answer (C) is incorrect because the price variance is unfavorable. The actual price is greater than the standard price. Answer (D) is incorrect because $14,850 is based on the standard unit quantity, not the actual quantity.

CMA EXAMINATION QUESTIONS Page 133 of 142

MANAGEMENT ADVISORY SERVICES STANDARD COSTS AND VARIANCE ANALYSIS

255 . Answer (D) is correct. The flexible budget overhead variance is the difference between actual overhead costs and the flexible budget amount for the actual output. Standard total fixed costs at any level of production are $27,000. Standard variable overhead is $9 per unit ($3 x 3 labor hours). Thus, total standard variable overhead is $14,850 for the actual output ($9 x 1,650 units), and the total flexible budget amount is $41,850 ($27,000 FOH + $14,850 VOH). Accordingly, the favorable flexible budget variance is $1,920 ($41,850 flexible budget amount - $39,930 actual amount). Answer (A) is incorrect because $3,270 is the flexible budget amount for an output of 1,800 units. Answer (B) is incorrect because $3,270 is the flexible budget amount for an output of 1,800 units. Answer (C) is incorrect because a favorable variance exists. Actual overhead is less than the standard overhead at the actual production level.

256 . Answer (A) is correct. The materials quantity variance equals the standard price times the difference between the actual and standard quantities. This variance is therefore equal to $9,900 favorable [$1.50 standard unit cost x (116 lbs. used per unit - 120 lbs. standard per unit) x 1,650 units produced]. Answer (B) is incorrect because the quantity variance is favorable. Answer (C) is incorrect because $28,710 is the magnitude of the price variance. Answer (D) is incorrect because $28,710 is the magnitude of the price variance.

257 . Answer (A) is correct. The materials price variance equals the actual quantity used times the difference between the actual and standard prices per unit. This variance therefore equals $28,710 Unfavorable [(116 lbs. x 1,650 units) x ($1.65 - $1.50)]. Answer (B) is incorrect because $29,700 is the magnitude of the variance if the standard quantity is used. Answer (C) is incorrect because the price variance is unfavorable. Answer (D) is incorrect because $29,700 is the magnitude of the variance if the standard quantity is used.

258 . Answer (D) is correct. The labor rate variance equals actual hours used times the difference between actual and standard rates. It is calculated in the same way as the materials price variance. Because the standard rate and actual rate were both $12 per hour, the labor rate variance is $0. Answer (A) is incorrect because $2,700 is the magnitude of the labor efficiency variance if 1,800 units are produced. Answer (B) is incorrect because $2,700 is the magnitude of the labor efficiency variance if 1,800 units are produced. Answer (C) is incorrect because $2,475 Unfavorable is the labor efficiency variance.

259 . Answer (D) is correct. The flexible budget overhead variance is the difference between total actual overhead costs and the amount shown on a flexible budget. Standard fixed costs within the relevant range of production are $54,000. Standard variable overhead is $6 per labor hour, or $18 per unit (3 x $6). Accordingly, budgeted total overhead at the actual output level is $83,700 [($18 VOH x 1,650 units) + $54,000 FOH], and the flexible budget overhead variance is $3,840 Favorable ($79,860 actual - $83,700 budgeted). Answer (A) is incorrect because a favorable variance exists. Answer (B) is incorrect because $6,540 Favorable assumes 1,800 units were produced. Answer (C) is incorrect because a favorable variance exists.

260 . ($48,000 x $15) – (48,000 x $16) = $48,000 favorable

261 . (48,000 x $16) – (10,000 x 5 x $16) = $32,000 favorable

CMA EXAMINATION QUESTIONS Page 134 of 142

MANAGEMENT ADVISORY SERVICES STANDARD COSTS AND VARIANCE ANALYSIS

265 . (22,500 – 22,000) x ($30,000/20,000) = $750 (U)

266 . $18,000 - (22,000 x (18,000/20,000)) = $1,800 (F)

267 . $4,500 F + $10,000 U = $ 5,500 U

268 . $4,500 F + $10,000 U + $15,000 U = $20,500 U; $40,000 U

269 . $4,500 F + $10,000 U + $15,000 U + $40,000 U = $60,500 U

270 Answer (C) is correct. The total employee benefits include 20% of supervision and direct and indirect labor costs. To find the amount associated with direct labor, 20% of supervision and indirect labor costs are subtracted from total employee benefits [$575,000 - 20% x ($250,000 + $375,000)], or $450,000. Answer (A) is incorrect because 20% of supervision and indirect labor costs need to be subtracted from total employee benefits to determine the employee benefits associated with direct labor costs. Answer (B) is incorrect because $75,000 is the result of deducting 80% of supervision and indirect labor costs from total employee benefits. Answer (D) is incorrect because 20% of supervision also needs to be deducted.

271 Answer (D) is correct. To determine the variable manufacturing O/H rate, all variable amounts must be totaled ($1,500,000) and divided by the capacity in DLH (300,000). Total Per DLHSupplies $ 420,000 $1.40Indirect labor 375,000 1.25Power 180,000.60Employee benefits: 20% direct labor450,000 1.50 20% indirect labor 75,000.25 Total $1,500,000 $5.00Answer

(A) is incorrect because $7.80/hr. is the fixed manufacturing O/H rate per direct labor hour. Answer (B) is incorrect because the variable manufacturing overhead rate is determined by dividing variable expenses (supplies, indirect labor, power, and direct and indirect labor benefits) by direct labor hours. Answer (C) is incorrect because $5.17/hr. incorrectly includes supervision benefits of $50,000.

272 Answer (A) is correct. The variable manufacturing cost to produce a 100-unit lot is 100 times the sum of direct materials, direct labor, and variable O/H per seat.Cushion materials Padding $2.40 Vinyl 4.00Total cushion materials $6.40 Cost increase 10% (given) x1.10Cost of cushioned seat $ 7.04Cushion fabrication labor ($7.50/DLH x .5 DLH)

3.75Variable overhead ($5.00/DLH x .5 DLH) 2.50Total variable cost per cushioned seat $13.29Total variable cost per 100-unit lot $1,329Answer (B) is incorrect because $1,869 is the transfer price plus the opportunity cost of $540 of the Office Division. Answer (C) is incorrect because $789 is the transfer price minus the opportunity cost of $540 of the Office Division. Answer (D) is incorrect

because the transfer price based on the variable manufacturing costs is $1,329.

273 Answer (A) is correct. Total fixed O/H is $2,340,000 (see below). It is divided by the 300,000-hour level of activity to determine the $7.80 hourly rate.Supervision $ 250,000Heat and light 140,000Property taxes and insurance 200,000Depreciation 1,700,000Benefits (20% of supervision) 50,000$2,340,000Answer (B) is incorrect because the fixed

manufacturing overhead rate is determined by dividing fixed expenses (supervision, heat and light, property taxes and insurance, depreciation, and supervision benefits) by direct labor hours. Answer (C) CMA EXAMINATION QUESTIONS Page 135 of 142

MANAGEMENT ADVISORY SERVICES STANDARD COSTS AND VARIANCE ANALYSIS

274 Answer (B) is correct. The labor hours used in cushion fabrication will be used to make the modified cushioned seat. Thus, the labor time freed by not making deluxe stools equals the frame fabrication and assembly time only. The number of economy office stools that can be produced is 125.

Labor hours to make 100 deluxe stools (1.5 x 100) 150 hr.Minus: Labor hours to make 100 cushioned seats (cushion fabrication .5 x 100)

(50)hr. Labor hours available for economy stool 100 hr.Labor hours to make one economy stool÷.8 hr.Stools produced by extra labor in economy stool production (100 ÷ .8 hr.)

125 stoolsAnswer (A) is incorrect because the total hours available for economy stools needs to be divided by the .8 hr. required to make an economy stool. Answer (C) is incorrect because the total hours available for economy stools needs to be divided by the .8 hr. required to make an economy stool. Answer (D) is incorrect because 150 is the number of hours required to make 100 deluxe stools

before considering the hours required to make 100 cushioned seats.

275 Answer (A) is correct. The contribution margin per unit is equal to the selling price minus the variable costs. Variable costs per unit for the deluxe office stool equal $33.30 and the selling price is $58.50. Thus, the contribution margin is $25.20 per unit ($58.50 - $33.30). The total standard cost is $45.00, which includes $11.70 of fixed O/H (1.5 hr. x $7.80), and the variable costs are $33.30 ($45.00 - $11.70). Answer (B) is incorrect because $15.84 is the contribution margin of the economy office stool. Answer (C) is incorrect because variable costs of $33.30 need to be deducted from the sales price of $58.50. Answer (D) is incorrect because $33.30 is the variable cost which must be deducted from the sales price to yield the contribution margin.

276 Answer (D) is correct. Opportunity cost is the benefit of the next best opportunity forgone. The opportunity cost here is the contribution margin forgone by shifting production to the economy office stool ($2,520 - $1,980 = $540).

DeluxeEconomySelling price$58.50$41.60Costs Materials $14.55$15.76 Labor ($7.50 x 1.5)11.25 ($7.50 x .8) 6.00 Variable O/H ($5 x 1.5)7.50 ($5 x .8) 4.00 Fixed O/H-- --Total costs $33.30$25.76Unit CM$25.20$15.84Units produced x 100x 125Total CM $2,520$1,980Answer (A) is incorrect because $789 is the transfer price of $1,329 minus the opportunity

cost of $540 of the Office Division. Answer (B) is incorrect because $1,869 is the transfer price of $1,329 plus the opportunity cost of $540 of the Office Division. Answer (C) is incorrect because $1,329 is the transfer price, not the opportunity cost of the Office Division.

277 . a. 7,110 units ($72,310 + 5,770 - 6,980) / (2 x $5)b. 14,220 SH $71,100 / $5c. 15,616 AH $78,080 / $5d. 21,330 lbs ($130,760 - 3,500 + 720) / $6e. 21,210 lbs $127,260 / $6

278 . a. 8,734 units ($105,560 - 14,560 - 3,660) / (2 x $5)b. 17,468 SH 8,734 x 2

CMA EXAMINATION QUESTIONS Page 136 of 142

MANAGEMENT ADVISORY SERVICES STANDARD COSTS AND VARIANCE ANALYSIS

c. 18,200 AH ($105,560 - 14,560) / $5d. $4,392 U (18,200 - 17,468) x 6e. $96,880 (18,200 x $6) - $12,320

279 . a. $4 ($600,000/300,000 + $2 variable)b. $1,280,000 ($4 x 320,000)c. $31,000 overapplied ($1,280,000 - $1,249,000)d. $9,000 unfavorable {$1,249,000 - [($2 x 320,000) + $600,000]}e. $40,000 favorable ($1,240,000 budgeted - $1,280,000 applied)

is incorrect because $5.17/hr. incorrectly includes supervision benefits of $50,000. Answer (D) is incorrect because $5.00/hr. is the variable manufacturing O/H rate per hour.

262 . $204,000 –(48,000 x $4) = $12,000 unfavorable

263 . (48,000 x $4) – (10,000 x 5 x $4) = $8,000 favorable

264 . $175,500 - (22,000 x $8) = $500 (F)

254 . Answer (B) is correct. The labor rate variance equals the actual hours used times the difference between the actual and standard rates. Consequently, the labor rate variance is zero [3.1 actual hours x 1,650 units x ($12 per hour standard rate - $12 per hour actual rate)].

Answer (A) is incorrect because $1,920 is the amount of the flexible budget overhead variance. Answer (C) is incorrect because $4,950 is the amount of the quantity variance. Answer (D) is incorrect because $4,950 is the amount of the quantity variance.

between actual and budgeted fixed factory overhead. Actual fixed overhead was $540,000. Budgeted fixed overhead was $5 per hour based on a capacity of 100,000 direct labor hours per month, or $500,000. Because these costs are fixed, the budgeted fixed overhead is the same at any level of production. Hence, the variance is $40,000 unfavorable ($540,000 - $500,000). Answer (B) is incorrect because $70,000 unfavorable is the difference between actual fixed overhead and the product of the standard rate and the actual direct labor hours. Answer (C) is incorrect because $460,000 unfavorable is the volume variance. Answer (D) is incorrect because $240,000 unfavorable is the difference between actual variable overhead and budgeted fixed overhead.

241 . Answer (C) is correct. The contribution margin volume variance is found by multiplying budgeted unit contribution by the difference between actual units and budgeted units. The budgeted unit contribution in this example is $61 ($122,000 ÷ 2,000 units). Therefore, the variance is $12,200 favorable [$61 per unit x (2,200 actual units - 2,000 budgeted units). Answer (A) is incorrect because $9,800 unfavorable results from multiplying by the actual unit contribution and reversing the order of subtraction. Answer (B) is incorrect because $9,800 favorable results from multiplying by the actual unit contribution. Answer (D) is incorrect because $14,660 unfavorable is the variance between budgeted and actual contribution margins.

280 . a. $5 ($200,000/100,000 + $3 variable)b. $450,000 ($5 x 90,000)c. $7,000 underapplied ($450,000 - $457,000)d. $13,000 favorable {$457,000 - [($3 x 90,000) + $200,000]}e. $20,000 unfavorable ($470,000 budgeted - $450,000 applied)

CMA EXAMINATION QUESTIONS Page 137 of 142

MANAGEMENT ADVISORY SERVICES STANDARD COSTS AND VARIANCE ANALYSIS

281 . a.

Actual Fixed Overhead Cost$325,880

Flexible Budget Fixed Overhead Cost$324,000Fixed Overhead Cost Applied to Work in Process

302,100 MH x $1.08$326,268Budget Variance,$1,880 UVolume Variance

$2,268 FTotal variance, $388 FComputations -- in this order:

(Note: When used in the below algebraic formulas, favorable variances are negative and favorable variances are positive.)

Volume variance = Total variance - Budget variance = $388 F - $1,880 U = -$388 - $1,880 = -$2,268 = $2,268 F

Fixed overhead applied = 302,100 MH X $1.08 = $326,268

Flexible budget fixed overhead = Fixed overhead applied - Volume variance = $326,268 - $2,268 F = $326,268 + $2,268 = $324,000

Actual fixed overhead = Fixed overhead applied + Total variance = $326,268 + $388 F = $326,268 - $388

CMA EXAMINATION QUESTIONS Page 138 of 142

MANAGEMENT ADVISORY SERVICES STANDARD COSTS AND VARIANCE ANALYSIS

= $325,880

b. Standard MH allowed for production, . 302,100Standard hours allowed per unit ÷ 0.1Units produced 3,021,000

c. Fixed overhead in flexible budget, (a) above $ 324,000Standard cost per machine hour ÷ $1.08MH assumed in flexible budget 300,000Standard hours allowed per unit ÷ 0.1Units assumed in flexible budget 3,000,000

233 . $25 [2200 – (10,000 x 0.20)] = 5,000 U

234 . (1050 x $14.75) - (10,000 x 0.10 x $15) = $487.50 U

214 . (110 - 100) x $2 = $20 (U)

203 . (5,600 x $40) + (5,600 x $49.20) = $499,520

204 . (800 x $40) + (800 x .75 x $49.20) = $61,520

205 . 100,000 x $.78 = $78,000

Answer (A) is incorrect because the variable overhead spending variance is $22,000U. Answer (B) is incorrect because the variable overhead spending variance is $22,000U. Answer (D) is incorrect because the variable overhead spending variance is $22,000U.

162 . 9,800 mh x $5.25 = $51,450

147 . $16,800 - $18,000 = $1,200 (F)

148 . $1200 (F) - $360 (U) = $1,560 (F)

149 . $37,000 - (5,800 x $3 x 2) = $2,200 (U)

150 . [12,000 – (5,800 x 2)] x $3 = $1,200 (U)

$562,000 Factory overhead applied at standard 572,000 Volume variance$(10,000)favorable138 . Answer (C) is correct. Three-way analysis of variance combines the fixed overhead

budget (spending) and variable overhead spending variances of four-way analysis of variance. It includes spending, efficiency, and volume variances. The spending variance is the difference between the actual overhead incurred and the budgeted overhead for the actual input.

283 . a. Flexible budget allowance, $845,000 [$200,000 + (30,000 x 1 x $4) + (30,000 x 2.5 x 7)]b. Budget variance: $45,000 unfavorable ($845,000 - $890,000)

284 . a. Flexible budget allowance, $438,000 [$300,000 + (23,000 x 3 x $2)]b. Budget variance: $18,000 favorable ($438,000 - $420,000)

CMA EXAMINATION QUESTIONS Page 139 of 142

MANAGEMENT ADVISORY SERVICES STANDARD COSTS AND VARIANCE ANALYSIS

Budgeted overhead [$10,500 + (5,250 x $3.80)] $30,450Actual overhead (22,500) $7,950 FAnswer (A) is incorrect because $9,660 is based on standard, not actual, hours. Answer (B)

is incorrect because $8,250 results from using actual fixed overhead to calculate budgeted overhead. Answer (D) is incorrect because the spending variance is favorable.

139 . Answer (B) is correct. The solutions approach to compute the variable overhead efficiency variance is to set up a diagram as follows:

AH x SR (10,500 x $3)$31,500SH x SR [(2 x 5,000) x $]$30,000Variable overhead efficiency variance (unfavorable)$ 1,500

Answer (A) is incorrect because the standard O/H rate is multiplied by the 94,000 DMH allowed (not 98,700 actual DMH) for the 23,500 equivalent units of production. Answer (B) is incorrect because $1,432,000 is for the original 100,000 DMH budgeted, not the 94,000 DMH budgeted for the production of 23,500 units. Answer (C) is incorrect because $1,358,250 is the total actual O/H.

125 . Answer (A) is correct. The total O/H variance is the over- or underapplied O/H, that is, the difference between applied O/H and the actual O/H. The applied O/H was determined to be $1,346,080. The actual O/H is $1,358,250 ($133,250 + $1,225,000). Consequently, the amount of underapplied O/H is $12,170 U ($1,358,250 - $1,346,080). Answer (B) is incorrect because the applied O/H is $1,346,080, which is based on the budgeted DMH for the equivalent units of production, not on the actual DMH. Answer (C) is incorrect because the applied O/H is $1,346,080, which is based on the budgeted DMH for the equivalent units of production, not on the actual DMH. Furthermore, because the actual O/H is greater than the O/H applied, the underapplied O/H results in an unfavorable variance. Answer (D) is incorrect because $73,750 favorable assumes that standard input for the actual output was 100,000 DMH and that overhead applied was therefore $1,432,000.

118 . Answer (C) is correct. The applied factory overhead equals the standard input allowed for actual output multiplied by the total standard overhead rate per hour.27,500($5.00 VOH + $1.50 FOH) = $178,750

Answer (A) is incorrect because $137,500 equals standard variable overhead applied. Answer (B) is incorrect because $176,250 results from subtracting the $2,500 overhead variance. Answer (D) is incorrect because $182,000 is based on the actual DLH worked.

119 . Answer (B) is correct. Pane applies overhead to products on the basis of direct labor cost. The rate is 1.4 ($448,000 budgeted OH ÷ $320,000 budgeted DL cost). Thus, $483,000 (1.4 x $345,000 actual DL cost) of overhead was applied, of which $24,000 ($483,000 - $459,000 actual OH) was overapplied. Answer (A) is incorrect because $11,000 equals the difference between budgeted and actual overhead. Answer (C) is incorrect because $11,000 equals the difference between budgeted and actual overhead. Answer (D) is incorrect because the overhead was overapplied.

Answer (A) is incorrect because $7,200 is the annual fixed cost. Answer (B) is incorrect because $12,000 is the variable cost. Answer (D) is incorrect because $19,200 is based on fixed costs of $7,200.

105 . Answer (D) is correct. The total flexible budget variance is the difference between the standard cost of labor and the actual cost of labor. Based on the standard hours and rates given, the standard cost of labor is $159,060 [(7,920 x $12.00) + (4,620 x $8.00) + (4,510 x $6.00)]. The actual cost of labor is $160,805 ($100,245 + $35,260 + $25,300). Thus, the total flexible budget variance is $1,745 unfavorable ($160,805 actual - $159,060 standard). Answer (A) is incorrect because $2,205 is the difference between the actual cost and the sum of the products of actual hours and standard rates. Moreover, the variance is unfavorable. Answer (B) is incorrect because $2,205 unfavorable is the labor rate variance. Answer (C) is incorrect because the variance is unfavorable.

because $820 is the labor rate variance. Answer (D) is incorrect because $00 is the difference between the direct labor efficiency variance and the cost difference ($0.20) times the standard hours.

CMA EXAMINATION QUESTIONS Page 140 of 142

MANAGEMENT ADVISORY SERVICES STANDARD COSTS AND VARIANCE ANALYSIS

83 . Answer (B) is correct. The direct labor efficiency (quantity) variance equals standard price times the difference between actual and standard amounts of labor hours. The standard price is $10 per hour. The actual amount of labor hours is 3,200 hours. The standard amount of labor hours is 3,000 (2 hours x 1,500 units). Thus, the direct labor efficiency variance is $2,000 [$10 x (3,200 - 3,000)]. The variance is unfavorable because more labor hours were used than the standard. Answer (A) is incorrect because $2,050 uses the actual labor price. Answer (C) is incorrect because $1,250 is the difference between the direct labor efficiency variance and the product of the cost difference ($.25) and the standard hours allowed. Answer (D) is incorrect because $1,200 is the difference between the labor efficiency and rate variances.

$1.55) = $.05]. Answer (C) is incorrect because $1,200 favorable results from multiplying the direct materials that should have been used for budgeted production [(3 x 8,000) = 24,000 lbs.] times the difference between standard price and actual price [($1.60 - $1.55) = $.05].

Answer (A) is incorrect because $2,500 unfavorable is the materials quantity variance. Answer (B) is incorrect because the price variance is $12,500, or $.50 per unit. Answer (D) is incorrect because the price variance is $12,500, or $.50 per unit.

Input quantity – Spoilage = Output amountX – .2X = 2 yards.8X = 2 yardsx = 2.5 yards

Thus, the standard direct material cost per unit of finished product is $7.50 (2.5 yards x $3).

47 . Answer (B) is correct. The efficiency (quantity or usage) variance for direct materials equals standard unit price times the difference between actual usage and the standard usage for the actual output. Accordingly, the direct materials efficiency variance is $800 U {[23,000 lbs. issued - (3 lbs. x 7,500 units)] x $1.60}. The variance is unfavorable because actual usage exceeded standard usage.

Answer (A) is incorrect because $775 unfavorable results from multiplying the difference between standard quantity [(3 x 7,500) = 22,500] and actual quantity used (23,000) times the actual price ($1.55). Answer (C) is incorrect because $1,600 results from multiplying the difference between standard quantity that should have been used for budgeted production [(3 x 8,000) = 24,000] and actual quantity used (23,000) times the standard price ($1.60). Answer (D) is incorrect because $3,200 favorable results from multiplying the difference between the actual quantity purchased (25,000) and actual quantity used (23,000) times the standard price ($1.60).

$.225 per liter ($135 total cost ÷ 600 liters). Thus, the yield variance is $94.50 unfavorable [(84,420 liters used - 84,000 liters allowed) x $.225]. Answer (A) is incorrect because $294.50 favorable is the materials quantity variance. Answer (B) is incorrect because $388.50 favorable is the materials mix variance. Answer (D) is incorrect because $219.50 favorable is based on the actual mix of purchases.

Answer (A) is incorrect because $900 equals the unit price variance ($.10) times 9,000 units (3 x 3,000) in the radios manufactured. Answer (B) is incorrect because $900 equals the unit price variance ($.10) times 9,000 units (3 x 3,000) in the radios manufactured. Answer (D) is incorrect because the variance is unfavorable. The actual cost was greater than the standard cost.

Answer (A) is incorrect because $100 F ($48,500 - $48,600) is the direct labor price (rate) variance. Answer (C) is incorrect because the flexible budget direct labor variance is unfavorable, not favorable. Total actual cost exceeds the total flexible budget amount. Answer (D) is incorrect because $2,000 U ($48,600 - $46,600) is the direct labor efficiency (usage) variance.

84 . x = $10 [2,000 - (900 x 2)]x = $2,000 unfavorable

CMA EXAMINATION QUESTIONS Page 141 of 142

MANAGEMENT ADVISORY SERVICES STANDARD COSTS AND VARIANCE ANALYSIS

91 . Answer (B) is correct. The total direct labor variance can be isolated into the price variance and the efficiency variance. The labor price variance equals the actual price minus the standard price, times the actual quantity. Actual price is $8.70 ($27,840 ÷ 3,200 hours). Hence, the variance is $2,240 U [($8.70 - $8.00) x 3,200]. Answer (A) is incorrect because the variance is unfavorable. Answer (C) is incorrect because $3,840 equals actual direct labor cost ($27,840) minus the standard direct labor cost of the budgeted production (1,200 x 2.5 hours x $8). Answer (D) is incorrect because $5,600 unfavorable is the direct labor efficiency variance.

282 . Budgeted fixed overhead rate = Fixed overhead/Denominator quantity = $84,800/53,000 direct labor-hours = $1.60/direct labor-hour

Actual fixed overhead = Budgeted fixed overhead + Budget variance = $84,800 + $7,200 = $92,000

Actual variable overhead = Total actual overhead – Actual fixed overhead = $262,500 - $92,000

126 . Answer (A) is correct. Two-variance analysis considers only budget (controllable) and volume variances. When actual and budgeted fixed overhead are equal, the budget (controllable) variance equals the difference between actual variable overhead and standard hours allowed times the variable overhead rate per hour. Thus, the variance is $3,000 favorable [(49,500 x $6) - $294,000]. A favorable variance results when actual is less than standard. Answer (B) is incorrect because $6,000 results from using actual DLH. Answer (C) is incorrect because $9,000 is the difference between standard and actual DLH, times the variable overhead rate per hour. Answer (D) is incorrect because $9,000 is the difference between standard and actual DLH, times the variable overhead rate per hour.

242 . Answer (C) is correct. The flexible budget provides for a cost of $1,000 per article ($10,000,000 ÷ 10,000 articles). Each article should require 20 hours of labor (200 hours ÷ 10 articles). Thus, the standard labor rate is $50 per hour ($1,000 ÷ 20 hours), and total standard variable labor cost is $9,500,000 (9,500 articles x 20 hours x $50 per hour). Accordingly, total expected costs are $10,100,000 ($9,500,000 + $600,000 FC). Answer (A) is incorrect because $9,500,000 equals variable labor costs only. Answer (B) is incorrect because adding the $600,000 of fixed costs to the $9,500,000 of variable labor costs produces a total cost of $10,100,000. Answer (D) is incorrect because labor costs will decline as production declines, but fixed costs will not.

243 . Answer (A) is correct. Budgeted fixed costs are $600,000. The actual fixed costs were $618,000 ($9,738,000 total costs - $9,120,000 flexible costs). Because actual costs were $18,000 higher than the budget, the variance is unfavorable. Answer (B) is incorrect because the variance was unfavorable. Answer (C) is incorrect because $48,000 is based on a false presumption that fixed costs will be less at a 9,500 production level than a 10,000 level. Answer (D) is incorrect because the variance was unfavorable.

244 . Answer (A) is correct. The labor efficiency variance is the difference between standard and actual hours, multiplied by the standard cost per hour. The standard labor rate is $50 per hour, and the standard time allowed for 9,500 articles is 190,000 hours (9,500 x 20). Actual hours worked totaled 192,000. Thus, an unfavorable variance of 2,000 hours occurred. The unfavorable labor efficiency variance was therefore $100,000 ($50 x 2,000 hours). Answer (B) is incorrect because $238,000 is the difference between total (fixed + variable) standard labor cost ($9,500,000) and total actual cost ($9,738,000). Answer (C) is incorrect because the variance is unfavorable. Answer (D) is incorrect because the efficiency variance is based on standard hours for actual production levels--in this case 190,000 hours.

CMA EXAMINATION QUESTIONS Page 142 of 143